面白い問題おしえて〜な 七問目

1 名前:132人目の素数さん投稿日:03/10/01 01:58
面白い問題、教えてください


2 名前:1投稿日:03/10/01 01:59
過去スレ

[1]面白い問題教えて
http://cheese.2ch.net/math/kako/970/970737952.html
[2]面白い問題教えて 第2版
http://natto.2ch.net/math/kako/1004/10048/1004839697.html
[3]面白い問題おしえてーな
http://science.2ch.net/math/kako/1026/10262/1026218280.html
[4]面白い問題おしえて〜な 四問目
http://science.2ch.net/math/kako/1044/10441/1044116042.html
[5]面白い問題おしえて〜な 五問目
http://science.2ch.net/math/kako/1049/10495/1049561373.html
[6]面白い問題おしえて〜な 六問目
http://science.2ch.net/test/read.cgi/math/1057551605/

3 名前:132人目の素数さん投稿日:03/10/01 02:04
>>1
もつ

4 名前:1投稿日:03/10/01 02:08
>>3
埋まってしまうかとひやひやしました。数学板なのにw

5 名前:supermathmania ◆ViEu89Okng 投稿日:03/10/01 13:25
これは簡単かな?
Σ_{k=0}^{n-1}exp(2πik/n)=0の成り立つ正整数nの範囲を求めよ。

6 名前:132人目の素数さん投稿日:03/10/01 14:26
expってなんだっけ?

7 名前:132人目の素数さん投稿日:03/10/01 15:17
>>1
あんな空気の中でよく立ててくれた。
マリガトウ

8 名前:132人目の素数さん投稿日:03/10/01 16:37
>>6
経験値

9 名前:132人目の素数さん投稿日:03/10/01 19:01
しつこくしつこく3回目の出題。
 
n≧2とする。n次元ユークリッド空間E^nからE^nへの写像f:E^n→E^nが
 
 d(P,Q)=1⇒d(f(P),f'Q))=1
 
をみたすときfは等長写像であることを示せ。ただしd(P,Q)はPQの距離をあらわすとする。

10 名前:132人目の素数さん投稿日:03/10/02 09:09
模範解答示せゴルァ

11 名前:9投稿日:03/10/02 19:30
そだね。もう3回目だし。来週ぐらいに解答かきまふ。ではまた来週!
サヨナラ。サヨナラ。サヨナラ。

12 名前:132人目の素数さん投稿日:03/10/03 19:00
等長写像ってなに?

13 名前:132人目の素数さん投稿日:03/10/03 19:47
等距離写像。

14 名前:132人目の素数さん投稿日:03/10/03 22:21
>>11
d(P,Q)=d(f(P),f(Q))が任意のP,Qについて成立するような写像。

15 名前:132人目の素数さん投稿日:03/10/04 00:14
9 名前:132人目の素数さん 投稿日:03/10/01 19:01
しつこくしつこく3回目の出題。
 
n≧2とする。n次元ユークリッド空間E^nからE^nへの写像f:E^n→E^nが
 
 d(P,Q)=1⇒d(f(P),f'Q))=1
 
をみたすときfは等長写像であることを示せ。ただしd(P,Q)はPQの距離をあらわすとする。

16 名前:132人目の素数さん投稿日:03/10/04 01:19
>>14
サンクス

17 名前:132人目の素数さん投稿日:03/10/04 20:20
難しい問題ばっかりが面白い問題じゃないだろってことで、

∫[0,2π] √( 1-(cosθ)^2 )dθ

を求めよ。
--
即答しろって言われたら間違えるんじゃね?

18 名前:132人目の素数さん投稿日:03/10/04 20:36
ここはつまらない問題を書き込むスレですか?

19 名前:132人目の素数さん投稿日:03/10/04 20:53
ttp://math.josai.ac.jp/fun/indexj.html
一番下のが分からんねやけど誰か教えて

20 名前:132人目の素数さん投稿日:03/10/04 21:26
よこの2面づつ2組がそれぞれ1面づつになるから5面だとおもう。

21 名前:19投稿日:03/10/04 21:54
あーあー
なるほど。サンクス。

22 名前:132人目の素数さん投稿日:03/10/04 22:22
正四面体の各辺の中点を結ぶと一辺が1/2になった正四面体が4つとれる。
この4つを取り外すと正八面体が出来る。これを2つに分割した奴の1つと
それにくっつく1/2正四面体1つを合わせた物が>>19のリンク先物体になるな。

23 名前:132人目の素数さん投稿日:03/10/07 01:14
平面上に凸図形がある。その周の長さはL、その面積はSである。
いま、この図形のほかに、二定点A,Bをとる。動点Pをとり、∠PAB=θとする。
APに平行な異なる二直線を引き、その両方が与えられた凸図形に接しているとする。
このとき、この異なる二直線の距離をf(θ)と置く。

∫f(θ)dθ

の値を求めよ。
----

簡単かな?

24 名前:132人目の素数さん投稿日:03/10/07 01:15
>>23
訂正。

∫[0,2π] f(θ)dθ。


積分範囲書き忘れた。

25 名前:132人目の素数さん投稿日:03/10/07 03:57
>>19
5面体になることの証明

http://ebizen.hp.infoseek.co.jp/cgi-bin/img-box/img20031007035243.jpg

26 名前:132人目の素数さん投稿日:03/10/07 22:14
正方形をどの2つも合同でない3つの相似な図形に分割せよ.

27 名前:132人目の素数さん投稿日:03/10/07 23:09
>>26
http://w2.oekakies.com/p/2chmath/p.cgi
の85です

28 名前:132人目の素数さん投稿日:03/10/08 02:25
>>26
答えは無限にある

29 名前:132人目の素数さん投稿日:03/10/08 08:00
>>28
それを説明しないと…。
>>28は、ゼミで「明らかである」と言って教授に突っ込まれるタイプ。

30 名前:132人目の素数さん投稿日:03/10/08 08:31
円周率の小数点以下10^10000桁目を求めよ(もち10進法な)。
できたら俺氏んでやる

31 名前:投稿日:03/10/08 08:56
>>26
直角3角形Aをつくるため対角線で切ると残りは
Aと掃除になるように気ると合同になる
(残りの直角の点から垂線を下ろして直角作ると
二分する各と二編同じ→合同)
また、Aに相似な物は頂点を切るしかないのに、
他の頂点では火掃除、頂点外では4角形。
(以上は三平方の証明の応用)

四角形に切るとき(最初の切り方は3,4角形しかない)
斜線で切ると、次も斜線で切らなければ行かず、
直角が2つ付く偏が出来ないので却下。

真直ぐ垂直に切るとき
どれくらいきる化を
一回目a
二回目b
つまりひは、
1:a
1-a:b
1-a:1-b
の三つ、ただし順序は以下で考える
ひをa(=a/1)としてやる(もちろん0<a<1 かつ 0<b<1)

i)a>bのとき

(√5-1)/2 < a < 1のとき
二回目の時に合同になるのでボツ(ちなみに日は2/3)
0=a^3-a^2+2a-1 なるaが
1/2 < a < (√5-1)/2
の範囲であって、合同名別け方じゃなければいいが
計算できないので、まかせます。(b=(2a-1)/a)

32 名前:投稿日:03/10/08 08:56
つづき
ii)b>aのとき(b=1+a(a-1))
同じ式
0=a^3-a^2+2a-1
がでてきて、(0<a<1)の範囲で買いがあって、
だいじょぶならおk
グラフで描かせたら、0.5以外でありそうなので、
二回目で別けたやつ同士は合同じゃないけど、
他が・・・・
3次式解ける人検算お願いします

33 名前:132人目の素数さん投稿日:03/10/08 15:01
>>29
実数x, yを、x^2+y^2-x-y+xy=0 を満たすように取る。
ただし0<x<y<1 とする。1辺1の正方形ABCDで、

ADを x:1-x に内分する点をE
BCを y:1-y に内分する点をF
CDを (1-y)/x:(1-x)/y に内分する点をG

とし、EFを結び、GからADに平行な直線を引いて
それがEFと交わる点をHとする。

こんな感じでどうかね。
たとえば x=3/7、y=6/7 などがその一例。

34 名前:NightKingOfMath ◆p38EzHwbPY 投稿日:03/10/08 15:21
30へ、こういう問題にはこんな回答を出す人が居る。
[π*10^10000]-10*[π*10^9999]
30はどうするのか?

35 名前:KingOfMath ◆p38EzHwbPY 投稿日:03/10/08 15:25
今は昼だ。
さて、111111111111111111111111111を27で割ったときのあまりを0以上27未満の整数で答えよ。
制限時間は180秒間。(30秒もあればできるだろう。)

36 名前:132人目の素数さん投稿日:03/10/08 17:48
>>30
小心者め。

37 名前:132人目の素数さん投稿日:03/10/08 17:53
>>35
111*1001001001001001001001001

もっと簡単なやり方ある?

38 名前:132人目の素数さん投稿日:03/10/08 18:40
>>34
[π*10(10^10000)]-10*[π*10^((10^10000)-1)]?


39 名前:132人目の素数さん投稿日:03/10/08 18:42
111-108=3を使えば。
0

40 名前:132人目の素数さん投稿日:03/10/08 18:55
(1)円を相似だが合同でない2つの図形に切り分けよ

(2)正三角形を相似だが合同でない2つの図形に切り分けよ

(3)正方形を相似だが合同でない2つの図形に切り分けよ

お絵かき掲示板
http://w2.oekakies.com/p/2chmath/p.cgi

41 名前:132人目の素数さん投稿日:03/10/08 19:03
>>40
http://w2.oekakies.com/p/2chmath/31.png?81

42 名前:132人目の素数さん投稿日:03/10/10 20:32
沈んでいるので問題を。数ヲタには簡単だろうけど…

任意の正の数a,b,c,d,e,fに対して、次の不等式を示せ。
ab/(a+b) + cd/(c+d) + ef/(e+f) ≦ (a+b+c)(d+e+f)/(a+b+c+d+e+f)

43 名前:132人目の素数さん投稿日:03/10/10 21:11
. .: : : : : : : : :: :::: :: :: : :::::::::::::::::::::::::::::::::::::::::::::::::::::::::::
    . . : : : :: : : :: : ::: :: : :::: :: ::: ::: ::::::::::::::::::::::::::::::::::::::
   . . .... ..: : :: :: ::: :::::: :::::::::::: : :::::::::::::::::::::::::::::::::::::::::::::
        Λ_Λ . . . .: : : ::: : :: ::::::::: :::::::::::::::::::::::::::::
       /:彡ミ゛ヽ;)ー、 . . .: : : :::::: :::::::::::::::::::::::::::::::::
      / :::/:: ヽ、ヽ、 ::i . .:: :.: ::: . :::::::::::::::::::::::::::::::::::::::
      / :::/;;:   ヽ ヽ ::l . :. :. .:: : :: :: :::::::: : ::::::::::::::::::
 ̄ ̄ ̄(_,ノ  ̄ ̄ ̄ヽ、_ノ ̄ ̄ ̄ ̄ ̄ ̄ ̄ ̄ ̄ ̄ ̄
       わからん・・・

44 名前:132人目の素数さん投稿日:03/10/10 21:47
円周率の小数点以下10^10000桁目を求めよ(2進法な)。
できたら俺氏んでやる



45 名前:42投稿日:03/10/10 22:17
           ,..-‐−- 、、
          ,ィ":::::::::::::::::::;;;;;iii>;,、
          /:::::::::::::::;;;;;;;;iii彡" ::ヤi、
        ./::::::::::::;:"~ ̄     ::i||li
        .|::::::::::j'_,.ィ>、、   .:::iii》  
        ヾi´`,  `‐-‐"^{"^ヾノ"  
         Y     ,.,li`~ ~i   そろそろ時間だ!
          i、   ・=-_、,  :/    答えを聞こうか!
          |ヽ    '' ..:/
          |  ` rー、.,ノ__
     /`ーヘl丶ー--‐ l |\ ̄ ニ-、
    ノ、ノ^⌒へ\ー--‐' /,_ \    \
   /⌒  ,◎、  \ / | :  ̄     \
  /::::    /|_.|イ-、 、V  ̄  : |     \
  >-― __/、ニEl(,,ノ : |o  i : o
 (   / 〈 ニニノ : |   ``'''―'⌒
  \|   _ーノ  : |
   \`ー´/ ̄   :|

46 名前:132人目の素数さん投稿日:03/10/10 22:17
勘で0

47 名前:132人目の素数さん投稿日:03/10/10 22:26
値を示す問題じゃないやーーん

48 名前:132人目の素数さん投稿日:03/10/10 22:46
勘で1

49 名前:42投稿日:03/10/10 22:58
r;;;;;ノヾ 
ヒ‐=r=;'  
ヽ二/    茶でも飲みながら、3分間待ってやる!
(、っiョc)
゙'ー'゙ー'

50 名前:132人目の素数さん投稿日:03/10/10 22:59
44と勘違いしていた。
スマソ

51 名前:132人目の素数さん投稿日:03/10/10 23:00
.       ∧_∧  / ̄ ̄ ̄ ̄ ̄ ̄ ̄ ̄ ̄ ̄ ̄ ̄ ̄ ̄ ̄ ̄
       (;´Д`)< スンマセン、直ぐに片付けます
  -=≡  /    ヽ  \________________
.      /| |   |. |
 -=≡ /. \ヽ/\\_  r;;;;;ノヾ
    /    ヽ⌒)==ヽ_)= ヒ‐=r=;'
-=   / /⌒\.\ ||  ||  ヽ二/  どこへ行こうというのかね?
  / /    > ) ||   || ( つ旦O
 / /     / /_||_ || と_)_) _.
 し'     (_つ ̄(_)) ̄ (.)) ̄ (_)) ̄(.))

52 名前:42投稿日:03/10/10 23:27
          |     |        |
          |     |        |
         │      ̄ ̄ ̄ ̄ ̄
      r;;;;;ノヾ
      ヒ‐=r=;'   誰も解かないのか…
      ヽ二/         
        (∩∩)────────────────
      /
    /
  /!

53 名前:132人目の素数さん投稿日:03/10/11 03:04
<血液型O型のいかがわしい特徴> ( なんでも気を許していると調子に乗ってつけこんでくるぞ! )
■その道の達人をコケにして安心しようとする。多数派のときだけやたら強気。才能ある人間を歪んだ目でしか見れない。
■場所が変わると手のひらを返すように態度が豹変する ( あの連中の中じゃヘコヘコさせられてきたが、うちに来たら見ていろ! 礼儀を教えてやる。 )
■読みが浅い。粘着に睨まれる事を言っておいて関係ない人に振り向けバックレようとする姑息さ ( どうせ>>1さんはヒッキーでしょ? / 関西人だってジサクジエンしてるぜ )
■早とちりが多く、誤爆と気付いても釈明できない。イイ人を装って忘れる ( そういう土地柄だったのさ )  他人の勘違いは許せない。
■話題と関係ない個人攻撃で場の空気を険悪にする。なぜか次第に自分から耐え切れなくなりあきれて話題をコロコロ変える。
■いつの間に恩を貸してるように振舞うが、自分のためであると見抜かれる。恥ずかしさをごまかすために大袈裟にキレるが相手がキレると今度はビビる。
■顔が見えない相手と喋っているのに毒舌かつ強気。面と向かった相手には他人が言ったことに。 ( 俺が言ったんじゃないけど、Kさんがあなたをアホかとバカかと… ヒヒ) ネット弁慶。
■無理な人の真似をしケチをつけ、批判される所には現れない。観察される側になると極度に焦る( 直接対決が苦手。言い返されにくい状況を計算する )
■自分が企んでいるのに相手を悪人に仕立て上げて餌食にする。わざと攻撃を誘って自分の憎悪に酔いしれる。人の裏切りを許せないわりに自分の裏切りは覚えていない鈍感さ。
■世間に通用しないような事ばかりしつこく繰り返して自分に課した目標から逃げる。あげくのはてに相手にわからない方法でキレる。
■半端な人生経験で集団的にばかり相手を気にする ( 君のような人はウチでは… / ○さんによく似てるから駄目だろうね / 背が高い奴はどいつも… )


54 名前:132人目の素数さん投稿日:03/10/11 03:58
>>50>>47と考えられる。以上っ!

55 名前:132人目の素数さん投稿日:03/10/11 19:18
  ||
/ヘ;;;;;  だれも挑戦してくれないのか…
';=r=‐リ  
ヽ二/ ヽ
 | |   | 
 ∪ / ノ   
  | || 
  ∪∪
   ;
 -━━-

56 名前:132人目の素数さん投稿日:03/10/11 21:24
2n+1個の整数 a(1),a(2),…,a(2n+1)を次の性質をみたす整数の集合とする。
性質:これらの整数のどの1つを除いても,残りの2n個の整数は,2つのn個の整数の集まりに分解でき,それらの和が一致する。

このとき, 2n+1個の数は全て等しい事を示せ。

57 名前:132人目の素数さん投稿日:03/10/11 21:31
で結局、ムスカは無視かよ?

58 名前:132人目の素数さん投稿日:03/10/11 21:37
>>55
不等式の問題か?

まじめに分からんので解説してくれ

59 名前:132人目の素数さん投稿日:03/10/11 22:07
二進法での円周率の小数点以下
10^(10^(10^(10^10)))桁目を求め、
どちらかの扉に進めと問題を出されて、
扉が2つある。片方は0 片方は1
どちらかは天国。どちらかは地獄へ通じている。
あなたは未来における、人類史上最強の数学者です。
制限時間は予め決められてます。
選択肢は3つあります。どれかを選ぶまでは時間はカウントされません。
悪魔は心を読めるので、どれかを選択してるとみせかけてその間、
解くのに時間をかけることはできません。解きだしたら@と見なされカウントされ出します。
一旦どれかに決定したら後から変更は出来ません。
@解けるまであがきつづける。(制限時間に間に合わないと悪魔に強制的に地獄送りされます)
A勘でどっちかに賭ける
Bあがくなんて無駄。諦めて悪魔に地獄に放り込んでもらってスッキリ

60 名前:132人目の素数さん投稿日:03/10/11 22:30
Cを選ぶ

61 名前:132人目の素数さん投稿日:03/10/11 22:43
二進法で10^(10^(10^(10^10)))桁目だろ
10^(10^(10^(10^10)) = 10^(10^(10^100)) = 10^(10^1000) = 10^10000 = 100000
十進法に直せば32じゃねーか。 制限時間によっては@を選ぶよ。

62 名前:132人目の素数さん投稿日:03/10/11 22:46
>>61はあほ

63 名前:132人目の素数さん投稿日:03/10/11 22:51
二進法で指数計算てどうやんの?
初めて見た

64 名前:132人目の素数さん投稿日:03/10/11 23:25
>>61
二進法だったとしても、

20035299304068464649790723515602557504478254755697514192650169737108940595563114
53089506130880933348101038234342907263181822949382118812668869506364761547029165
(19440桁ほど略)
59888384311451189488055212914577569914657753004138471712457796504817585639507289
5337539755822087777506072339445587895905719156736桁目の計算なのですが?

65 名前:132人目の素数さん投稿日:03/10/11 23:45
>>56
問題の前提となっている性質は
任意の定整数kを一つ取って,各a(i)のそれぞれからkを引いたa(i)-k についても
成り立つ性質である.必要なら順番を入れ替えてa(1)≦a(2)≦...≦a(2n+1)
としておいてよい.今,kとしてa(1)を選び,各iについてa(i)-kをあらためてa(i)とすると
2n+1個の整数 0,a(2),a(3),...,a(2n+1) がまた問題の性質をもつ.
さて,a(2),...,a(2n+1)を適当にn個づつ2組に分けそれぞれの和をつくると
等しくなるから,a(2)+a(3)+・・・+a(2n+1)は偶数である.
ここで任意のa(i)をa(1)=0で置き換えても同様の理由から偶数であるから,
どのa(i)も偶数であることがわかる.すべてのa(i)が偶数であるから
それぞれを2で割ったものをあらためて,a(1)=0,a(2),・・・,a(2n+1)とすると
やはり同じ性質をもつ.するとa(1)=0だから,やはりa(2)以下すべては偶数である.
よってa(i)の中に0でないものがあるなら,結局どれかのa(i)が奇数になるまで
割っていくことができ,それら全体がやはり問題の性質をもちつつ
相変わらずa(1)=0なのだから,これは矛盾である.
よって,最初にk=a(1)を引いた時点ですべては0になっている.
つまり,a(i)同士はみな等しい.


66 名前:132人目の素数さん投稿日:03/10/12 00:38
http://www.2ch.net/tako/comic/kako/958/958465192.html

67 名前:132人目の素数さん投稿日:03/10/12 02:47

■■    ■■
  ■■ ■■

の2つのパターンのみがランダムに現れるテトリス。
横のサイズは10、縦のサイズは20とする。
出現するパターンの分かるものは、今落ちているパターンの他に次に落ちてくるパターンのみとする。

有限時間内に負けない方法があるのならばその手順を、
有限時間内に必ず負けるのであればその証明を示せ。

68 名前:67投稿日:03/10/12 03:09

■■ ■■■■
■■

の2パターンのみがランダムに出現する場合はどうか?

69 名前:Galois投稿日:03/10/12 03:22
<<67
有限時間内に負けない方法がある と
有限時間内に必ず負ける では すべての場合を覆えない。
有限時間内に負けない方法がある と
有限時間内に(ブロックの出現のある場合に対しては)負ける可能性
がある ではないでしょうか?
67は難しいが
68では有限時間内に負ける可能性がある

70 名前:Galois投稿日:03/10/12 04:39
例えば、■■■■がずっと出た場合。テトリスは消えない
なぜなら、横10列より。 証明終わり

71 名前:67投稿日:03/10/12 04:46
>>69
>有限時間内に(ブロックの出現のある場合に対しては)負ける可能性がある

これを「有限時間内に必ず負ける」と表現したつもりです。
無限にゲームを行い続けようとしても高々有限時間内に必ず負ける、と。

>>70
えっと、テトリスなんで回転は出来ますが。

72 名前:Galois投稿日:03/10/12 04:52
失礼。
それで68は負けない方法はありますね。
厳密な証明はまだ考えてないですが。
また考えてみます。

73 名前:132人目の素数さん投稿日:03/10/12 07:01
経験上、深さが20段あれば、どんなブロックがどんなパターンで
降ってきても負けることはない。

もちろん、思考時間と操作時間が十分与えられた場合の話ね。
問題は単純なアルゴリズムでそれを実現することか…。

あと、どう配置しても必ず負ける降らせパターンが存在するような、
深さの最大値はいくつになるのか興味がある。

74 名前:132人目の素数さん投稿日:03/10/12 07:19
>>73
高々数百回数千回の経験でもって
無限に続けられると結論付けるのは数学屋としてどうか。

75 名前:pam投稿日:03/10/12 08:53
★エイト・クイーンズの解
□□□□□□□□ エイト・クイーンズというひとり遊びのゲームがある。 
□□□□□□□□ ルールは、
□□□□□□□□  ◆8×8のチェス盤の上に8つのクイーンを配置する。
□□□□□□□□  ◆クイーン同士がお互いをとることのできない位置に配置する。
□□□□□□□□   クイーンは縦、横、斜めにどこまでも移動可能。
□□□□□□□□   つまり、クイーンを置いたマスの縦、横、斜めのライン上には
□□□□□□□□   他のクイーンがあってはならない。
□□□□□□□□
例えば、
□□□□□□●□
□□□□●□□□ このように、配置する。
□□●□□□□□ 
●□□□□□□□ さて、このゲームの解は何通りもあるが、
□□□□□●□□ (1)一番隅のマスにクイーンを配置した解は存在するか?
□□□□□□□● 
□●□□□□□□ (2)解は全部で何通りあるか? 
□□□●□□□□    ただし、回転、反転して同じ解になるものは全てあわせて一通りとする。

76 名前:132人目の素数さん投稿日:03/10/12 09:18
>>73
普通にゲームした場合には同じブロックが数十回連続で出てくることなんてまず無いでしょ。
でも数学的に無限に続けられるかどうかを論じるためにはそのようなケースも考慮しないと駄目。

77 名前:132人目の素数さん投稿日:03/10/12 09:19
>>72
>>68の負けない方法教えてー

78 名前:132人目の素数さん投稿日:03/10/13 01:05
>>68って負けない方法ある?
いや、一見負けそうにはないんだけど、どうも手順が見つからないような

79 名前:132人目の素数さん投稿日:03/10/13 06:49
0≦p,q,r,s≦1 の実数p,q,r,sを考え
x=p-q 、 y=r-s とする。このとき、x,yのとりうる値をx-y平面上に図示せよ。
---

図示すると形がきれいになるので出題。

80 名前:132人目の素数さん投稿日:03/10/13 06:53
>>79
忘れてた。
p+q+r+s=1
の条件が抜けてる。。。

81 名前:132人目の素数さん投稿日:03/10/13 09:55
|x+y|≦1,|x-y|≦1

82 名前:132人目の素数さん投稿日:03/10/13 11:01
>>79-80
|x|+|y|≦1

83 名前:132人目の素数さん投稿日:03/10/13 19:53
毎時(1時台、2時台・・・)必ず1本だけバスが来る停留所があります。
ただし何時台のバスもその時間の何分に来るのかまでは分かりません。
何分に来るのかは各時間ごとに全くのランダムです。

さて、時計を見ずに適当な時間にこのバス停に来た人の
バスの平均待ち時間は何分でしょうか?

84 名前:132人目の素数さん投稿日:03/10/13 20:21
30分

85 名前:132人目の素数さん投稿日:03/10/13 20:53
>>84
ちゃう

86 名前:132人目の素数さん投稿日:03/10/13 21:14
>>83
(1分+1時間59分)/2= 1時間

答え:1時間

これで満足か?



87 名前:132人目の素数さん投稿日:03/10/13 21:17
>>86
意味不明

88 名前:132人目の素数さん投稿日:03/10/13 21:20
>>87
答えが合ってるんだからいいだろ。禿げ。
具体的に説明したところでおまえのような奴はいいがかりつけるだけだろ。

89 名前:132人目の素数さん投稿日:03/10/13 21:22
>>88
答えも間違ってんだよ禿げ

90 名前:132人目の素数さん投稿日:03/10/13 21:31
25分

91 名前:132人目の素数さん投稿日:03/10/13 21:36
>>90
ちゃう

92 名前:132人目の素数さん投稿日:03/10/13 21:39
35分だっ。

93 名前:132人目の素数さん投稿日:03/10/13 21:42
50分

94 名前:132人目の素数さん投稿日:03/10/13 21:42
>>92
せーかい!

95 名前:132人目の素数さん投稿日:03/10/13 21:49
35分

96 名前:132人目の素数さん投稿日:03/10/13 21:49
x分に人が来たとすると、
1)その前にバスが来ていない確立は 1-x/60
そのときの平均待ち時間は、(60-x)/2

2)その前にバスが来ている確立はx/60
そのときの平均待ち時間は(90-x)

和をとって、-x^2/120 +x/2+30
Σ(-x^2/120 +x/2)/60=5(0<x<59)
30+5=35

97 名前:95投稿日:03/10/13 21:50
ああ遅かったか

98 名前:132人目の素数さん投稿日:03/10/13 22:12
毎分1/60の確率でバスが来る停留所があります。

このバス停に来た人の
バスの平均待ち時間は何分でしょうか?

だったら1時間になるけど
低脳にはそれがわからなかったのね(プクス



99 名前:132人目の素数さん投稿日:03/10/13 22:19
> 毎分1/60の確率でバスが来る停留所があります。

いやなバス停だな

100 名前:132人目の素数さん投稿日:03/10/13 22:19
>>98も低脳

101 名前:132人目の素数さん投稿日:03/10/13 22:23
>>96
>和をとって、-x^2/120 +x/2+30
>Σ(-x^2/120 +x/2)/60=5 (0<x<59)

積分したほうが良さげ。

x分からx+dx分の間に人が来る確率はdx/60

よって
S (-x^2/120 + x/2 + 30) dx/60 (0から60まで) = 35

102 名前:132人目の素数さん投稿日:03/10/14 00:04
>>73
単純に
■■
  ■■
のみを無限に落とし続ければ、プレイヤーは負ける。
高さが何段あってもだ。

103 名前:102投稿日:03/10/14 00:16
ごめん、そうでもないな。

104 名前:132人目の素数さん投稿日:03/10/14 00:34
  ■  ■  ■  ■  ■
■■■■■■■■■■ 列がそろったら
■  ■  ■  ■  ■

      ↓

  ■  ■  ■  ■  ■ そろった列が消えて
■  ■  ■  ■  ■   消えた分だけ落ちる

だよね、テトリスは。
# ボンブリス(知ってる奴いる?)だと確か全部消えてしまう

105 名前:132人目の素数さん投稿日:03/10/14 00:38
>>104の下図の後は、
まさに>>104の上図のパターン通りに下図にブロックを置けば、
2ライン消えて再び下図に戻るね。

これは無限に負けない。

106 名前:132人目の素数さん投稿日:03/10/14 17:19
>>83と似てるんだけど、基本問題で、

「ランダムだが平均して1時間に1本バスがくる停留所がある。
バス停にきた人が1時間以内にバスに乗れる確率は?」

これ、知らない人が結構いるみたい。

107 名前:132人目の素数さん投稿日:03/10/15 15:27
>>106
お客がバス停に来た時点を時刻0として時間軸を取り、
この時点から先T時間の様子を考える。この間に
T本のバスが来て、それらは独立で、時間 [0,T] 内に
一様に分布しているとしてよい。

このとき、ある1本のバスが時間 [0,1] 内に来ない確率は
(T-1)/T で与えられる。これがT本あるから、時間 [0,1] 内に
1本もバスが来ない確率は((T-1)/T)^Tとなる。

ここでT→∞とすればこれは1/eに収束する。
よって求める確率は 1-(1/e)

108 名前:106投稿日:03/10/15 15:39
>>107
正解です。そして、計算するとわかりますが、
意外?にも待ち時間の平均が1時間になるんですよね。
いわゆるポアソン過程というやつです。

>>83の「1時間にランダムに1本」を「N時間にランダムにN本」として、
さらにN→∞とした場合に対応します。

109 名前:132人目の素数さん投稿日:03/10/15 15:40
>>9をだいぶ前(初回の出題時)から暇を見ては考えているんだが
上手い方針が全く思いつかない。ヒントくれよ!

せめて

・fの連続性
・fの全単射性
・d(P,Q)=1⇔d(f(P),f'Q))=1

のどれか1つでも示せればだいぶ違ってくるんだが。

110 名前:132人目の素数さん投稿日:03/10/15 22:23
>>109
おお、考えてるしといたのか。とりあえずn=2の場合やってみると高次元化は簡単。
n=2の場合の証明のポイントは
(I) d(P,Q)=√3⇒d(f(P),f(Q))=√3 or 0を示す。
(II) d(P,Q)=√3⇒d(f(P),f(Q))=√3 を示す。
(III) あるe<1が存在しd(P,Q)=e⇒d(f(P),f(Q))=eをしめす。
(IV) 等長写像であることをしめす。
でいける。(I)はかんたん。(I)⇒(II)はd(P,Q)=√3である2点にたいしある2等辺3角形を
考えるといける。(II)⇒(III)は3辺のながさが1の正三角形と3辺の長さが1,1,√3の3角形を
重ねたものをつかう。(III)⇒(IV)は3角不等式。
(II)⇒(III)、(III)⇒(IV)の証明のポイントは
d(P,Q)=e⇒d(f(P),f(Q))=e (∀P,Q)が成立するときd(P,Q)≦ne (n≧2)⇒d(f(P),f(Q))≦ne
がなりたつことを利用することがポイント。

111 名前:109投稿日:03/10/16 21:14
サンクス。(II)までは俺も到達してたんだけど、
そこから先が続かないのでこの方針ではダメかと思ってた。
ヒントを元にもうちっと考えてみるわ。

112 名前:132人目の素数さん投稿日:03/10/17 22:15
平面上の点集合Sについて、Sのどの二点を取ってもその距離が1を超えないとする。
関数f:S→Sを考え、
S∋A,Bならば、
d(A,B)≦d(f(A),f(B))
が成立している。 このとき、任意のS∋A,Bに対して
d(A,B)=d(f(A),f(B))
を示せ。


最近、少ないね。ネタ切れか?

113 名前:132人目の素数さん投稿日:03/10/17 22:36
>>112
なんか一瞬自明に見えた

114 名前:132人目の素数さん投稿日:03/10/18 00:18
テトリスの証明しよう

115 名前:132人目の素数さん投稿日:03/10/18 00:19
テトリスが数オタノ心をがっしりキャッチすることの証明か。

116 名前:132人目の素数さん投稿日:03/10/18 00:40
>>112
反例みつからんか?

117 名前:132人目の素数さん投稿日:03/10/18 20:40
>>112
ヒントくれアゲ

118 名前:132人目の素数さん投稿日:03/10/18 21:41
>>117
S⊂T を満たす点集合Tを考える。
Tをある特殊な集合として、さらに写像fにも必要に応じて、回転、平行、対称移動を加える。
すると、あら不思議 TとSの差集合に属する T-S∋X 任意の点Xに対して、T-S∋f(X)が成立してしまいます。

ポイントとしては、写像fはある程度いじっても距離には影響を与えない所
あとは。。。Tをどう考えるか。

119 名前:132人目の素数さん投稿日:03/10/18 21:44
S上の写像fがT上に拡張できる根拠は?

120 名前:132人目の素数さん投稿日:03/10/18 21:48
>>119
それも問題を解きながら考えてくれ。

121 名前:132人目の素数さん投稿日:03/10/18 21:52
>>118
まったく意味わからないんだけど・・・Tぐらいおしえてよ。

122 名前:132人目の素数さん投稿日:03/10/18 21:52
>>112
なんか、出題者の方が間違ってる気がするが・・・一応ヒント通りにやってみるか。

123 名前:132人目の素数さん投稿日:03/10/18 22:02
Tぐらいおしえてくれあげ

124 名前:132人目の素数さん投稿日:03/10/18 22:05
問題文の S, f は固定ですよね?

125 名前:132人目の素数さん投稿日:03/10/18 22:10
十分に大きい実数Rに対して、半径Rの円盤TはS⊂Tを満たすとして良い。
x-y座標を導入し、点集合Tをx^2+y^2≦R^2と定義して以下の議論を進める。
ちなみに、俺には写像fをSからT上に拡張する方法が分からないので、写像fは無条件にT上で考えても良いとする。だって、わからないんだもん。

A=(R,0)  B=(-R,0) の二点A,Bに対して、d(A,B)=2R さらに、f(A)、f(B)∈Tより、d(f(A),f(B))≦2R
よって、d(f(A),f(B))=2R。この事から、f(A)=( Rcosα , Rsinα )  f(B)=( Rcos(α+π) , Rsin(α+π) )
が成立し、写像fに回転を加えると、一般性を失わずf(A)=A f(B)=Bとしてよい。
さらに、C=(0,R) D=(0,-R) とすれば、上と同様にして、f(C)=C f(D)=D またはf(C)=D f(D)=Cが得られ、ここでも写像fに必要に応じて対称移動を加える事で、f(C)=C f(D)=Dとして良い事が分かる。

さて、T∋Xの任意のXに対して条件より
d(A,X) ≦d(A,f(X))
d(B,X) ≦d(B,f(X))
d(C,X) ≦d(C,f(X))
d(D,X) ≦d(D,f(X))
が成立する。このような条件を満たすのはf(X)=Xのみである。従って、f:T→Tは恒等写像となる。

さて、俺にはここが限界なのだが・・・

126 名前:132人目の素数さん投稿日:03/10/18 22:14
f:T→T が恒等写像なので、X,Y∈S⊂Tの任意のX,Yについても
d(X,Y)=d(f(X),f(Y))
が成立する。


思いっきり間違いつつもQ.E.D.

誰か補完キボンヌ

127 名前:132人目の素数さん投稿日:03/10/18 22:27
>>125
かいてある部分については大体ただしそうだけどこの方針でいくならS,fを条件をみたす
ものとするときあるO∈E^2とR>0とTとgで
(1)S⊂T、g:T→T、g|S=f、d(A,B)≧d(g(A),g(B)) (∀A,B∈T)
(2)∀A∈T、d(O,A)≦R
(3)∃A,B,C,D∈T d(A,O)=d(B,O)=d(C,O)=d(D,O)=R
をみたすものがとれる。がいえればよいと。この方針でいいの?>出題者

128 名前:132人目の素数さん投稿日:03/10/21 14:50
円形の台紙の中央に針が一本ついてて、その針をくるくる回すルーレットのようなものを考える。
台紙の12時の位置に印を付けておく。
誰か他人に自分の見えないところで適当にランダムに針を回させて、
台紙の12時の位置から針の止まったとこまでを時計回りに黒く塗らせた。
(これで台紙が領域黒と領域白に分かれたことになる)

準備完了。

(1)さて、こうしてできたルーレットを、
  やはり自分の見えないところで誰かに1回まわさせたとき、
  針が領域黒に止まる確率は?

(2)さて、こうしてできたルーレットを、
  やはり自分の見えないところで誰かに2回まわさせたとき、
  針が2回とも領域黒に止まる確率は?

129 名前:132人目の素数さん投稿日:03/10/21 15:13
>>128
領域黒の割合b (0<=b<=1)として、
領域黒がbとなる密度関数p(b)=1、
領域黒に針が止まる確率はbなので
(1)\integral_{b=0}^{1} bp(b)db = 1/2
(2)\integral_{b=0}^{1} b^2p(b)db = 1/3

130 名前:132人目の素数さん投稿日:03/10/21 15:23
>>129
やはり瞬殺ですか。

(2)の答えが(1)の二乗にならないのがちょっと面白い。
ちょっとだけね。

131 名前:132人目の素数さん投稿日:03/10/21 15:30
       ∧  ∧
       |1/ |1/
     / ̄ ̄ ̄`ヽ、
    /        ヽ
   /  ⌒  ⌒   |  
   | (●) (●)   | < 自称 有栖川です
   /          | 
  /           |
 {            |
  ヽ、       ノ  |
   ``ー――‐''"   |
    /          |
   |          | |
   .|        |  | |
   .|        し,,ノ |
   !、          /
    ヽ、         / 、
     ヽ、  、   /ヽ.ヽ、
       |  |   |   ヽ.ヽ、
      (__(__|     ヽ、ニ三



132 名前:129投稿日:03/10/21 16:52
>>130
じゃあ問題を追加してみたー

(3)さて、こうしてできたルーレットを、
  やはり自分の見えないところで誰かに1回まわさせたとき、
  領域黒に止まったことを教えられた。
  次の1回の試行で領域黒に止まる確率は?

133 名前:132人目の素数さん投稿日:03/10/21 17:23
2/3?

134 名前:132人目の素数さん投稿日:03/10/21 17:41
>>112
なんとなくできたかも。まわりくどい証明かもしれないけど。なんにせよおもろかった。
(解答)
集合Sと写像f:S→Sについての以下の条件をかんがえる。
Sは有界で∀A,B∈S d(A,B)≦d(f(A),f(B))・・・(※)
 
補題1
(S,f)が(※)をみたすとき、(T,g)でこれも(※)をみたしS⊂T、g|S=f、
Tはコンパクト集合となるものがとれる。
(∵)T=clS(Sの閉包)とおく。Tの各元Aにたいして点列P(A,i)をP(A,i)∈S、limP(A,i)=A、
limf(P(A,i))は収束、A∈SのときP(A,i)=Aをみたすように選ぶ。(Sが有界ゆえ可能)
そこでg(A)=limf(P(A,i))と定めると(T,g)が求める条件をみたすことがわかる。□
 
補題2
A1,A2,A3,A4,B1,B2,B3,B4がd(Ai,Aj)≦d(Bi,Bi)≦d、d(A1,A2)=d(A3,A4)=dをみたすとき
d(Ai,Aj)=d(Bi,Bj)
(∵)A1=B1=(0,0)、A2=B2=(d,0)として一般性を失わない。さらにy(A3),y(B3)>0としてよい。
(ここにx(P)はPのx座標、y(P)はPのy座標。)このときd(Ai,Aj)≦d(Bi,Bi)≦dより
y(A4),y(B4)<0もわかる。d(A1,A3)=p、d(A2,A3)=q、d(A1,A4)=r、d(A2,A4)=sとおきさらに
X={P|d(A1,P)≧p、d(A2,P)≧q、y(P)>0}、Y={P|d(A1,P)≧r、d(A2,P)≧s、y(P)<0}とおけば
A3,B3∈X、A4,B4∈Yであり簡単な計算からd(P,Q)≧d (∀P∈X,∀Q∈Y)、
d(P,Q)=d ⇔ P=A3、Q=A4(∀P∈X,∀Q∈Y)であることがわかる。よって仮定からA3=B3、A4=B4。□
 
系3
(S,f)が(※)を満たすときd=diamS=sup{d(P,Q)|P,Q∈S}とおき
X={P∈S|∃Q∈S d(P,Q)=d}とおくときf|Xは等長写像である。

135 名前:132人目の素数さん投稿日:03/10/21 17:42
補題4
Cが点Oを中心とする半径d/2の円でYをその周の稠密な部分集合とする。このときCの周または
内部の点Rに対しDの点Piを
∀S d(O,S)≦d/2,d(Pi,S)≧d(Pi,R)⇒S=R
が成立するようにとれる。
(∵)略
 
補題5
(S,f)が(※)を満たし、Sがコンパクトとする。このときあるnが存在しf^nが不動点をもつか
fは等長写像である。
(∵)d=diamS=sup{d(P,Q)|P,Q∈S}、X={P∈S|∃Q∈S d(P,Q)=d}とおく。f(X)⊂Xであり
補題3よりf|Xは等長写像である。f|Xがべき等、つまり(f|X)^n=idとなる自然数nが
存在するときはf^nが不動点をもつ。
そうでないと仮定する。f|Xはべき等でない等長写像なのである点Oとθ/πが有理数でない
実数θが存在しf|XはOを中心としてθだけ回転させる写像である。このときP,Q∈Xをd(P,Q)=d
となるようにとりY={f^n(P)}、Z={f^n(Q)}とおく。このときY,ZはそれぞれOを中心とする
半径d(O,P)、d(O,Q)の円の円周上の稠密な部分集合となる。よってこのとき
d(O,P)+d(O,Q)=sup{d(P',Q')|P'∈Y,Q'∈Z}≦dであるがd(O,P)>d(O,Q)と仮定すると
P',P''∈Zをd(P',P'')>d(O,P)+d(O,Q)をみたすようにとれるゆえdの定義に矛盾。
ゆえにd(O,P)=d(O,Q)=d/2である。この円の周または内部にないSの元Rがあるとすると
P'∈Yをd(P',R)>dととれるので矛盾。ゆえにSはこの円の周または内部に含まれる。
f|Xは等長写像ゆえg:E^2→E^2を等長写像でg|S=fとなるものととりh=g^(-1)fとおく。
h|Xは恒等写像となる。R∈Sをとるとき補題4よりP1,P2,P3,P4を
∀S d(O,S)≦d/2,d(Pi,S)≧d(Ai,R)⇒S=R
をみたすようにとれる。h(Pi)=Pi、d(Pi,h(R))=d(h(Pi),h(R))≧d(Pi,R)により
h(R)=Rである。∴f(R)=g(R)。つまりfは等長写像である。□

136 名前:132人目の素数さん投稿日:03/10/21 17:42
補題6
(S,f)が(※)をみたすとする。任意のP∈SにたいしX={f^n(P)|n≧0}とおくとき
f|Xは等長写像である。
(∵)TをXの閉方とする。補題1よりg:T→Tをf|Xの拡張で(T,g)が(※)をみたすようにとれる。
gが等長写像であることをしめせばよい。
補題5より自然数nでg^nが不動点をもつと仮定してよい。Rを不動点とする。
このとき0≦m<nおよび自然数列(ni)をlimg^(n・ni+m)(P)=Rとなるようにとれる。このとき
0=limd(g^(n・ni+m)(P),R)=limd(g^(n・ni+m)(P),g^(n・ni)R)≧d(g^m(P),R)
よりg^m(P)=Rである。非負整数lに対しl+r≡m (mod n)となる非負整数rをとれば
d(g^(l+n)(P),g^l(P))≦d(g^(l+n+r)(P),g^(l+r)(P))=d(g^(m+kn+n(P),g^(m+kn(P))=0
よりg^n|Y=idでありとくにT=Yとなりgは(※)をみたすべき等写像なので等長写像である。□

定理7
(S,f)が(※)をみたすとするときfは等長写像である。
(∵)P,Q∈Sをとる。X={f^n(P)|n≧0}、Y={f^n(Q)|n≧0}、T=(X∪Yの閉包)とする。
fのgへの拡張を(T,g)が(※)をみたすようにとる。
d(P,Q)=d(g^n(P),g^n(Q)) (∃n)をしめせばよい。
gが等長写像なら明らか。gは等長写像でないと仮定する。補題5より
ある自然数nをとってg^nは不動点R∈Tをもつと仮定してよい。RはXの閉包の元と
仮定して一般性を失わない。よってある0≦m<nと自然数列(ni)で
limg^(n・ni+m)(P)=Rとなるものがとれる。このとき
0=limd(g^(n・ni+m)(P),R)=limd(g^(n・ni+m)(P),g^(n・ni)R)≧d(g^m(P),R)
よりg^m(P)=Rである。非負整数lに対しl+r≡m (mod n)となる非負整数rをとれば
d(g^(l+n)(P),g^l(P))≦d(g^(l+n+r)(P),g^(l+r)(P))=d(g^(m+kn+n(P),g^(m+kn(P))=0
よりg^n|X=idである。
補題6よりg|Yは等長写像なのでg|Yはべき等であるかまたは
点Oとθ/πが無理数である実数θをg|Yが点Oを中心とするθ回転となるものがとれる。
前者なら主張は成立。
後者ならYはOを中心とする半径d(O,Q)の円の稠密な部分集合となる。
O≠Pと仮定すると自然数k,lをk<l,d(g^k(Q),P)>d(g^l(Q),P)となるようにえらべるが
これは仮定に反する。ゆえにP=Oであり特にd(g^n(P),g^n(Q))=d(P,g^n(Q))=d(P,Q)
である。□

137 名前:132人目の素数さん投稿日:03/10/21 17:46
>>132
条件付き確率で、
(2)の答え / (1)の答え = 2/3 ですな。

138 名前:132人目の素数さん投稿日:03/10/21 18:52

世の中には3種類の人間がいる。
数をかぞえられる人間と、数をかぞえられない人間だ。

139 名前:132人目の素数さん投稿日:03/10/21 23:36
あと一種類は?

140 名前:132投稿日:03/10/22 00:37
>>133,137
そうです。結果を聞いただけで確率が変わるようにみえるのが
ちょっと面白い、かな。ほんとは当然なんだけど。
>>138
例外の無い法則は無い。

141 名前:132人目の素数さん投稿日:03/10/22 00:46
>>139
>>138が「数を数えられない人間」なんだろう。

142 名前:132人目の素数さん投稿日:03/10/22 01:22
>>140
「例外の無い法則は無い。」 … ★
と書いているが、この法則の真偽は?

143 名前:132人目の素数さん投稿日:03/10/22 01:35
★ ⇔ 「法則に例外がある」

★が真ならば、★にも例外があって、★は偽になる。
★が偽ならば、★に例外がないから、★は真になる。

ということかに?

144 名前:132人目の素数さん投稿日:03/10/22 02:05
どんな事柄についても「絶対」なんてことは、絶対にありえない。

145 名前:132人目の素数さん投稿日:03/10/22 04:39
真っ白い正N面体がある。
これを転がして下にきた面を黒く塗る。
M回目転がしたとき、下に白い面が来る確立は?

146 名前:132人目の素数さん投稿日:03/10/22 04:55
確立と書いてるところが気に食わん。
しぬぇ〜〜〜!

147 名前:132人目の素数さん投稿日:03/10/22 04:57
>>138
それって、俺が「頭の悪い発言」スレに書いた事じゃねえか・・・
しかも、それに対してのレスもまったく一緒・・・

148 名前:132人目の素数さん投稿日:03/10/22 13:25
>>145
N個のアイテムをランダムに選ぶとき、特定のアイテムが、
M-1回の試行の後に一度も選ばれていない確率ってことかな?
((N-1)/N)^(M-1)

149 名前:132人目の素数さん投稿日:03/10/22 14:15
>>148を読んで思い出したけど、
「麻雀をm局やって焼き鳥が発生する確率を求めよ。
ただし流局や複数ロンは無いものとする。」

これはまあいいとして、これを一般化した次の問題は
結構難しかった記憶がある。

「n人でやるゲームをm回行なう。焼き鳥(一度も勝てない者)が
発生する確率はいくらか?ただし1回のゲームでは必ず1人の
勝者があり、実力は全員等しいものとする。」

150 名前:132人目の素数さん投稿日:03/10/23 17:32
>>149 難しいねー

全ての場合の数 P0(n,m)=n^m
特定の1人が(最低1回)勝っている場合の数 P1(n,m)=P0(n,m)-P0(n-1,m)
特定の2人が(両方最低1回)勝っている場合の数 P2(n,m)=P1(n,m)-P1(n-1,m)

と真面目に計算していって、Pn(n,m)をもとめ
1-Pn(n,m)/P0(n,m)を計算するしかないのかな?

たとえば4人の場合は1-(4^m-4*3^m+6*2^m-4)/4^mで、
m=0-3→1
m=4→29/32
m=5→49/64

あってるかな?

151 名前:132人目の素数さん投稿日:03/10/24 02:59
漸化式の形からも
Pn(n, m) = 1 * 4^m - 4 * 3^m + 6 * 2^m - 4 * 1^m + 1 * 0^m からも
Pn(n, m) = \sum_{i=0}^n (-1)^i nCi i^m が予想できる。

あとはこれを漸化式に放り込んで証明すればいい。

152 名前:132人目の素数さん投稿日:03/10/24 03:00
二行目は P4(4, m) だった

153 名前:132人目の素数さん投稿日:03/10/25 00:40
未解決問題を少し変えた所から出題。

p,qを素数。 a,bを2以上の自然数として
p^a - q^b = 1
を満たすp,q,a,bを全て求めよ。

ギブアップした方はこちら。
http://www.kalva.demon.co.uk/putnam/psoln/psol763.html

154 名前:132人目の素数さん投稿日:03/10/25 14:51
αがβをかっぱらったらεしたなぜだろう?

155 名前:132人目の素数さん投稿日:03/10/25 19:29
>>153
プログラム組めばいけるけどコンピューター使わないで理詰めでいけるのか?

156 名前:132人目の素数さん投稿日:03/10/25 20:02
>>155
おれ出題者じゃないけど理詰めでいけるみたいよ。

157 名前:132人目の素数さん投稿日:03/10/25 21:21
わかんなくなったら、リンク先見ればいいじゃん。
答え書いてあるよ。

158 名前:132人目の素数さん投稿日:03/10/26 00:43
数直線上の原点0に点Pがある。
1回につき、0.4の確立で正方向に1、0.6の確立で負の方向に1すすむ。
この試行を何回か繰り返し、点Pが+10か-10の位置に来たら終了とする。
このとき、+10で終了する確立は?

159 名前:132人目の素数さん投稿日:03/10/26 00:54
((3/2)^0-(2/3)^10)/(3/2)^10-(2/3)^10)?

160 名前:132人目の素数さん投稿日:03/10/26 23:48
Σ[k=1,n] ( tan(kπ/2n+1) )^2
をもとめんしゃい。

161 名前:132人目の素数さん投稿日:03/10/27 01:16
箱の中に赤い球が10個白い球が20個ある。
ランダムに2個同時に取り出し、
・同色なら1個は捨て1個は箱に戻す
・違うなら白は捨て赤は戻す
これを繰り返す。
この時最後に白い球が残る確率は?

162 名前:132人目の素数さん投稿日:03/10/27 01:22
すぐ答え分かっちゃった。

163 名前:161投稿日:03/10/27 01:31
>>162
中1の時の数学の先生がこういう論理パズル好きで
授業中出された問題の一つなんだけど、
答えが意外だったのと説明が簡単なのとで
当時「すげー面白え」と思ったんで出してみた。
今見ると簡単過ぎるな、やっぱw。
ま、偶にはこういうのもありかと。

164 名前:132人目の素数さん投稿日:03/10/27 01:45
箱の中に赤い玉が10個、白い玉が11個入っている。
箱からランダムに偶数個取り出したとき
赤玉の数と白玉の数が等しいときはその玉をもらうことができる。
何個取り出すのが戦略的にもっとも得か?

165 名前:132人目の素数さん投稿日:03/10/27 20:29
>161
(メール欄)ですか?

166 名前:132人目の素数さん投稿日:03/10/27 21:53
>>165
すみません。どこのメール欄をさしていってるか教えてもらえないでしょうか。

167 名前:132人目の素数さん投稿日:03/10/27 23:36
自然数からランダムに1つ数を選ぶとする。
そのとき数字3を含む数を選ぶ確率はどれくらいか?

たとえば100までには、19個の数がある。
(3,13,23,30〜39,43,53,63,73,83,93)

出典:クリフォード・A・ピックオーバー「無限へチャレンジしよう」
森北出版、1997年7月22日、ISBN4-627-01890-8、4200円。

ちなみにこの本にある3つの質問が面白かったです。

・読者はπを一兆桁計算しようという計画に賛成か?
・政府がその計画に100万ドルを支出するというのを支持するか?
・研究の結果、πの数字の分布がランダムでない
  (すなわち偏りや非一様性)が発見されたとしたとき、
  読者はこれを大発見と思うか?(p70)

168 名前:132人目の素数さん投稿日:03/10/28 00:08
>>167
無限個から1つの数を選ぶというのは設定が難しいので、
次のように考える。

自然数N以下に存在する、3が含まれる自然数の個数を
f(N)とする。lim[N→∞]f(N)/N を求めよ。

N=10^n としたとき、f(N)=10^n-9^n だから、
lim[n→∞]f(10^n)/10^n=1。
従って、f(N)が収束するとすれば、1以外の値にはなりえない。
問題はこれが収束するかだが、直観的には収束しそうだけど
厳密にはわからん。

169 名前:132人目の素数さん投稿日:03/10/28 00:16
>>167
10^nまでに数字3を含む数は10^n-9^n個ある。
よってこの時数字3を含む数を選ぶ確率は(10^n-9^n)/10^n=1-(9/10)^n
nを無限に飛ばすと1

170 名前:132人目の素数さん投稿日:03/10/28 00:16
他の飛ばし方すると変わってくるかも。
めんどい。

171 名前:132人目の素数さん投稿日:03/10/28 00:38
そもそも自然数をランダムに3つというのは well defined な概念なんだろうか?
有界上の一様分布の極限で代用できるとすると Saint Petersburg Paradox
みたいなのにハマる気がするんだが。

172 名前:132人目の素数さん投稿日:03/10/28 00:47
>>171
他にも2つの封筒の話とか、変なのがでてくるよな。
なんで可算無限個からランダムに選ぶ想定だと
奇妙な話が起こるのだろうか。

連続濃度から1つを選ぶことを想定する問題では、
そのような変なことが起こる例は聞いたことがないのに。

173 名前:132人目の素数さん投稿日:03/10/28 00:59
>>172
濃度よりも有界性が問題だろう。R上の一様確率分布というものもないわけで。

174 名前:132人目の素数さん投稿日:03/10/28 01:11
>>173
開区間 (0,1) にある有理数全体に一様確率分布を定義できる?

175 名前:132人目の素数さん投稿日:03/10/28 01:32
>>174
なるほど。
考えてみるに一様分布を定義できるのは
コンパクトで連結成分が有限個の場合だけなんじゃなかろうか。
というかその場合しか思いつかん。

176 名前:132人目の素数さん投稿日:03/10/28 01:36
(0,1) の一様分布というのはアリだからコンパクトは必要じゃないや。
有界で連結成分が有限個ってことかな。

177 名前:132人目の素数さん投稿日:03/10/28 01:40
零集合を除いても一様分布になるのか。連結成分の有限性は関係ないや。難しい。

178 名前:132人目の素数さん投稿日:03/10/28 02:43
円周率についての問題を見つけてきた。

[x] をxを超えない最大の整数と定義する。
[π^n]が素数になる自然数nは有限個しか存在しない事を示せ。

http://problems.math.umr.edu/FMPro?-db=problems.fp3&-format=%2fmaindetail.html&-lay=all&StrippedProblem=prime&-recid=34708&-find=

http://problems.math.umr.edu/FMPro?-db=problems.fp3&-format=%2fmaindetail.html&-lay=all&StrippedProblem=integer&-recid=33939&-find=#Comments0


179 名前:132人目の素数さん投稿日:03/10/28 03:12
[0,1]∩Q上で、P([a,b]∩Q)=b-aと定義したら、[0,1]∩Q上の一様確率分布
になるかしら??
(完全加法性はもたないだろうけど、有限加法性はもつ?)


180 名前:132人目の素数さん投稿日:03/10/28 16:05
素数って整数なんじゃないの?

181 名前:132人目の素数さん投稿日:03/10/28 17:03
整数でない素数は有限個しか存在しないことを示せ

182 名前:132人目の素数さん投稿日:03/10/28 17:12
>>181
こりゃ面白い!

183 名前:132人目の素数さん投稿日:03/10/29 00:40
整数でない素数は存在しません・・・定義より。
よって求める素数の数は0個であり、有限です。

184 名前:132人目の素数さん投稿日:03/10/29 00:42
ん? >>178が間違ってるん?

185 名前:132人目の素数さん投稿日:03/10/29 00:44
1とその数以外の整数で割れない数字が素数の定義だったら
∞にあるな


ってそんな定義じゃねーよウワアン

186 名前:132人目の素数さん投稿日:03/10/29 00:55
atamawaruso

187 名前:Ron投稿日:03/10/29 15:39
とっても おもしろい良問を1つ
 37人にビー玉962個を次のようにくばる。
 くじで あたった7人に]個
 くじで はずれた30人にはY個 くばる
 X>Yとするとき、
  ]、Yの組合わせを全て 答えなされ。

 

188 名前:Ron投稿日:03/10/29 15:43
ついでに もう1問!
半径 1.2.3.4.5cmの中央に穴のある円盤が5枚ある。
これを、棒にさしこむ。
例えば、
 上から、4.2.5.1.3の順にさし、真上から見ると2枚見える。 
(1)ちょうど4枚 見える差し込み方は何通りあるか?
(2)ちょうど2枚 見える差し込み方は何通りあるか?

189 名前:132人目の素数さん投稿日:03/10/29 16:40
もうしわけございません。
当スレでは宿題は取り扱っておりませんのでお引き取りください。

190 名前:132人目の素数さん投稿日:03/10/30 11:27
188は拡張するとちょっとおもしろいかも。

「半径が1,2,3,…,nの計n枚の円盤がある。これを棒にさしこんだとき、真上からみて見える枚数の期待値を求めよ。」


191 名前:132人目の素数さん投稿日:03/10/30 15:59
n/2

192 名前:132人目の素数さん投稿日:03/10/30 18:22
>>190
棒が水平においてあるので、真上からみると全ての円盤が見えます。

というオチじゃないとすると、結構難しいような。
>>188 の5枚の場合でも
5枚見える…1通り
4枚見える…10通り
3枚見える…35通り
2枚見える…50通り
1枚見える…4!=24通り
なので、期待値は2.283…。n/2は違うかと。
正直わからん。

193 名前:132人目の素数さん投稿日:03/10/31 00:30
n 枚のとき m 枚見えるのを P(n, m) とおりとすると

m >= 1 のとき P(n+1, m+1) = P(n, m) + n P(n, m+1)
P(n+1, 1) = n!

だから E(n+1) = (Σ_{m=1}^{n=1} (m+1)P(n+1, m+1) + P(n+1, 1)) / (n+1)!
こっからあとはまかせた

194 名前:132人目の素数さん投稿日:03/10/31 05:40
>>42
そろそろ答えを教えて下さい

195 名前:132人目の素数さん投稿日:03/10/31 22:36
>>96
今更だが96の解説がいまいちわからん。
誰か教えて

196 名前:132人目の素数さん投稿日:03/10/31 22:40
どこが分からんのか言ってくれないと説明のしようがない

197 名前:132人目の素数さん投稿日:03/10/31 22:42
>和をとって、-x^2/120 +x/2+30

ここ。どこの和?

198 名前:132人目の素数さん投稿日:03/10/31 22:54

1)その前にバスが来ていない確立は 1-x/60  ←A
そのときの平均待ち時間は、(60-x)/2       ←B

2)その前にバスが来ている確立はx/60     ←C
そのときの平均待ち時間は(90-x)         ←D

AとBを掛けたものとCとDを掛けたものの和です

199 名前:132人目の素数さん投稿日:03/10/31 22:58
>>154
ドラえもんに聞かないと分からない。

200 名前:195投稿日:03/10/31 23:03
・・・計算あわねェ。ナンデダロ

A×B=(60-61x+x^2)/120

C×D=(90x-x^2)/60

だよねぇ?

201 名前:132人目の素数さん投稿日:03/10/31 23:18
1-x/60は1-(x/60)ですよ。

202 名前:195投稿日:03/10/31 23:20
ぐはぁ、そうでした。
お手数かけてどうもすんまそん。(;´д`)

203 名前:132人目の素数さん投稿日:03/11/01 00:31
お前ら
Σ[n=1,∞] (sin(n))/n
Σ[n=1,∞] (sin(n))/(n^2)
の二つを計算してみ。


204 名前:132人目の素数さん投稿日:03/11/01 00:52
>>203
答え逆三角関数とかつかう?

205 名前:132人目の素数さん投稿日:03/11/01 01:18
Σ[n=1,∞] (sin(n))/(n^2)=0

206 名前:132人目の素数さん投稿日:03/11/01 01:45
>>204
使わん。

207 名前:132人目の素数さん投稿日:03/11/01 12:45
>>190
E(n)=納k=1〜n](1/k)
調和数列の和

208 名前:132人目の素数さん投稿日:03/11/01 18:38
>>203
とりあえずe^i=ρとおいて
im敗in(n)/n
=im(ρ^n)/n (←これが収束するとすると仮定してAbelの定理より)
=imlog(1-ρ) (logsはRes>0で定義された正則関数でlogs=log|s|+iargs
=arg(1-ρ)
=arctan(sin1/(1-cos1)) となったんだけど。よく考えるとさらに
=arctan(2sin(1/2)cos(1/2)/2sin^2(1/2))
=arctan(cot(1/2))
=arctan(tan(π/2-1/2))
=π/2-1/2
あってる?

209 名前:132人目の素数さん投稿日:03/11/01 19:02
>>203
今気付いた。納n=1,∞]sin(nx)/n=(1/2)(π-x)にx=1ほりこんだらいいだけか・・・

210 名前:132人目の素数さん投稿日:03/11/01 19:05
>>205
なぜ?

211 名前:132人目の素数さん投稿日:03/11/02 00:47
>>160
教えてくれ。

212 名前:132人目の素数さん投稿日:03/11/02 13:25
>>160
2n+1=mと置く
ドモアブルの定理より (cosx+isinx)^m=cos(mx)+isin(mx)
二項定理より (cosx+isinx)^m=Σ[K=0,m]comb(m,k)*(i^k)*cos^(m-k)(x)*sin^k(x)
虚部を比較して sin(mx)=Σ[k=0,n](-1)^k*comb(m,2k+1)cos^(m-2k-1)(x)*sin^(2k+1)(x)
この式の両辺をcos^(2*n)(x)*sinxで割ると 
sin(mx)/{cos^(2n)(x)sinx}=Σ[k=0,n](-1)^K*comb(m,k)*tan^(2k)(x)
となる。左辺はx=k*pi/m k=1〜nで0になるので、右辺でtan^2(x)をxと置きなおし
Σ[k=0,n](-1)^K*comb(m,k)*x^k=0 というn次方程式を立てると、解は
x=tan^2(k*pi/m) k=1〜n 解と係数の関係より
Σ[k=1,n]tan^2(k*pi/m)=comb(2n+1,2n-1)/comb(2n+1,2n+1)=n*(2n+1)

213 名前:132人目の素数さん投稿日:03/11/02 17:00
>>213
さんくす。

214 名前:132人目の素数さん投稿日:03/11/02 17:46
Σ[n=1,∞] arctan( 2/(n^2) )

ほい、三角関数の計算問題。

215 名前:132人目の素数さん投稿日:03/11/02 18:33
>>203
(2)の答えおしえてくれage

216 名前:132人目の素数さん投稿日:03/11/03 19:27
203=215(ぷっ

217 名前:132人目の素数さん投稿日:03/11/03 19:34
>>216
ちがう。オマエできたの?きになるだろ。フツー
 
Σ[n=1,∞] (sin(n))/(n^2)
 
これどうやんの?


218 名前:132人目の素数さん投稿日:03/11/03 19:48
スマン、出題ミスだ
Σ[n=1,∞] (sin(n)/n)^2

二乗がね、分数の外にかかってるんだよ。ごめん。

答えは
(π-1)/2
がんばれや。

219 名前:132人目の素数さん投稿日:03/11/03 19:50
>>214
これもわかりません。答えかせめてヒントを〜

220 名前:132人目の素数さん投稿日:03/11/03 19:53
>>214
arctan(1/n)
がヒント。

これいじょうだせっかい。

221 名前:132人目の素数さん投稿日:03/11/03 19:55
じゃせめて出典おしえてくらはい。

222 名前:132人目の素数さん投稿日:03/11/03 20:06
こっちは解決
(sin(n)/n)^2
=(1-cos2n)/(2n^2)
ここでフーリエ展開の公式
把osnx/(n^2)=(1/4)(x-π)^2/4-π^2/12
にx=2を代入して
把os2n/(n^2)=(1/4)(2-π)^2-π^2/12
だから
=(1-cos2n)/(2n^2)
=(1/2)(1/n^2-把os2n/(n^2)
=(1/2)(π^2/6-(1/4)(2-π)^2+π^2/12)
=(1/2)(π^2/4-(1/4)(2-π)^2)
=(1/8)(π+(2-π))(π-(2-π))
=(1/8)(2)(2π-2)
=(π-1)/2
まあ敗in(n)/nの方でフーリエ級数使ったから次もそうだろうなとあたりはついてたんだけど。

223 名前:132人目の素数さん投稿日:03/11/03 20:48
>>214
出典は忘れた

a(n) = arctan(1/n)として、
tan( a(n+1)-a(n-1)

でも考えてみ。

224 名前:132人目の素数さん投稿日:03/11/03 20:58
http://www.cms.math.ca/Competitions/MOCP/2003/prob_apr.html
出典探してきた。


スレ汚しスマソ。

225 名前:132人目の素数さん投稿日:03/11/03 21:00
>>223
とりあえず計算機つかって強引にといた。計算機つかって第3項以降たしてみると
だいたいπ/4になることからb(n)=2/(n+2)^2とおいて数列anを
a(0)=1,a(n)=(a(n-1)-b(n))/(1+a(n-1)b(n))
を計算機に計算させるとa(n)=(2n+5)/(n^2+5n+5)になった。初項と2項の和はπ/2
だから答え3π/4だね。なんとも不思議なもんだ。
・・・
とおもってたら答えみたいなヒントが・・・答えきいてみるとなーんだってかんじだね。

226 名前:132人目の素数さん投稿日:03/11/04 00:56
昔々小学生の時、担任から大学の試験で出る問題と言うのを聞いた

「1+1=2」を証明せよ

あれから十年ほど立つが未だにわからん。誰か教えてたも
あとここに書いたのは面白そうだからかな

227 名前:132人目の素数さん投稿日:03/11/04 01:18
>>226
自然数の公理と自然数に関する加法の定義より
1+1=xを満たすxは一意に存在し、
後者関数をsとしてs(s(0))と表せる。
このs(s(0))に与えられた名前が2である。

つづきは以下へ
http://science.2ch.net/test/read.cgi/math/986656055/l50

228 名前:132人目の素数さん投稿日:03/11/04 19:34
サンクスコ
まさか本当に証明できるとは思って無かったよ

229 名前:132人目の素数さん投稿日:03/11/05 16:04
1,2,3,4,5,6,7,8,9の数字を一回ずつ全て用いて10進数9桁の数字を作る。
このとき、作った数が自然数の自然数乗になるとき、その数を求めなさい。

もちろん、一乗ってのは無し。2乗以上ね。

230 名前:132人目の素数さん投稿日:03/11/05 18:12
>>229
求める数をa^nとすると、
a >= 2
n >= 2
10^8 < a^n < 10^9
a>=10でこれをみたすnがないのは明らか。またaとして4=2^2, 8=2^3, 9=3^2
はa=2,3の場合に含めるとすれば、ありうる数は、
2^27, 2^28, 2^29, 3^17, 3^18, 5^12, 6^11, 7^10の8つのみ。
そしてこれらで、
「1,2,3,4,5,6,7,8,9の数字を一回ずつ全て用いて」
という条件を満たすものはない。

あ、あれ?違ってる?

231 名前:132人目の素数さん投稿日:03/11/05 18:14
a>=10でこれをみたすnがないのは明らか。

すまんがここが俺にとって明らかではない。説明しる。

232 名前:230投稿日:03/11/05 18:41
>>231
まちがってた、スマン。
2乗とか考えればすぐわかる、すごいぼけかただな…

233 名前:230投稿日:03/11/05 19:34
やりなおし。

求める数をa^nとすると、
a >= 2
10^8 < a^n < 10^9
なので、2 <= n <= 29。nが素数の場合のみ検討すれば十分なので、
n=2,3,5,7,11,13,17,19,23,29について
10^(8/n) < a < 10^(9/n)
の範囲で題意を満たす数を探せば良い。結果、次の30個の数が答。
(以下略)

234 名前:132人目の素数さん投稿日:03/11/05 22:51
>>233
>nが素数の場合のみ検討すれば十分なので、

説明しる!

235 名前:132人目の素数さん投稿日:03/11/06 11:08
>>234
n=pm, p:prime
のとき、もとめたい数がa^nとすると、a^n=a^(pm)=(a^m)^pなので、
ベキ数が素数の場合だけ調べれば十分、じゃないかな。

236 名前:132人目の素数さん投稿日:03/11/07 23:49
n <= 29
なぜ?

237 名前:132人目の素数さん投稿日:03/11/08 00:04
8/log(10)a <= n <= 9/log(10)a
ってことか。

238 名前:132人目の素数さん投稿日:03/11/08 00:17
a^n≡0(mod9)
a^6≡1(mod9)だからかなり絞られるね。

239 名前:132人目の素数さん投稿日:03/11/08 00:36
あまりに簡単で自明だと思ったのですが、
いざ証明しろと言われると、悔しいことに証明できませんでした。
誰か賢い方、証明を教えてください・・・。


相異なるn個の実数が格納されている配列a[1]〜a[n]がある。
この配列の中から最大値を調べるためには、すべての配列を調べる必要がある
 (つまり、どんなにうまいアルゴリズムでもnに比例した時間がかかってしまう)
ことを証明しなさい。

240 名前:132人目の素数さん投稿日:03/11/08 00:44
>>239
仮にa[1]からa[n-1]まで調べたとしても、
依然としてa[n]が最大値である可能性も最大値でない可能性も残されてるので、
結論は出せない。
よって全て調べる必要がある。

これじゃ駄目かい?

241 名前:132人目の素数さん投稿日:03/11/08 00:47
>>240さん

おそらくはそれでいいのだと思いますが、
なんとなく「言葉で説明している」感が強いじゃないですか。

>>a[n]が最大値である可能性も最大値でない可能性も残されてるので
あたりを、数式ですっきり説明できないか、と思ったのです。

242 名前:132人目の素数さん投稿日:03/11/08 01:00
>>241
ううむ。
一つの配列を調べることで得られる情報量の評価をすればいいのかしら?
よくわからん

243 名前:132人目の素数さん投稿日:03/11/08 01:18
>>241
∃a=(a[1],…,a[n])∈R^n.(max{a[k]|k=1,…,n}>max{a[k]|k=1,…,n-1})
とかか?
何か大袈裟になっただけのような気がせんでもないが。


244 名前:132人目の素数さん投稿日:03/11/08 01:43
>>239
単純に任意の自然数1≦k≦nのkに対してa[k]が最大になる確率は1/n。 (なぜならば、a(n)は相異なる実数)
故に、a(n)からm(≦n)個の数列を取り出したとき、その中に最大の数が含まれていない確率は・・・


じゃ、ダメなのか?

245 名前:132人目の素数さん投稿日:03/11/08 01:52
>>241
言葉で説明している感が強いのはダメなのか。じゃあε(ry

246 名前:132人目の素数さん投稿日:03/11/08 02:02
max{a[0],...,a[n-1]}<=max{a[0],...,a[n]}

247 名前:132人目の素数さん投稿日:03/11/08 02:06
241です。みなさん、いろいろすごいお答えをいただいて恐縮です。

ことに
>>243さんの答えは、
たぶん>>240を数式化したものだと思いますが、すごいと思います。

>>245さん
「言葉の説明でも、立派な証明だ」と言い切れるのでしたら、僕も安心なのです。
ただ僕は「言葉の証明は証明ではない。もっと厳密にやれ」と教師に言われたら、
自信を持って言い返せないのです。言い返していいんでしょうか?

>>244さん
確率が決して0にはならないから、ってことですよね。
たぶんそれでもいいと思います。
ただ、その、これも>>245的な不安がいまいち残ってしまうのです。

せっかくの皆さんの答えに、
いちいち文句つけてるような感じで、すみません。もうやめます。

いちおう出典を示します。

ジョン・ベントリー
「珠玉のプログラミング-本質を見抜いたアルゴリズムとデータ構造-」ピアソン・エデュケーション、
2000年10月25日、3400円、ISBN4-89471-236-9。
(の問題を、すこし変形しました。ちなみにこの本には解答は載っていませんでした)

248 名前:132人目の素数さん投稿日:03/11/08 02:11
247です。先ほどは失礼しました。
お詫びに「珠玉のプログラミング」からちょっと変形した問題を紹介。

いま長方形の、ある程度の厚みがある、
同じ大きさ・厚さのプラスチックのカードが1024枚あり、
1〜1024までの連番が振ってあります。
そして、ごちゃごちゃに混ぜられています。

ところがいたずらな子供が、一枚を盗んで捨ててしまいました。

1〜1024まで、すべてのカードが揃っている必要があるので、
子供が捨てた番号のカードを、急いで作り直す必要があります。

つまり問題は、
「欠番となってしまった番号を、すばやく探し出す方法を述べよ」です。

1〜1024までのカードを、番号順に並べなおせばいいのでしょうが、
それはとてつもない時間がかかります。
もう少しラクな方法はないでしょうか?

249 名前:132人目の素数さん投稿日:03/11/08 02:46
>>248
欠番を探すためには結局全てのカードにアクセスしなきゃいけないから
どううまく番号順に並べるか、になりそうだが…。


250 名前:132人目の素数さん投稿日:03/11/08 04:01
普通にバケットソートでもいいが一枚ずつとってきて書かれている番号を全部足していけばいい。
1+2+...+1024=1024*1025/2 だからいくつ少ないかを計算すれば欠番がわかる。

251 名前:132人目の素数さん投稿日:03/11/08 11:59
>>248
>1〜1024までのカードを、番号順に並べなおせばいいのでしょうが、
>それはとてつもない時間がかかります。
これが、どういうオーダーをさしているのかがわからないのですが、
ともかく(もとがn枚とすれば)n-1枚の表示を調べなければならないので
nのオーダーは必要ですよね。
たとえば>250さんの方法でも同じですが、一番素直には、1-1024の数字を
書いた升目を用意してそこにカードを並べていくんじゃないかな。
「探し出す」だけならこれ以上にもこれ以下にもならないと思うのですが…

252 名前:132人目の素数さん投稿日:03/11/08 12:11
盗んだ子供をとっつかまえるのが早い

253 名前:132人目の素数さん投稿日:03/11/08 12:35
>>247
> 「言葉の説明でも、立派な証明だ」と言い切れるのでしたら、僕も安心なのです。
> ただ僕は「言葉の証明は証明ではない。もっと厳密にやれ」と教師に言われたら、
> 自信を持って言い返せないのです。言い返していいんでしょうか?

それは良くない誤解の典型。記号で書けば立派というわけではない。
記号は分かりやすさのために使われるに過ぎない。
言葉で書いた方が分かりやすいなら言葉で書いた方が良い。

それはそれとして、多くのプログラミング言語の場合、a[n] まで
調べる必要がいつもあるわけではない。なぜなら多くのプログラミング言語で
実数と呼ばれるものは浮動小数点数であり IEEE754 によって規定される。
IEEE754 では最大の浮動小数点数が定義されている。だから、
k<n なる a[k] が最大の浮動小数点数ならばその時点で
探索を終了して良い。しかし「時間が f(n) に比例する」というのは
最悪の場合の時間を意味するので、「nに比例した時間がかかってしまう」
ということは正しい。

254 名前:132人目の素数さん投稿日:03/11/08 13:23
>>253
ひとつひとつの要素について「マシンで表現可能な最大値」と
比較するだけで、処理時間は倍になる。


255 名前:132人目の素数さん投稿日:03/11/08 14:00
定数倍ならオーダーは変わらん。

256 名前:132人目の素数さん投稿日:03/11/08 14:06
>>251
それがバケットソート。
バケットソートでも>>250の方法でも計算量はO(n)だが係数がだいぶ違う。

257 名前:132人目の素数さん投稿日:03/11/08 14:21
>>256
揚げ足を取るようで悪いが、人間がやるとしたら
係数が逆転することも考えられるね。
つまり計算1ステップの時間が違う。


258 名前:132人目の素数さん投稿日:03/11/08 15:18
コンピュータなら速く正確に足し算できますが、
人間なら間違うこともあるし、
途中できっといやになっちゃいますね。

コンピュータなしにひとりでやるとしたら、
とりあえず最初に全部のカードを調べて
100番台、200番台・・・てな山に分けておいて、
それぞれ枚数を数えて、
足りないところだけじっくり調べる・・・かな。


259 名前:132人目の素数さん投稿日:03/11/08 16:37
>>258
> それぞれ枚数を数えて、
ある程度の厚みのあるカードなので、重ねて高さを比較すれば ok.

260 名前:132人目の素数さん投稿日:03/11/08 22:42
>>248です

もしかしたら、本を下手に変形したため、出題ミスになってしまったかもです。
こちらの想定していた正解は >258, 259さんの答えと同じです。

つまりまず1024枚のカードを、「小」つまり1〜512までと、「大」つまり513〜1024に分けます。
その2つの山を積み重ねると、どちらか一方だけが1枚、少ないはずです。
欠番はその山にあります。
いま仮に「大」の山のほうが少なかったとしましょう。
あとは繰り返しです。つまり
「大」の山513〜1024を、さらに半分、
つまり「513〜768」と「769〜1024」に分けます。
すると、どちらか一方だけが1枚、少ないはずです。
そして・・・と続けていくと、欠番のカードがみつかる、という話です。

261 名前:132人目の素数さん投稿日:03/11/08 22:45
ペンダントが豪華なケースに入って、6万4500円で売られている。
「ケースだけの値段は?」とたずねると、ペンダントの値段はケースよりも
6万4000円高いといわれた。
では、ケースだけの値段はいくらになるだろうか。

暗算でやってみましょー

262 名前:132人目の素数さん投稿日:03/11/08 22:48
>>260です。
お詫びに、既出かも + 数学じゃないかも、ですが紹介します。

サイト「cyberglass.net」
http://www.cyberglass.net/
に行き、「flash」の「flash mind reader」を選択してください。

10〜99までの数をひとつ、思い浮かべてください。
たとえば75とします。
で、10の位の数と1の位の数を足してください。7+5=12。
この数を、もとの数から引いてください。75-12=63。

さて、水晶玉の右にある表を見てください。
63の横に模様がありますね?

で、水晶玉をクリックすると・・・なんと、その模様があらわれます!
あなたがどんな数字を心の中で選んでも、いつも当たります。
なぜ当たるのか、が問題です。
(が、わかった人も種明かしをしないほうが、みんなに親切かもです)

263 名前:132人目の素数さん投稿日:03/11/08 22:49
>261
似た問題この前どっかのスレで見たなあ

264 名前:132人目の素数さん投稿日:03/11/08 22:50
じゃあソート問題。
5枚のカードがあってすべて異なる強さが定義されている。
カードの強さは見た目ではわからず、2枚のカードの強弱を決める機械が
ありこれを使わないと知ることができない。
強弱の順序関係、A>BかつB>CならA>Cはもちろん成立している。
カードを強さ順に並べたい。機械を何回使えば必ずソートが完了するか?
(ワーストケースでの最小値という意味です)

265 名前:132人目の素数さん投稿日:03/11/08 22:51
>>261
500円 引き算だろ

266 名前:132人目の素数さん投稿日:03/11/08 22:56
>>265
ドボン

267 名前:265投稿日:03/11/08 22:58
聞かなかった事にしてください_| ̄|○

268 名前:132人目の素数さん投稿日:03/11/08 23:02
>>267
まじでひっかかったのかよw
どんまい

269 名前:132人目の素数さん投稿日:03/11/08 23:03
>>264
単に「n回で可能」って答えじゃダメ?「n−1回以下では不可能」の証明もいるの?
パッと見7回くさいんだけど。

270 名前:132人目の素数さん投稿日:03/11/08 23:05
>>264
7回で出来る?がんばっても8回になるんだけど、漏れアホだ。

271 名前:132人目の素数さん投稿日:03/11/08 23:09
>>270
数えまちごた。8回でつ。吊ってくる・・・

272 名前:132人目の素数さん投稿日:03/11/08 23:10
7○9|6○○|5○3
○○○|○4○|8○○
6○○|○○5|○1○
――― ――― ―――
2○○|○9○|35○
○1○|5○7|○4○
○○5|○2○|○○7
――― ――― ―――
○3○|2○○|○○5
○○4|○7○|○○○
8○○|○51|9○6

よく新聞のパズル欄にあるやつです。
朝からやってるのに解けません・・・。誰か救いの手を!!

273 名前:264投稿日:03/11/08 23:14
>>269-271
そう、6回で不可能なのはすぐわかるし、8回でできるのもすぐわかる。
問題は7回で(多分)できないんだけど、それをどう説明するかなんですよ。

274 名前:132人目の素数さん投稿日:03/11/08 23:23
>>261
わからない 答えと理由きぼんぬ
500じゃないのか

275 名前:132人目の素数さん投稿日:03/11/08 23:30
>>274
ケース500円ならペンダント64000円で差は63500円じゃん。

276 名前:132人目の素数さん投稿日:03/11/08 23:31
>>273
んー自信ないけど、「ワーストケースを考えているので
どのカードも必ず2回は測らないといけないから」かな。

277 名前:132人目の素数さん投稿日:03/11/08 23:42
つーとケースが250円か?
するとペンダントは64250円。
差額64000円。

あってる?

278 名前:132人目の素数さん投稿日:03/11/08 23:46
>>272
できた
749|618|523
351|742|869
628|935|714
──┼──┼──
267│894│351
913│567│248
485│123│697
──┼──┼──
136│289│475
594│376│182
872│451│936

279 名前:132人目の素数さん投稿日:03/11/08 23:51
くわしい解説きぼんた

280 名前:132人目の素数さん投稿日:03/11/08 23:54
>>279
どれの?

281 名前:132人目の素数さん投稿日:03/11/08 23:55
261のです
まんまとひっかかってて理解できないぽ

282 名前:132人目の素数さん投稿日:03/11/08 23:58
>>281
>>275-277以上に説明するのはムリポ

283 名前:132人目の素数さん投稿日:03/11/08 23:59
数の単位を100円とかに落とすとか・・・むりでしょうか。。

284 名前:愛子投稿日:03/11/09 00:21
ttp://aiko.fam.cx/

285 名前:132人目の素数さん投稿日:03/11/09 00:44
>>281
連立一時方程式を立てて解け。


286 名前:264投稿日:03/11/09 00:59
>>264は7回でできるわ。
問題の意図は、log(5!)/log(2)=6.906...
なのに7回でできない、というのを狙っていたんだけど…
ワーストケースでも7回ですむアルゴリズムは?という問題にしとこう。

287 名前:132人目の素数さん投稿日:03/11/09 11:39
ん?7回で出来ないことを証明できてしまった。

かなり泥くさい証明なので、書くのが難しいが・・・。
>>286 と意見が分かれてもーた

288 名前:287投稿日:03/11/09 11:52
全部で 120 通りの可能性がある。

1回目、AとBを比べる。
A>B となったとする。残り可能性 60 通り。

2回目
i)AとCを比べた場合
A>C となると、残り可能性は 40 通り。2^5=32 なので、残り5回では決定できない。
ii)CとDを比べた場合
C>D となったとする。残り可能性 30 通り。

3回目
i)AとEを比べた場合
A>E となったとすると、残り可能性は 20 通り。2^4=16 を上回るのでアウト。
ii)AとDを比べた場合
A>D となったとすると、残り可能性は 25 通り。2^4=16 を上回るのでアウト。
iii)AとCを比べた場合
A>C となったとすると、残り可能性は 15 通り。

この時点で、残ってる可能性は

a)ABCD
b)ACBD
c)ACDB
のどこかにEを入れた15通りである。
と考えると、以下の計算が楽になる。

289 名前:287投稿日:03/11/09 11:53
続き。

4回目
i)Eを比較に使わなかった場合
(a)(b)(c) から候補が減らせるだけで、残り可能性は 10 通りまでにしか減らない。
2^4 = 8 を上回るのでアウト。
ii)Eを比較に使った場合
Eが何番目に入るか分かったとしても、残り可能性は 9 通りまでにしか減らない。
2^4 = 8 を上回るのでアウト。

よって、7回での判定は無理。
天秤を使って、2枚と2枚を比較できればどうなるか分からないけど。


290 名前:132人目の素数さん投稿日:03/11/09 19:03
>>264をn枚のカードにしたらどうなるんだろう?

291 名前:277投稿日:03/11/09 19:28
折れは間違ってるの?

292 名前:132人目の素数さん投稿日:03/11/09 20:16
42 :132人目の素数さん :03/10/10 20:32

任意の正の数a,b,c,d,e,fに対して、次の不等式を示せ。
ab/(a+b) + cd/(c+d) + ef/(e+f) ≦ (a+b+c)(d+e+f)/(a+b+c+d+e+f)

293 名前:132人目の素数さん投稿日:03/11/09 20:31
>>292
それ解けないんだよ。あげ

294 名前:132人目の素数さん投稿日:03/11/09 20:51
数を減らして出来ないかな? たとえば、
ab/(a+b) + cd/(c+d) ≦ (a+c)(b+d)/(a+b+c+d)
が成り立つとか? で、それを繰り返し使うのでは?
ていうか、問題が間違ってるんじゃないの?
右辺が、こうじゃないと綺麗くないYO

ab/(a+b) + cd/(c+d) + ef/(e+f) ≦ (a+c+e)(b+d+f)/(a+b+c+d+e+f)

295 名前:132人目の素数さん投稿日:03/11/09 21:03
>>294
みつけた
http://search.yahoo.com/search?p=ab%2F%28a%2Bb%29%2Bcd%2F%28c%2Bd%29%2Bef%2F%28e%2Bf%29+positive&ei=UTF-8&n=20&fl=0&x=wrt

しかし、解答はなかった…

296 名前:132人目の素数さん投稿日:03/11/09 21:07
いい仕事だ >>295

http://www.kalva.demon.co.uk/bmo/bmo83.html

元ネタはイギリス数学オリンピックって所だな。

297 名前:132人目の素数さん投稿日:03/11/09 21:16
すう折はやっぱり難しいんだな

298 名前:132人目の素数さん投稿日:03/11/09 21:17
ンナコトないと思うぞ。 問題が間違ってなければ解いてみせる・・・・・・


自信ないけどな。

  ちょいまってろ

299 名前:132人目の素数さん投稿日:03/11/09 21:20
むずかしいっていうか・・・

300 名前:132人目の素数さん投稿日:03/11/09 21:35
>>292
おかしいね。これやっぱり。a→∞の極限とると
左辺=b+cd/(c+d)+ef/(e+f)
右辺=d+e+f
であきらかに左辺≦右辺はb,c,d,e,f>0で成立しないね。

301 名前:132人目の素数さん投稿日:03/11/09 21:40
とりあえず>>294ならできた。
f(x)=(x+c+e)(b+d+f)/(x+b+c+d+e+f) - xb/(x+b) + cd/(c+d) + ef/(e+f)
とおく。x>0で単調減少なのでf(x)≧0 (∀x>0)⇔lim[x→∞]f(x)≧0
ここでlim[x→∞]f(x)=d+f-cd/(c+d)-ef/(e+f)=d^2/(c+d)+f^2/(e+f)≧0

302 名前:301投稿日:03/11/09 22:06
スマン・・・>>301まちがい・・・吊ってくる

303 名前:132人目の素数さん投稿日:03/11/09 22:31
>>294の方針じゃダメですか?

304 名前:132人目の素数さん投稿日:03/11/09 22:40
>>294
与式⇔b^2/(a+b)+c^2/(c+d)+f^2/(e+f)-(b+d+f)^2/(a+b+c+d+e+f)=Sとおく。
Sをa,c,eの3変数関数とみなして極値
⇒b^2/(a+b)^2=c^2/(c+d)^2=f^2/(e+f)^2
⇒b/(a+b)=c/(c+d)=f/(e+f)=1/kとおくと
f(bk-b,b,dk-d,d,fk-f,f)=(1/k)(b+d+f-b-d-f)=0
∴極値は(存在したとしても)≧0。あとは

305 名前:132人目の素数さん投稿日:03/11/09 22:41
>>303
できると思う。

306 名前:132人目の素数さん投稿日:03/11/09 22:42
数オリでは微分は範囲外だから、
もっと簡単な方法があるはず・・・

307 名前:132人目の素数さん投稿日:03/11/09 22:56
まあ・・・あるかもしれないけど・・・そうなると「中学入試スレ」とかでいわれてたみたいに
それはもはや「数学」とはいえない。ただの「数学的パズル選手権」みたいになってしまう。

308 名前:132人目の素数さん投稿日:03/11/09 23:04
>>307
考えてみる価値はありそう。さっき気づいたんだけど、>>294
ab/(a+b) + cd/(c+d) ≦ (a+c)(b+d)/(a+b+c+d) を変形すると
1962.Vietnum.Q1 と同じ式になった。まだ証明は出来てませんが…
http://www.kalva.demon.co.uk/vietnam/viet62.html

何故すぐ同値な問題を発見したかと言うと、不等式オタだから…。
夏休み頃から あちこち検索して不等式を収集していたので (/ρ\)
ただ不等式が好きなだけで、解く力が弱いので質問しまくってますが…

309 名前:132人目の素数さん投稿日:03/11/09 23:05
>>308の問題の解法を教えて下さい。

310 名前:132人目の素数さん投稿日:03/11/09 23:10
>>309
偏微分つかっちゃいかんの?

311 名前:132人目の素数さん投稿日:03/11/09 23:13
できれば、微積なしでお願いしたいです。
理解する力が乏しいので… (泣)

312 名前:132人目の素数さん投稿日:03/11/09 23:14
いま ab/(a+b) + cd/(c+d) ≦ (a+c)(b+d)/(a+b+c+d) の差をとって
分子をゴリゴリと変形してるけど、グチャグチャになってます。

313 名前:132人目の素数さん投稿日:03/11/09 23:25
>>309
すくいなくとも右辺-左辺=(a+c)(b+d)/(a+b+c+d)-ab/(a+b)-cd/(c+d)だから
f(x)=(x+c)(b+d)/(x+b+c+d)-xb/(x+b)-cd/(c+d)とおいてf(x)の増減表をかくと
x=bc/dのとき最小がわかるので最小値計算すると
f(bc/d)=0になった。
↑これもダメ?

314 名前:132人目の素数さん投稿日:03/11/09 23:31
なるほど、分かりそうです。
微分して増減評書いてみます。
ありがとうございます。

315 名前:286投稿日:03/11/09 23:34
>>287
最初3回はまったく同じで、
A>B, C>D, A>C
としても一般性を失わず、15通りのパターンが残る。
4回目CとEの比較で(ここで数え間違えていると思います)
C>E→残り8パターン、5回目D,E比較
C<E→残り7パターン、5回目A,E比較
以下、結果に従い分岐していくが、4回目以降を羅列すると
C>E→D>E→B>D→B>C : ABCDE
C>E→D>E→B>D→B<C : ACBDE
C>E→D>E→B<D→B>E : ACDBE
C>E→D>E→B<D→B<E : ACDEB
C>E→D<E→B>E→B>C : ABCED
C>E→D<E→B>E→B<C : ACBED
C>E→D<E→B<E→B>D : ACEBD
C>E→D<E→B<E→B<D : ACEDB
C<E→A>E→B>C→B>E : ABECD
C<E→A>E→B>C→B<E : AEBCD
C<E→A>E→B<C→B>D : AECBD
C<E→A>E→B<C→B<D : AECDB
C<E→A<E→B>C : EABCD
C<E→A<E→B<C→B>D : EACBD
C<E→A<E→B<C→B<D : EACDB

316 名前:132人目の素数さん投稿日:03/11/10 00:12
>>292
両辺に(a+b+c+d+e+f)をかけて右辺-左辺
=(a+c+e)(b+d+f) -ab(a+b+c+d+e+f)/(a+b) -cd(a+b+c+d+e+f)/(c+d) -ef(a+b+c+d+e+f)/(e+f)
=(a+c+e)(b+d+f) -ab -ab(c+d+e+f)/(a+b) -cd -cd(a+b+e+f)/(c+d) -ef -ef(a+b+c+d)/(e+f)
=ad+af+bc+cf+be+de -ab(c+d+e+f)/(a+b) -cd(a+b+e+f)/(c+d) -ef(a+b+c+d)/(e+f)
=ad+bc -ab(c+d)/(a+b) -cd(a+b)/(c+d) + af+be -ab(e+f)/(a+b) -ef(a+b)/(e+f) + cf+de -cd(e+f)/(c+d) -ef(c+d)/(e+f)
={(ad+bc)(a+b)(c+d) -ab(c+d)^2 -cd(a+b)^2}/(a+b)(c+d) + {(af+be) -ab(e+f)^2 -ef(a+b)^2}/(a+b)(e+f) + {(cf+de) -cd(e+f)^2 -ef(c+d)^2}/(a+b)(e+f)
=(ad-bc)^2/(a+b)(c+d) +(af-be)^2/(a+b)(e+f) + (cf-de)^2/(a+b)(e+f)
>0

317 名前:132人目の素数さん投稿日:03/11/10 00:21
す、すごい (自分はとっくに挫折してました)
神キタ━(゚∀゚)━!!!!
印刷して計算を追ってみます。大感謝!
数学板の住人ってスゴすぎる…。

318 名前:316投稿日:03/11/10 00:30
>={(ad+bc)(a+b)(c+d) -ab(c+d)^2 -cd(a+b)^2}/(a+b)(c+d) + {(af+be) -ab(e+f)^2 -ef(a+b)^2}/(a+b)(e+f) + {(cf+de) -cd(e+f)^2 -ef(c+d)^2}/(a+b)(e+f)
ここミスった
正しくは
={(ad+bc)(a+b)(c+d) -ab(c+d)^2 -cd(a+b)^2}/(a+b)(c+d) + {(af+be)(a+b)(e+f) -ab(e+f)^2 -ef(a+b)^2}/(a+b)(e+f) + {(cf+de)(c+d)(e+f) -cd(e+f)^2 -ef(c+d)^2}/(a+b)(e+f)

319 名前:316投稿日:03/11/10 00:36
まだミスってる
={(ad+bc)(a+b)(c+d) -ab(c+d)^2 -cd(a+b)^2}/(a+b)(c+d) + {(af+be)(a+b)(e+f) -ab(e+f)^2 -ef(a+b)^2}/(a+b)(e+f) + {(cf+de)(c+d)(e+f) -cd(e+f)^2 -ef(c+d)^2}/(c+d)(e+f)

何度もスマン_| ̄|○

320 名前:132人目の素数さん投稿日:03/11/10 02:32
既に3行目から間違ってるよ。

321 名前:132人目の素数さん投稿日:03/11/10 02:36
この手の問題自動的に解答つくるアルゴリズムつくれないかな。

322 名前:132人目の素数さん投稿日:03/11/10 04:46
Lagrangeの未定乗数法とかでゴリゴリ解けばいいのでは……。

323 名前:316投稿日:03/11/10 12:20
いつの間にか>>294の下 = >>296を解いてた…

324 名前:316投稿日:03/11/10 12:58
>>292
a=2, b=2, c=2, d=1, e=1, f=1のとき

左辺 = (2*2)/(2+2) + (2*1)/(2+1) + (1*1)/(1+1) = 13/6
右辺 = (2+2+2)(1+1+1)/(2+2+2+1+1+1) = 2

これでどう?

325 名前:132人目の素数さん投稿日:03/11/10 22:06
>>324
>>292が成立しないのは外出なんだが・・・

326 名前:132人目の素数さん投稿日:03/11/10 23:10
ごめんなソーリー

327 名前:132人目の素数さん投稿日:03/11/12 02:05
面白そうなので質問スレよりコピペ。
http://science.2ch.net/test/read.cgi/math/1065589228/836
ちょっと考えてみたがわからん。

0≦r<1

Σ[k=0,∞]r^(2^k)
=r+(r^2)+(r^4)+(r^8)+(r^16)+…
=???

328 名前:132人目の素数さん投稿日:03/11/12 15:50
マルチすんなクズ

329 名前:132人目の素数さん投稿日:03/11/14 14:43
>>327の答えは出たの?

330 名前:132人目の素数さん投稿日:03/11/14 15:04
出てないみたいだよ

331 名前:132人目の素数さん投稿日:03/11/14 19:25

http://up.isp.2ch.net/up/fdc7b7e279e9.gif
↑なんですが 64 = 65???
矛盾がわかりません。

332 名前:132人目の素数さん投稿日:03/11/14 19:39
>>331
実際にやると真ん中に小さい隙間が空く。それが丁度面積1


333 名前:132人目の素数さん投稿日:03/11/14 20:01
センスを問います 
問題:正しい事を確率4/5で言う3人がいる。この3人の前でコインを5枚投げたところ
3人とも「5枚とも表が出た」と言った。
1)このとき本当に5枚とも表が出ている確率は4/5より、大きいか?小さいか?
  (解答時間15秒以内)
2)その確率を求めよ (メインは1)

334 名前:132人目の素数さん投稿日:03/11/14 20:12
出目がどうあれ、5人とも正しいことを言う確率は
かなり小さい気がする。もちろん直観だけど。

335 名前:132人目の素数さん投稿日:03/11/14 20:14
うわ、5人じゃなくて3人だった。
計算してみるか…

336 名前:132人目の素数さん投稿日:03/11/14 20:17
正しい事を高い確率で言う人が揃って正しいと主張するから
それは一人の場合よりあたる確率は高くなる なんちゃって。

337 名前:132人目の素数さん投稿日:03/11/14 20:26
答えはこうなったんだけど。
見直しもしてないので自信ない。

338 名前:132人目の素数さん投稿日:03/11/14 20:28
>>337 と同じになった。

・・・予想はずれたよヽ(`Д´)ノウワァァァン!!

339 名前:132人目の素数さん投稿日:03/11/14 20:31
確率p  n人 m枚  での一般解はどうなるの?

340 名前:132人目の素数さん投稿日:03/11/14 20:33
99%正しい事言う奴が100人集まって表でたっていったら
ほぼ間違いなくあたってんじゃねーの?
99%より下がっちゃうんですか? ヽ(`Д´)ノナットクデキン!!


341 名前:132人目の素数さん投稿日:03/11/14 20:38
少しでも嘘をつく可能性があるヤシは、
たくさん集まれば集まるほど信頼性が低くなるってことでは?

342 名前:132人目の素数さん投稿日:03/11/14 20:47
素朴な疑問でごめんなさい
8割本当のことを言う人が二人いてその二人がA=Bといったとき
A=Bである確率ってどうだすのですか?
4/5・4/5ではないですよね?4/5+4/5のはずないですよね?
1−(1−4/5)(1−4/5) ですか ???よく分かりません

343 名前:132人目の素数さん投稿日:03/11/14 21:04
>>342
P_A(B) で、「Aが起こったという条件下でBが起こる確率」を表す
ことにすると、

P_A(B) = P(A∩B)/P(A)

これで計算する。

344 名前:132人目の素数さん投稿日:03/11/14 21:08
>>333
問題に不備がある気がする。
そのままだと、ある1人が、5枚のうちに裏があるのに
「全部表だ」と答える確率が計算できない。

問題:
正しい事を確率4/5で言う3人がいる。
この3人の前でコインを5枚投げ、
「5枚とも表が出たか?」と問うたところ、
3人ともyesと言った。以下同文。

こうしないとダメなんじゃないだろうか。

345 名前:132人目の素数さん投稿日:03/11/14 22:41
>>344
俺もそう思う

346 名前:132人目の素数さん投稿日:03/11/14 22:47
>>344とすると、>>337で正解、なのかな?
自分もそうなったんだけど…

347 名前:132人目の素数さん投稿日:03/11/14 22:48
(10^210)/(10^10+3)の整数部分のけた数と、1の位の数字
を求めよ。
ただし、3^21=10460353203を用いてよい。



348 名前:132人目の素数さん投稿日:03/11/14 23:19
4/5より大きいか否か、って聞かれるとちょっと困惑するね。
計算手順から言っても、直感で決められるものじゃないから。
>>340人数が増えるほど、信頼性は高くなるはずだけど(未確認)
たとえば登場人物が一人の場合の時から、
確率は99%を下回っているので(コインの枚数にもよるが)
100人の場合が99%より高いかどうかは一概には言えない。

かなり寝ぼけているので間違ってたらすまん。

349 名前:132人目の素数さん投稿日:03/11/14 23:25
>>337は当たってるけど、一人の場合の確率は4/5よりかなり低い。

350 名前:132人目の素数さん投稿日:03/11/14 23:27
魔方陣5*5を完成させよ(縦横斜の和が全て等しい)
使う数字は1〜25各1つづつ。また手順を説明せよ。

351 名前:132人目の素数さん投稿日:03/11/14 23:40
>>350
もう有名すぎてダsb;いうあsdgんさkjdんgぁ

352 名前:132人目の素数さん投稿日:03/11/15 00:12
>>348
Excel で試行してみた。

1人 0.114285714
2人 0.340425532
3人 0.673684211
4人 0.891986063
5人 0.970616114
6人 0.992488490
7人 0.998111483
8人 0.999527201
9人 0.999881758
10人 0.999970437
11人 0.999992609
12人 0.999998152
13人 0.999999538
14人 0.999999885
15人 0.999999971
16人 0.999999993
17人 0.999999998
18人 1

353 名前:333投稿日:03/11/15 00:12
337さん正解です はやいですねー
解答はいうまでもなく条件付確率です 
とりかかると単純だけど直感ではなにかひっかかる問題

354 名前:132人目の素数さん投稿日:03/11/15 00:30
pをある事象Xが起こる確率、qが一人が本当のことを言う確率、nがその人数だとすると
全員が「xが起こった」と言ったとき、本当にxが起こっている確率は
p*q^n/{(1-p)(1-q)^n + p*q^n}となり
q>1/2のとき、1-q<qなので
p*q^n/{(1-p)(1-q)^n + p*q^n}=p*q^(n+1)/{(1-p)(1-q)^n*q + p*q^(n+1)}  (分子分母*q)
<p*q^(n+1)/{(1-p)(1-q)^(n+1) + p*q^(n+1)}  (∵{(1-p)(1-q)^n*q>(1-p)(1-q)^(n+1))
よってq>1/2の時に限り、nが増えるほど信頼性は増す。

355 名前:132人目の素数さん投稿日:03/11/15 01:10
>>347
灯台文型の入試問題だった気がする。

356 名前:132人目の素数さん投稿日:03/11/15 02:57
単位正方形Sと同一平面上にあり、Sの各頂点からの距離が
0でない有理数であるような点は存在するか?

357 名前:132人目の素数さん投稿日:03/11/15 17:38
>>356
ヒント希望AGE


358 名前:132人目の素数さん投稿日:03/11/15 23:26
>>356
オレは答えキボンあげ

359 名前:132人目の素数さん投稿日:03/11/15 23:52
[x]でxを超えない最大の整数を表す。
a(n)= [ √(2*n - (7/4)) + 1/2 ]
と数列を定義するとき、
kを固定して、a(n)=kとなるnの個数をkで表せ。

360 名前:132人目の素数さん投稿日:03/11/16 00:13
>>356
問題を読み違えてなければ、いっぱいあると思う。

361 名前:132人目の素数さん投稿日:03/11/16 00:14
>>360
座標で書いてくれ。
正方形の各頂点の座標を(0,0)、(0,1)、(1,0)、(1,1)として
どこに点をおけばいいの?

362 名前:132人目の素数さん投稿日:03/11/16 00:24
問題読み違えてたぽ。
正方形の中に取らなきゃいけないのか。

363 名前:132人目の素数さん投稿日:03/11/16 00:31
>>362
外でもいいから例を示してくれ

364 名前:132人目の素数さん投稿日:03/11/16 00:36
ごめんちがったw

365 名前:132人目の素数さん投稿日:03/11/16 00:39
>>364
つまり外でもみつかってないでオケ?

366 名前:132人目の素数さん投稿日:03/11/16 00:40
オケ。多分ないんじゃないかなぁ

367 名前:132人目の素数さん投稿日:03/11/16 00:44
とりあえず、正方形の各頂点の座標を(0,0)、(0,1)、(1,0)、(1,1)として
点( x , y )からこれら四点へのきょりが有理数になるとき、x,yが有理数になる事は簡単に示せた。
っていうことは、背理法で示すとして、この図形を適当に拡大すれば、

各頂点、および、考えている点が全て格子点であって
かつ、それらの距離も整数値。という常識では考えられないパターンに行き着くわけだが・・・

368 名前:132人目の素数さん投稿日:03/11/16 00:52
>>367
>点( x , y )からこれら四点へのきょりが有理数になるとき、x,yが有理数になる事は簡単に示せた。

その証明おしえて

369 名前:132人目の素数さん投稿日:03/11/16 01:25
あるサイトからコピペ。

10人が輪になってすわっています。
それぞれの人に「右隣の人はどんな人ですか?」と聞いてみたところ,
全員が「私の右隣の人はうそつきだ!」と答えました。

さて,この中に正直な人は何人いるでしょう。

370 名前:132人目の素数さん投稿日:03/11/16 01:30
嘘つきが2連続することはない。
正直者が2連続することもない。
よってありうるのは嘘つき5人と正直者5人が
交互に座っている場合のみ。

371 名前:132人目の素数さん投稿日:03/11/16 01:51
>>368
(0,0)〜(x、y)への距離が有理数になるため、距離の定義より
√( x^2 + y^2 ) が有理数。この事から、x^2 + y^2=有理数^2であり、x^2 + y^2も有理数となる。
また、(0,1)〜(x、y)への距離が有理数である事から、
√( x^2 + y^2- 2y + 1 ) が有理数。x^2 + y^2 - 2y + 1が有理数になるので、上の結果とあわせれば
2y-1が有理数。結局、yが有理数となる。xも同様。


ダメかな? ひょっとして、これだけ分かれば、問題の解答ができたとか・・・
ちょっと期待!

372 名前:132人目の素数さん投稿日:03/11/16 01:57
>>368
A(0,0)、B(1,0)、P(x,y) とおき、
APとBPがともに有理数であるとする。

AP = √(x^2+y^2) = q/p
BP = √((x-1)^2+y^2) = s/r

これらを辺々2乗してx^2+y^2を消去すれば、
xは有理数になることがわかる。

yについてはB(1,0)の代わりにC(0,1)を考えればいい。

373 名前:372投稿日:03/11/16 01:58
かぶったすまん。

374 名前:132人目の素数さん投稿日:03/11/16 02:29
 サインカーブを書いて切り取って
丸めたとき円柱を切った切り口になる
ってのはどうやって証明するんでしょう?

375 名前:132人目の素数さん投稿日:03/11/16 02:40
>>374
分からないスレっぽい話だが・・・暇なので考えてみよう。
円柱を x^2+y^2≦1 -1≦z≦1 として、平面z=yで切ったときを考える。
その断面の周はパラメータθを用いて
x=cosθ y=sinθ z=sinθ
と表される。
よって、これを広げると、サインカーブができあがる。逆も明か。。。

あんま、面白くない。スレ違い。

376 名前:132人目の素数さん投稿日:03/11/16 03:00
>>356
今日は出題者みてないのかな?
ヒントほしい。これ
 
>同一平面上にあり
 
ってのは正方形の外部もかんがえないとダメなの?

377 名前:132人目の素数さん投稿日:03/11/16 03:00
ヒントくれアゲ

378 名前:132人目の素数さん投稿日:03/11/16 03:07
>>376
外部ありなら明らかに存在するっつーような問題じゃないんだから、
仮に出題者が内部のみと考えてたとしても、
普通に外部ありで考えてやればいいじゃないか。

まあ俺はもともと外部ありでの出題だと思ったけど。

379 名前:132人目の素数さん投稿日:03/11/16 03:09
普通に考えると平面って言うのは、正方形内部の事を指してる訳じゃないよ。
だから、同一平面って言ったら、正方形内部に限定される物じゃない。

内部、外部、周上、全てを考えなきゃダメ。・・・なんだが・・・ちょっと証明が思い浮かびそうだな。

380 名前:132人目の素数さん投稿日:03/11/16 03:13
>>378-379
なんとなく内部、周はできそうなんだけど外部が・・・まったく手のつけようがないん・・・
どうしたもんかと・・・もしかして出題ミスじゃないのかなと思ったんだけど。
そうだねオレもまさか>>356の文面がうっかり内部限定をわすれてるとは思えないんだけど。
外部の場合の証明がまったく手がでない・・・。

381 名前:132人目の素数さん投稿日:03/11/16 03:21
さて、背理法でいってみよう。 ちょい、コピらせてもらう。

A(0,0)、B(1,0)、P(x,y) とおき、 APとBPがともに有理数であるとする。
AP = √(x^2+y^2) = q/p   BP = √((x-1)^2+y^2) = s/r
これらを辺々2乗してx^2+y^2を消去すれば、
xは有理数になることがわかる。 yも同様。

また、△XYZにおいて、X,Y,Zの座標がどれも有理数で表されるのならば、
その面積も有理数になる。なぜならば、X,Y,Zを平行移動させ、Xが原点になるようにすれば、
Y,Zの座標を(y(1),y(2)) (z(1),z(2))として、| det( (y(1),y(2))(z(1),z(2)) ) |/2が△XYZの面積。
これは有理数である。

以上より、PA,PB,PC,PDが有理数になるときは、適当な自然数を見つけ全座標を等倍してやることにより
各頂点の値が整数、△PAB、△PBCなどの面積も整数値をとる事が分かる。

・・・ぜーんぜん、分かりません。

382 名前:132人目の素数さん投稿日:03/11/16 18:05
中間結果(0,1)(0,9)(1,0)の3点からの距離がすべて有理数であるような
点は(少なくとも1つは)存在する。これに加えて今のところ(1,1)からの
距離が有理になるような点の存在は否定出来ていない.
(x,y)=(80/45,84,45)
x^2+y^2=(116/45)^2
(x-1)^2+y^2=(91/45)^2
x^2+(y-1)^2=(89/45)^2



383 名前:132人目の素数さん投稿日:03/11/16 18:06
>>382
(0,9)=>(0,0)です。訂正します。

384 名前:132人目の素数さん投稿日:03/11/16 18:29
(x,y)=(80/45,84/45) ですね。

385 名前:132人目の素数さん投稿日:03/11/16 18:38
なんとなく証明できたかも・・・面積つこた。

386 名前:132人目の素数さん投稿日:03/11/16 19:37
>>385
お願い。

387 名前:132人目の素数さん投稿日:03/11/16 19:39
>>386
すまん。まちがってた。吊ってくる・・・

388 名前:132人目の素数さん投稿日:03/11/16 20:10
>>382
>(0,1)(0,0)(1,0)の3点からの距離がすべて有理数であるような
>点は(少なくとも1つは)存在する。

(3/4,0)とか(12/5,0)とか色々(無限に)あるでよ。

389 名前:132人目の素数さん投稿日:03/11/16 20:15
やはり証明は、ピタゴラス数をこね回す類のものになるんだろうかね。

390 名前:132人目の素数さん投稿日:03/11/16 20:40
出題者のカキコらしきもの全然ないんだけど。これホントに絶対正しいの?自信あるの?

391 名前:132人目の素数さん投稿日:03/11/16 20:43
x軸、y軸上に無いことくらいは証明できないかなあ。
考えたけど、難しいなあ、これ。

392 名前:132人目の素数さん投稿日:03/11/16 20:56
>>390
正しいも何も、出題者はあるかどうかを聞いてるだけだろ。

393 名前:132人目の素数さん投稿日:03/11/16 21:00
x軸上、y軸上にもありそうな気がする…
手計算では見つからんけど

394 名前:132人目の素数さん投稿日:03/11/16 21:08
>>392
いや、正解があるにせよないにせよ証明、もしくは実例をあげる必要がある。
実例のほうならまちがいもなにもないけど証明の方ならもってる証明がホントに
あってるのかどうかは疑わしいよ。何人もの査読を通ってるわけじゃないんだから。

395 名前:392投稿日:03/11/16 21:11
>>394
あぁ、つまり、自分で証明できたのかってことか。
別に考えるのが面白ければ、解決してない問題を出す事もあっていいと思ったから
上みたいに書いてみた。

ただ単純に、単位正方形に対して、条件を満たすような点が存在するかどうか
それを聞くだけだったら、解決できるかどうかはともかく、問題としては間違ってない
って言ってみただけ。   スマソ

396 名前:132人目の素数さん投稿日:03/11/16 21:16
とりあえず自分できちんと答えもってるのかどうかだけでも答えてもらわないと
はじまらないんだが。数学板には“この問題はきっととけるハズだ”みたいな問題を
この手のスレに平気ではる香具師もいるし。

397 名前:132人目の素数さん投稿日:03/11/16 21:25
>>391
ってか、x軸、y軸上にないことを示せばあとは簡単なんじゃ?
んなことないかな。

398 名前:132人目の素数さん投稿日:03/11/16 21:37
>>397
なんで?

399 名前:132人目の素数さん投稿日:03/11/16 23:12
誰か四次元の場合のヘロンの公式知ってるヤツいないか?
あれを使えば、この問題が解けるような気がしてきた。

点Pと正方形ABCDの五点を考えて、どの二点を結んでも線分の長さが整数になる。
このとき、四次元単体PABCDの体積を考えて、仮に0にならないようにできるのなら、
平面性から矛盾を導く事ができるのだが・・・誰か知らんか?

400 名前:132人目の素数さん投稿日:03/11/16 23:17
正方形の辺が軸に平行でないならいくらでも例がつくれるからその方針では無理くさい。

401 名前:132人目の素数さん投稿日:03/11/16 23:19
そうだったな。 はやとちった。

402 名前:132人目の素数さん投稿日:03/11/16 23:26
いやまて・・・
例えば、一辺の長さが5の菱形で、対角線の長さが6,8の物を考えると、確かに
四角形ABCDと点PでAB,BC,CD,DA、PA,PB,PC,PDが整数の例を作ったりすることが可能だけど、
問題の場合、AB=BC=CD=DA PA≠PD PB≠PC (※) が成立しているだろ。ここら辺利用できんかな?


正方形ABCDの対角線をADとする。仮にPA=PDとすれば点Pは対角線BC上(または、その延長線上)に存在する。
ABが整数である事から、BCは無理数。よって、PB+PCが無理数になるか、またはPB-PCが無理数。
どちらの場合も、PB,PCの少なくとも一つは無理数になるため、条件を満たさない。

403 名前:132人目の素数さん投稿日:03/11/16 23:29
>>402
嘘だ。PC≠PA PB≠PDだった。

役にたってない・・・

404 名前:132人目の素数さん投稿日:03/11/17 18:22
>>347できないの?

405 名前:132人目の素数さん投稿日:03/11/17 22:32
>>404
外出すぎでつまらんの。前の方でそう指摘されてるじゃん。

406 名前:132人目の素数さん投稿日:03/11/20 22:33
   あげ

407 名前:132人目の素数さん投稿日:03/11/21 00:12
具体的な数字が一切与えられずに、関数が求まってしまうのが興味深い問題。

次の条件を満たす関数fのうち、最も次元の低いものを求めよ。

fと単位円が共有するすべての点で、ある正方形をなす。

408 名前:132人目の素数さん投稿日:03/11/21 00:19
f(x) = ±√2/2

409 名前:132人目の素数さん投稿日:03/11/21 00:27
>>407
問題の意味がわからん

>>408で正解なの?

410 名前:132人目の素数さん投稿日:03/11/21 00:48
次元?次数?

411 名前:132人目の素数さん投稿日:03/11/21 00:48
>>408は明らかに間違いだろ。
とりあえず3次式になりそうだが計算がめんどくさい。

412 名前:132人目の素数さん投稿日:03/11/21 00:58
>>407
f(x)はxについての多項式・・・だよね?

413 名前:132人目の素数さん投稿日:03/11/21 01:47
>>407
ある正方形をなす ってのは 正方形の4頂点となる ってこと?

414 名前:132人目の素数さん投稿日:03/11/21 02:27
具体的な説明が一切与えられず、問題の答えが求まってしまう興味深い問題。
>>407


415 名前:132人目の素数さん投稿日:03/11/21 02:30
次元っつうぐらいだから、多変数関数なのだろう・・・。

416 名前:132人目の素数さん投稿日:03/11/21 02:43
>>414
ワラタ

417 名前:132人目の素数さん投稿日:03/11/21 03:22
問題きれたみたいだから出題してみる。
 
※※※※※問題※※※※※※
f(x)を最高次の係数が1である整数係数の次数nの多項式とし
α1・・・αnをf(x)=0の解とする。D=Π[i<j](αi-αj)^2とおく。
f'(x)x^(i-1)をf(x)でわったあまりを(ai1)+(ai2)x+(ai3)x^2+・・・+(ain)x^(n-1)とおく。
(つまりaijは剰余の多項式の係数。)行列AをA=(aij)とおく。
(detA)/Dをもとめよ。
 
結構ムズイと思う。

418 名前:417投稿日:03/11/21 03:25
あ、A=(aij)のiは1〜nまでうごかしまつ。つまりA=(aij)はf'(x)x^(i-1)をf(x)でわった
あまりの係数をi=1〜nまでならべたn次の正方行列でつ。

419 名前:132人目の素数さん投稿日:03/11/21 04:01
P(x)をxについての多項式として、P(2)=nを満たすとする。(ただし、nは自然数)
P(x)の係数が0,1,2,3のどれかであるとき
考えられる多項式P(x)の総数をnを用いて表せ。

420 名前:132人目の素数さん投稿日:03/11/21 10:22
整数論で出てくる話だな

421 名前:132人目の素数さん投稿日:03/11/21 23:44
>>419
ヒントきぼんアゲ

422 名前:132人目の素数さん投稿日:03/11/21 23:57
>>419
わかった。
係数が0,1,2,3である多項式の全体をSとおく。S(k,n)を{P∈S|P(k)=n}とおく。
もとめたいのはS(2,n)対応f:S×S→Sを(Q,R)→P(x)=Q(x^2)+xR(x^2)でさだめる。
この対応は全単射でP=f(Q,R)のときP(2)=Q(4)+2P(4)。よって
#S(2,n)=納m=0,[n/2]]]#S(4,m)×#S(4,n-2m)。
一方で4進数展開をかんがえればS(4,n)=1(∀n)。
∴S(2,n)=[n/2]+1。

423 名前:132人目の素数さん投稿日:03/11/22 17:04


424 名前:132人目の素数さん投稿日:03/11/24 04:58
                    .    /\
                       / あ \
                      ,´彡  げ ..\
                     ノ ノっ\  て \
                   ./ /´   \  も .\
                 / /´.      \  い \
                 |  / /ヘ;;;;;      .\  い \
                 |  ) ';=r=‐リ        \ か. \
                 |    ヽ二/         \ ?....>
                 )ヽ `_  〈、_ _、___,-っ_ / |!
                 〉,; /   ,⌒´  ,,____,,、τイミ、
                 /  `  イ/⌒ ̄      .⌒` i!|!;
              /⌒\   /              ;i!|
            / 人  ヽ、/               .!i
           / ,/´ |\. .\               !;
          / ;/    > ヽ  )

425 名前:132人目の素数さん投稿日:03/11/24 16:11
次スレは 「面白い問題おしえてたも 八匁」 だな

426 名前:132人目の素数さん投稿日:03/11/27 03:25
A(n)を1からnまでの素数の積とする。
このとき尿(n)/n!(n=1〜∞)を求めてください。

427 名前:132人目の素数さん投稿日:03/11/27 23:12
あげ

428 名前:132人目の素数さん 投稿日:03/11/28 01:14
>>356
遅い上俺数学得意じゃないから間違ってるかもしれないけど
細かい定義は省いて極座標で考えると
APをLとする。Lは有理数。AP^2=L^2(有理数)、BP^2=有理数+sinR
CP^2=有理数+sinR+cosR、DP^2=有理数+cosR
sinRとcosRが同時に有理数になるのはπ/2の倍数の時ですよね?
よって点Pは正方形の辺の延長上以外に存在しない
で (L^2+1)^(1/2) はLが0以外の時無理数だよね?
だから題意に合う点Pは無い。

429 名前:428投稿日:03/11/28 01:28
もろ間違いでした...

430 名前:132人目の素数さん投稿日:03/11/28 03:22
>>429
これ未解決問題だってさ
救済スレってのがあるから、見てみな。

431 名前:132人目の素数さん投稿日:03/11/30 09:30
ふと思いついた問題。面白くないかも。

まんじゅうを買うために4人の人が並んでいます。
彼らは出来るだけたくさん買いたいと思っているのですが、
一度に買えるのは3つまでと決まっていて、
nこ買ったらn人分後ろに並びなおさなければ
ならないというルールがあります。
(1個買ったら1人に順番を譲る。
3個買ったら一番後ろに並びなおし。)
まんじゅうは全部で10個あります。
彼らはそれぞれ何個買えるでしょう。

432 名前:132人目の素数さん投稿日:03/11/30 10:56
>>431
自分の総獲得数が同じであるケースが2通り以上あった場合、
どういう戦略をとるの?

例えば、できるだけ多くの人にまんじゅうを与えたいと思うのか、とか。

433 名前:132人目の素数さん投稿日:03/11/30 14:01
>>431
順に買っていけば5個づつ買えるということで、前2人が結託する。
しかし、3個づつ買った時点で裏切ったほうが6個ゲット。
裏切られた方が4個購入。とか。

まあ、この条件では決まらないかな。

434 名前:132人目の素数さん投稿日:03/11/30 14:55
>>426


435 名前:132人目の素数さん投稿日:03/11/30 15:14
>>426 解けそうで解けん


436 名前:132人目の素数さん投稿日:03/11/30 21:35
>>426は「素数名人スレ」で当時素数名人をなのってたコテハンが自分で思いついた
とけるアテもなくつくった問題。

437 名前:132人目の素数さん投稿日:03/11/30 21:54
また未解決問題かよ

解答用意してない出題者は
ちゃんとその旨を銘記してくだちい

438 名前:132人目の素数さん投稿日:03/11/30 21:55
凸多面体の辺の数をkとする。
k=7となる凸多面体は存在しない事を示せ。

k=6、または、k≧8を満たす自然数kに対して
辺の数がk個となる凸多面体が存在する事を示せ。

439 名前:438投稿日:03/11/30 21:55
出題From数蝉

解けるよ。

440 名前:132人目の素数さん投稿日:03/11/30 21:59
>解けるよ。
 
そらそやな・・・

441 名前:132人目の素数さん投稿日:03/11/30 22:07
まだ頭の中でうまく整理できて無いのでめちゃくちゃな日本語になるが。
まだ正しいのか判断してないし直感なんでトンデモとレスが付くかも知れないが。

3次元空間内で多面体が多面体として存在するためには
各頂点から3本以上の辺が無ければならない。
しかしk=7の場合、全ての頂点から出てる辺の数を3本以上にする事が出来ない。
よって、k=7の多面体は存在しない。終わり

tu-ka、これ辺が曲線ってのもありだったらk=7の多面体が存在する事になるよな?
馬鹿な俺に教えて偉い人 もしくはエロイ人

442 名前:132人目の素数さん投稿日:03/11/30 22:13
各頂点から3本以上の辺が無ければならない。
しかしk=7の場合、全ての頂点から出てる辺の数を3本以上にする事が出来ない。

ここを証明しなきゃ駄目だよ。

443 名前:132人目の素数さん投稿日:03/11/30 22:41
辺の数が7の凸多面体があるとする。
もし4角形以上の面があれば辺の数は8以上。
つまりあればすべての面は3角形でなければならない。
面の数をsとするとき3s=2・7でないとだめだけどそんなこた−ない。

444 名前:132人目の素数さん投稿日:03/12/01 00:04
>>438
後半教えて

445 名前:132人目の素数さん投稿日:03/12/01 00:16
>>436
ありがとう。

446 名前:132人目の素数さん投稿日:03/12/01 00:17
n≧3にたいし(0,0,1)と(cos(kπ/(2n)),sin(kπ/(2n)),0) (k=0〜n-1)のn+1点の凸閉は辺の数が2nになる。
さらに(cos(π/(4n),sin(π/(4n),1/2)をくわえた凸閉は辺の数が2n+3になる。
よって辺の数が6,8,9,・・・の凸多面体は存在する。
ちなみに
辺の数がnで三角形の面を少なくともひとつ凸多面体が存在する
⇒辺の数がn+3で三角形の面を少なくともひとつ凸多面体が存在する
が証明できることと
辺の数が6,8,9で三角形の面をもつ凸多面体は具体的に構成できるので
(それぞれ正4面体、底面が正方形である錐、正四面体2つを一つの面ではりあわせたもの)
これから帰納的にもしめせる。

447 名前:132人目の素数さん投稿日:03/12/01 02:01
辺の数が6,8,10で三角形の面をもつ凸多面体は具体的に構成できるので
(それぞれ正4面体、底面が正方形である錐、底面が正5角形形である錐)
これから帰納的にもしめせる。
↑これに訂正。

448 名前:132人目の素数さん投稿日:03/12/01 04:17
pを奇素数u,v,wをpと互いに素である整数とするとき合同方程式
ux^2+vy^2+wz^2≡0 (mod p)
の解の個数がp^2-1であることをしめせ。

449 名前:132人目の素数さん投稿日:03/12/02 19:25
↑まちがった。(x,y,z)=(0,0,0)をわすれてた。
の解の個数がp^2であることをしめせ。に訂正


450 名前:132人目の素数さん投稿日:03/12/04 01:14
>>417-418
↑これ面白くない?挑戦中の人とかいない?答イラネ?

451 名前:132人目の素数さん投稿日:03/12/04 02:15
※※※※※問題※※※※※※
f(x)を最高次の係数が1である整数係数の次数nの多項式とし
α1・・・αnをf(x)=0の解とする。D=Π[i<j](αi-αj)^2とおく。
f'(x)x^(i-1)をf(x)でわったあまりを(ai1)+(ai2)x+(ai3)x^2+・・・+(ain)x^(n-1)とおく。
(つまりaijは剰余の多項式の係数。)行列AをA=(aij)とおく。
(detA)/Dをもとめよ。

ここに貼ってみる。

452 名前:132人目の素数さん投稿日:03/12/05 03:13
次の条件を満たす正の整数nをすべて求めよ。

条件:nは(2^n)+1を割り切る

453 名前:132人目の素数さん投稿日:03/12/05 03:33
たくさんあり過ぎて困る。

454 名前:132人目の素数さん投稿日:03/12/05 05:21
f(x)x^(i-1)をf(x)でわったあまりは常に0ではないんでしょうか?

455 名前:132人目の素数さん投稿日:03/12/05 06:10
f'(x)とf(x)は違う罠。

456 名前:132人目の素数さん投稿日:03/12/05 06:51
★☆★もうすぐクリスマス★☆★おまんこプレゼント★☆★
http://marimo.s6.x-beat.com/marimo.html

457 名前:132人目の素数さん投稿日:03/12/05 07:37
>>451
B=(bij)、bij=(αj)^(i-1) とおけば、detB≠0であり

さらに、C=(cij) cij=δij*f'(αi) とおいて、BC=ABとなることから detA=detC=Π[i=1〜n]f'(αi) ・・・@

f(x)=Π[i=1〜n](x-αi) より f'(αi)=-αiΠ[j≠i](αi-αj)

従って D=Π[i=1〜n]f'(αi)=(-1)^(n(n+1)/2)*Π[i=1〜n]αi ・・・A
@Aより
(detA)/D=(-1)^(n(n+1)/2)*Π[i=1〜n]αi となる。


458 名前:132人目の素数さん投稿日:03/12/05 12:51
他スレで誰も答えてくれましぇん
@与えられた四面体の6つの2面角(即ち隣り合う面の間の角)の内5つが等しいときこの四面体は正四面体であるかどうかを示せ。
A1辺の長さが2の立方体の内部(表面とは限らない)に立方体の最も遠い2つの頂点を結んでいる折れ線がある。折れ線の頂点は立方体の表面にあり折れ線を構成する各辺の長さは3である。このような折れ線の辺の数の最小値を求めよ。
B平行で相違なる2枚の平面Π1,Π2上に各々凸多角形α=A1A2...Am,β=B1B2...Bnがある。点P,Qが各々多角形α,β上を動くとき線分PQが動いてできる立体Tをα,βを底面とするプリズム体と呼ぶ。Π1とΠ2の丁度中央(両平面から等距離)にある平面Π3によるTの切り口をμとする。
α,β,μの面積がa,b,mであり,Π1とΠ2の間の距離がhであるときプリズム体Tの体積をa,b,m,hを用いて表せ。

459 名前:132人目の素数さん投稿日:03/12/05 16:54
ここは宿題スレじゃないので放置
つかマルチか?

460 名前:132人目の素数さん投稿日:03/12/05 18:15
マルチで解答済みだす

461 名前:132人目の素数さん投稿日:03/12/06 17:08
平面上に有界な凸図形Sがある。
次の問に答えよ。

(1) Sの面積を二等分する直線が無数に存在する事を示せ。
 また、Sの面積を二等分する直線を全て求めよ。

(2) ある点Pが存在し、(1)で求めた無数の線分のうち少なくとも
 三本がその点で交わる事を示せ。

(3) 点Pのうち条件を満たす物を一つ求めよ。

(4) (1)で求めた直線のうち、どの4つをとっても、同一の点で交わらないような
 図形Sが存在する事を示せ。

462 名前:132人目の素数さん投稿日:03/12/06 17:09
>>461

(3)までは解答を持っています。 (4)は適当なので、外れてるかも知れないし
未解決かも知れないし、すぐに解かれるかも知れません。

463 名前:132人目の素数さん投稿日:03/12/06 22:30
超難問
定規だけで(2点を結ぶ直線を引くだけで)正方形の面積を7等分する方法を記せ。

464 名前:132人目の素数さん投稿日:03/12/06 23:01
正方形の一辺をSとすると正方形の面積はS^2
七等分された面積の


この方法じゃ詰まることが分かったから書くのやめた

465 名前:132人目の素数さん投稿日:03/12/06 23:05
正方形ABCDにおいて線分ABをB側に伸ばした先に点Eをとる。
DEとBCの交点をFとし、AFとBDの交点をGとする。
EGを伸ばした直線とADの交点をHとするとAE*EF*DH=BE*DF*AHより
HはADの中点となる。

BHを伸ばした直線とCDの交点をIとするとCD=DI。
同じようにしてABをA側に延ばした先にAJ=ABとなるJが取れるので
ABCDの隣に正方形ADIJを作れる。

A(0,0) B(1,0) C(1,1) D(0,1)とすると上の方法によって任意の格子点を
求められる事が分かるので、a=1〜6について(-a,1)と(7-a,0)を結んだ線分と
ADの交点を求めればADを7等分出来る。BCも7等分出来るので
ABCDも7等分出来る。


466 名前:132人目の素数さん投稿日:03/12/06 23:40
>BHを伸ばした直線とCDの交点をIとするとCD=DI。

ここが嘘

467 名前:132人目の素数さん投稿日:03/12/06 23:47
CD=2DIだろバカ

468 名前:132人目の素数さん投稿日:03/12/06 23:52
>>463
過去ログ探せばどっかにあるぞ。

469 名前:132人目の素数さん投稿日:03/12/07 00:08
AからEまでの長さをnとすると
n*CD=DIになる。と思う。
証明はシテナイガナー
Function Viewで再現してみたらそんな感じになった

470 名前:132人目の素数さん投稿日:03/12/07 00:27
コンパス使えない=長さを移せないってこと?

471 名前:465投稿日:03/12/07 00:59
http://w2.oekakies.com/p/2chmath/49.png?99
AH=DHで直線ABと直線CDが平行だからAJ=DI=AD=IJになると思うけど…違う?

472 名前:132人目の素数さん投稿日:03/12/07 01:30
>>465
さくらスレ82の489辺りに昔答えを書いたぞ

473 名前:132人目の素数さん投稿日:03/12/07 01:48
>>465
>求められる事が分かるので、a=1〜6について(-a,1)と(7-a,0)を結んだ線分と

(-a,1)じゃなくて(0.1)でない?
それ以外はOKだと思う。

474 名前:132人目の素数さん投稿日:03/12/07 01:51
>>473
(0,1)→(1.0)に訂正

475 名前:132人目の素数さん投稿日:03/12/07 01:57
>>473
更に訂正

訂正はなかったことに。

476 名前:132人目の素数さん投稿日:03/12/07 02:00
>>475
>訂正はなかったことに。

訂正→指摘 に訂正

i.e. >>473それでOK

477 名前:132人目の素数さん投稿日:03/12/07 02:02
>>473-476
明日休みといっても無理して起きてる必要は無いぞ。
寝た方がいいんじゃね?

( ´∀`)いい夢見ろよ

478 名前:132人目の素数さん投稿日:03/12/07 02:06
>>477
これからエアマスの実況がありますから

479 名前:465投稿日:03/12/07 02:09
>>473-476
確認作業に感謝

>>472
そっちの解答は正方形から線分がはみ出てないのね。素晴らしい

>>463のように正方形内に定規のみを使って作図出来る点or線分は、
 『途中で引く線分が正方形内からはみ出さないようにする』というルールを
 加えても必ず作図出来る事を示せ」
って問題を昔考えたけど『作図』を厳密に考えるのに難儀してた結果止めた記憶がある。
でも結局これは正しいような気がする。

それじゃ自分も寝ます
 

480 名前:132人目の素数さん投稿日:03/12/07 02:30
>>461
教えてAGE

481 名前:132人目の素数さん投稿日:03/12/07 03:04
http://66.98.132.154/upro/source3/No_0002.mpg

482 名前:132人目の素数さん投稿日:03/12/07 19:05
>>457
ちがうよ。f(x)=x^2-3x+2とかで実験してミソ。

483 名前:132人目の素数さん投稿日:03/12/07 20:14
>>482
>f(x)=Π[i=1〜n](x-αi) より f'(αi)=-αiΠ[j≠i](αi-αj)

>従って D=Π[i=1〜n]f'(αi)=(-1)^(n(n+1)/2)*Π[i=1〜n]αi ・・・A
>@Aより
>(detA)/D=(-1)^(n(n+1)/2)*Π[i=1〜n]αi となる
ここを大幅修正(というか微分を間違えてどうする漏れ!)

f(x)=Π[i=1〜n](x-αi) より f'(αi)=Π[j≠i](αi-αj)

従って detA=Π[i=1〜n]f'(αi)=Π[i=1〜n]Π[j≠i](αi-αj)
          =(-1)^(n(n-1)/2)Π[i<j](αi-αj)^2=(-1)^(n(n-1)/2)*D …A
@Aより
(detA)/D=(-1)^(n(n-1)/2) となる

484 名前:132人目の素数さん投稿日:03/12/07 21:52
>>483
御名算

485 名前:132人目の素数さん投稿日:03/12/08 00:30
>>472
再うp禿しく希望。

486 名前:132人目の素数さん投稿日:03/12/08 01:41
>>461
答えきぼんアゲ

487 名前:132人目の素数さん投稿日:03/12/08 02:38
>>485
さくらスレ82
http://science.2ch.net/math/kako/1048/10486/1048630335.html

488 名前:132人目の素数さん投稿日:03/12/08 16:41
>>461

えーと、ヒントはまだ出しません。
ちなみに(4)は自力で解けて、どうやら簡単な形をしたある図形が
条件を満たしてくれるようです。

489 名前:バカ投稿日:03/12/08 17:16
>>164の答えお願いします。

490 名前:132人目の素数さん投稿日:03/12/08 17:46
もらえる玉の個数の期待値なら、直観で20個取り出した時が最大かな。

491 名前:132人目の素数さん投稿日:03/12/08 18:39
秤を使わないでジャンボジェット機のだいたいの重さを知りたい。
どうすればよいか。

492 名前:132人目の素数さん投稿日:03/12/08 18:44
とりあえず「ジャンボジェット 重量」でぐぐってみる。

493 名前:132人目の素数さん投稿日:03/12/08 19:00
>>461
全部解けたつもり。が、書くのがメンドイ。
直線を連続的に動かしていけばいいんだよな。

(1) では、Sの周上の一点 R を固定しておくと、
R を通る直線によって S が二つに分割されている。
小さい方の面積を大きく、大きい方の面積をするように直線を動かすと
どこかで面積のつりあいがとれる。

(2) は、面積を二等分する直線(L)を一本固定する。
その直線のコピー( L'(t) )をSをニ等分する性質を保たせたまま
半回転させる。そのときの、LとL'(t)の交点Q(t)の移動をおっていくと
ある t で Q(t) = Q(t') となる。このとき、L, L'(t), L'(t') は一点で交わる。

(4) は、正三角形でどう?

494 名前:132人目の素数さん投稿日:03/12/08 19:03
正三角形でいいならどんな三角形でもいい。


495 名前:493投稿日:03/12/08 19:15
じゃあ半円でファイナルアンサー。

496 名前:132人目の素数さん投稿日:03/12/08 19:17
>>491は某社の入社試験。

497 名前:132人目の素数さん投稿日:03/12/08 19:31
MSだな

498 名前:132人目の素数さん投稿日:03/12/08 23:07
ある立体がある。これをどんな平面で切断しても、切り口は円になるという。

球以外に上記の性質を満たすものがあれば例をあげ、
なければそれを証明せよ。



499 名前:132人目の素数さん投稿日:03/12/08 23:28
>>495
半円とは?

500 名前:132人目の素数さん投稿日:03/12/08 23:57
>>493
(3)は重心

501 名前:132人目の素数さん投稿日:03/12/08 23:59
>500
重心がどうしたの?

502 名前:132人目の素数さん投稿日:03/12/09 00:04
>>498
ないと思う。3次元の場合は簡単に証明出来る。

503 名前:132人目の素数さん投稿日:03/12/09 00:20
いや、>>461の(3)の答えとして重心が挙げられると思って。。。

証明もあるけど、いる?

504 名前:バカ投稿日:03/12/09 00:43
>>164の解き方お願いします。

505 名前:132人目の素数さん投稿日:03/12/09 00:51
>>502
証明してよ

506 名前:132人目の素数さん投稿日:03/12/09 00:54
>>503
おながいします。

507 名前:132人目の素数さん投稿日:03/12/09 01:04
>>505
502 じゃないけど。こんな筋書き?

(1) 凸である。
(2) 有界である。
(3) 半径が最大となる切り口の中心が球の中心。

508 名前:132人目の素数さん投稿日:03/12/09 01:30
一つ平面をとる。
その平面上の円の中心を通りその平面に垂直な平面を全部考える。
垂直な平面上の円の中心は最初の円の中心を通り
最初の平面に垂直な直線上にあるので
その直線との共通部分が全ての円の直径になるので球。


509 名前:132人目の素数さん投稿日:03/12/09 04:52
レベル低下中

510 名前:132人目の素数さん投稿日:03/12/09 05:10
題意の性質を持つR^3の部分集合をSとする。

切り口が常に1点の場合、Sは1点よりなる

切り口が半径r(>0)の円となるものがあるときその一つをD1その中心をO_1とする。
O_1を通りD1に垂直な平面を一つ取りその平面でのSの切り口の円をD2その中心をO_2とする。
∂D1と∂D2の交点をP、QとしO_1、O_2をとおる直線をLとする。

ここで中心O_2 半径 PO_2の球S_0を考えると
S_0はD1、D2を切り口に持ち題意を満たす。

逆に互いの乗る平面が直交し共通部分が一方の(ここではD1とする)中心を
通る線分となるような2つの円盤D1,D2をとり D1,D2を切り口に持ち題意を満たす集合をSとしたとき
D1∩D2=PQ D1の中心をとおりD2に直交する直線をL ∂D1=C1、∂D2=C2 として

Lを含む平面によるSの切り口に現れる円は少なくとも
C1上のある点とそのC1での対蹠点、そしてC2とLとの交点の4点を通らなくてはならず
これはC1,C2(従ってD1,D2)により一意的に定まる。

以上のことから題意を満たすSは球となる。

511 名前:訂正投稿日:03/12/09 05:13
>D1∩D2=PQ D1の中心をとおりD2に直交する直線を…
D2に直交→D1に直交

512 名前:132人目の素数さん投稿日:03/12/09 20:09
>>508でいいんじゃないの?

513 名前:132人目の素数さん投稿日:03/12/09 22:14
「ある立体がある。これをどんな平面で切断しても、切り口は円になるという。」
を少し変えて
「ある立体がある。これをどんな平面で切断しても、切り口はある特定の図形Aに相似となる。」

とすると>>498の問題の難易度が上がる。

514 名前:132人目の素数さん投稿日:03/12/10 01:26
レベル低下中

515 名前:132人目の素数さん投稿日:03/12/10 01:30
>>448-449を誰かやってみてちょ。

516 名前:132人目の素数さん投稿日:03/12/12 15:30
Aにどんな条件をつけたらいいんだ。
思いついた問題を書き込めばいいってもんなじゃない。





x^7+7*y^7が素数となる時のx,yの性質若しくは、その素数の性質を考えよ。

517 名前:132人目の素数さん投稿日:03/12/13 20:32
>>516
答えおしえてくれAGE

518 名前:132人目の素数さん投稿日:03/12/14 01:25
>>517

516は思いついた問題を書き込めばいいってモンじゃないっていう
実例を示しただけだと思うが・・・

519 名前:132人目の素数さん投稿日:03/12/14 01:27
>>518
でも答えが知りたいsage

520 名前:132人目の素数さん投稿日:03/12/14 01:33
>>518
そうなのか・・・なんだ・・・ちなみに>>513は解けないの?つまり
「ある立体がある。これをどんな平面で切断しても、切り口はある特定の図形Aに相似となる。」
つまり
「ある立体がある。これをどんな平面で切断しても、切り口はすべて相似となる。
このときある立体はどんなものか? 」
と読めるとおもうんだけどこれって球しかないような気はするんだけど。
到底とけそうにないの?

521 名前:132人目の素数さん投稿日:03/12/14 01:33
>>516
x,yを実数とする。明らかにx^7+7*y^7は任意の素数をとりうる。
よって、素数をpとしてp=x^7+7*y^7ならば、pは任意の素数である。

また、x^7,7*y^7は任意の実数値をとりうるので、どのようなxに対しても必ず
あるyが存在し、p=x^7+7*y^7が成立する。
従ってxは任意の実数。


ほれ。どうだ。文句あるか?

522 名前:132人目の素数さん投稿日:03/12/14 01:44
>>513は見た事があるんだけどなぁ。解き方は忘れた。

解けるはずだと思う。根拠はない。ただ、思いっきり記憶のどこかに引っかかってる。

523 名前:132人目の素数さん投稿日:03/12/14 01:47
>>520
自分は解けなかったんでスレを引用しとく
http://cheese.2ch.net/math/kako/979/979198460.html

>>522
貴方は↑のスレの誰かさんですか?

524 名前:132人目の素数さん投稿日:03/12/14 01:52
>>523
おお、こんなスレが昔あったのか・・・でもこのスレでも結局未解決のまま終わったんだね。

525 名前:132人目の素数さん投稿日:03/12/14 01:57
>>523
そこのスレの事は知らんかったが、10年ほど前に見た事がある問題。
その時は数学の知識も乏しく、この問題が自分に解けるのか?ということはおろか
未解決問題なのか、それとも、解決した問題なのかすらわからなかった。

当時の俺の行動から考えて、数学セミナーのエレガントな解答を求むあたりを
探してみると、案外見つかるかも知れない。今度の休みに行ってみる。
ちなみに・・・いつになる事やら・・・

526 名前:132人目の素数さん投稿日:03/12/14 02:13
さて、口直しに少々簡単な問題を出すとするか。どこかの国の数学オリンピックから出題だ。

1でない任意の有理数 k に対し、kに対応する、次の条件を満たした二つの集合A,Bが
存在する事を示せ。

1) A∪B = N  ただし、Nは正の整数の集合
2) A∩B = 空集合
3) Aに属するどの二つの自然数x,yをとってもx/yはkにならない。
  また、Bに属するどの二つの自然数z,wをとってもz/wはkにならない。

527 名前:132人目の素数さん投稿日:03/12/14 02:29
>>526
できた。
素数pを(k,p)_p=-1であるようにとる。(ただし(-,-)_pはヒルベルト記号)
A,Bを
A={n|(n,p)_p=1}
B={n|(n,p)_p=-1}
とおけばこれがもとめるものである。・・・ダメ?

528 名前:132人目の素数さん投稿日:03/12/14 02:30
あ、だめだ。しまった。kが平方数ならそんな素数pはとれないや。吊ってきます。

529 名前:132人目の素数さん投稿日:03/12/14 02:43
>>526
再挑戦
k=Π(pi)^(ei) (piはすべてことなる素数)としてp=p1、e=e1とおきさらに
有理数rにたいしv(r)=(pの多重度)とおく。そこで
A={n|v(n)≡0,1,2,・・・,e-1 (mod 2e)}
B={n|v(n)≡e,e+1,e+2,・・・,2e-1 (mod 2e)}
とおく。するとa,a'∈Aおよびb,b'∈Bにたいしv(a/a')=v(a)-v(a')≠e、v(b/b')=v(b)-v(b')≠e
なのでa/a'もb/b'もkにはなりえない。

530 名前:132人目の素数さん投稿日:03/12/14 02:48
x∈Nに対してx*k^n∈Nとなる最大のn∈N+{0}をf(x)とする。
(ただしkが自然数の時はx*k^nでなくてx/k^n)
A={x∈N|f(x)が偶数} B={x∈N|f(x)が奇数}でどうでしょ。

531 名前:530投稿日:03/12/14 02:49
被ったー。ごめんなさい

532 名前:132人目の素数さん投稿日:03/12/14 02:55
正解。 ちなみに、出題元はこれ。
http://www.kalva.demon.co.uk/aus-pol/ap78.html

533 名前:132人目の素数さん投稿日:03/12/14 22:26
今回発見されたフセインが本物である確率は?

534 名前:132人目の素数さん投稿日:03/12/14 22:48
>>521
x,yが整数の場合に限るとどうなるか?

535 名前:132人目の素数さん投稿日:03/12/14 22:56
素数p,q,rが次の式を満たすとき、その値を求めよ。
 p | q^r + 1
 q | r^p + 1
 r | p^q + 1


536 名前:132人目の素数さん投稿日:03/12/14 22:57
>>534
>>516って問題として成立してないんじゃないの?

537 名前:132人目の素数さん投稿日:03/12/15 00:24
とりあえず間違いでも自分が答えもってる問題以外はやめてほしい。

538 名前:132人目の素数さん投稿日:03/12/15 00:34
>>537
「難しい問題といて〜な」っていうスレを作れば、ええんちゃう?

539 名前:132人目の素数さん投稿日:03/12/15 00:39
答えを知らないことを明記しておけば、構わないと思う。

540 名前:132人目の素数さん投稿日:03/12/15 00:42
>>539
それはそうだね。しかし過去未解決問題とか自作の答えない問題かいてる香具師で
そういう気遣いする香具師ゼロ。2、3日メチャメチャ悩んで未解決問題だったときすげ―ショックなんだが。
ホントやめてほしい。

541 名前:132人目の素数さん投稿日:03/12/15 00:55
彼らを隔離するためにも、ここは一つ
「お前ら表に出ろ! 俺の作った問題を解け!」

542 名前:132人目の素数さん投稿日:03/12/15 01:42
>>536
http://www.unl.edu/amc/a-activities/a7-problems/questions/q-tse/-pdf/tse03.pdf

まだ分からんが、一応解ける問題らしいな。

543 名前:132人目の素数さん投稿日:03/12/15 01:54
>>542
こんなページがあったのか〜

544 名前:132人目の素数さん投稿日:03/12/15 02:55
>>542
>>516のってないけど?

545 名前:132人目の素数さん投稿日:03/12/15 03:18
恐らく、>>535と間違ったんだろう。
>>535なら載ってる

546 名前:132人目の素数さん投稿日:03/12/15 04:44
>>535ならでけた。
容易にすべて相異なる素数ではある。
まずどれか一つは2であることをしめす。全部奇数と仮定する。
pが最大としてよい。r|p^q+1よりp^q≡-1 (mod r)。フェルマの小定理よりp^(r-1)≡1 (mod r)。
ここで(q,r-1)=1 or q
(i)(q,r-1)=1のとき
uq+v(q-1)=1となる整数u,vをとると仮定よりuは奇数。∴p≡p^uq・pv(q-1)≡-1 (mod r)
∴r|p+1。rが奇数だから(p+1)/rは偶数。∴q^(p+1)=(q^r)^((q+1)/r)≡1 (mod p)
フェルマの小定理よりq^(p-1)≡1 (mod p)。∴q^2≡1 (mod p)。∴p|q^2-1。
∴p|q+1 or p|q-1。これはpの最大性およびすべて奇数に反する。
(ii)(q,r-1)=qのとき
r=kq+1とおけるのでr≡1 (mod q)。仮定よりr^p≡-1 (mod q)。∴2≡r^p+1≡0 (mod q)
∴q=2。これは仮定に反する。
以上でどれかひとつは2であることがいえた。
以下r=2と仮定すると与式はp|q^2+1、q|2^p+1 (p,q は奇素数)と同値。
4^p≡1 (mod q)。フェルマの小定理より4^(q-1)≡1 (mod q)。
ここで(p,q-1)=1 or p
(i)(p,q-1)=1のとき
このとき4≡1 (mod q)。∴q=3。p|q^2+1よりp=5。
(p,q,r)=(5,3,2)は与式をみたすのでこれは解の一つ。
(ii)(p,q-1)=pのとき
q=kp+1とおけるのでq≡1 (mod p)。∴2≡q^2+1≡0 (mod p)。これはpが奇素数であることに矛盾。
以上より解は(p,q,r)=(5,3,2)およびその置換にかぎる。

547 名前:132人目の素数さん投稿日:03/12/15 16:10
次の等式を、組合せ論的解釈により証明せよ。
ただし、C(n,r)はn個の中からr個を取り出す組合せの数とする。
Σ[k=0 to n]C(2n-2k,n-k)C(2k,2k) = 4^n


548 名前:132人目の素数さん投稿日:03/12/15 17:38
正直者(常に正しいことを言う人)か
または嘘つき(常に間違ったことを言う人)である人に
電話で「あなたは男ですか?」と尋ねた。
(1)「はい。私は男です」という答えが返ってきた。彼は男か?
(2)「私が男だというのは正しいね」という答えが返ってきた。彼は男か?

既出だったらスマソ

549 名前:132人目の素数さん投稿日:03/12/15 17:42
>>548
確実に武富士社員に盗聴されてるな。間違いない。

550 名前:尺取虫投稿日:03/12/15 18:30
有るところに尺取虫がいた。
そして立方体があった
尺取虫はその立方体の辺上を歩くものとする
しかしその立方体の頂点のうち一つだけ毒が塗ってある
その毒の上を尺取虫が歩くと死ぬ
尺取虫が頂点A(毒が塗っていない任意の頂点)
を出発した
一辺進むのに一年かかる。
毒に当たらない限り尺取虫が死なないとすると
この尺取虫の平均寿命は何年か

551 名前:132人目の素数さん投稿日:03/12/15 18:40
58/7年。


552 名前:尺取虫投稿日:03/12/15 18:49
何故?

553 名前:132人目の素数さん投稿日:03/12/15 19:35
>>550
毒の塗ってある頂点と、虫のいる頂点との距離によって寿命が違うと思うんだけど、
頂点Aはランダムと考えるの?
そうなら>>551と同じになるんだけど。

554 名前:132人目の素数さん投稿日:03/12/15 20:15
>>553
君、ランダムの意味を辞書で確認した方がいいと思うよ。

555 名前:132人目の素数さん投稿日:03/12/16 00:22
出発点が、毒でない7つの頂点から等確率で選ばれ、
なおかつ、各頂点での虫の進む道が、3方向から等確率で
選ばれるとすると、>>551になる。

556 名前:132人目の素数さん投稿日:03/12/17 00:11
n個の非負実数、a(1),a(2),…,a(n)は次の不等式を満たす事を示せ

( Σ[k=1,n] a(k) )( Σ[k=1,n] (a(k)^(n-1)) ) ≦ nΠ[k=1,n]a(k) + (n-1)Σ[k=1,n] (a(k)^n)


えぇっと、解答は持ってます。
とある本からの出題です。Web上では同じ問題を見つける事ができませんでした・・・

557 名前:132人目の素数さん投稿日:03/12/17 00:53
不等式ヲタがこのスレに興味を持ったようです。
|  |
|_| ゚∀゚).ハァハァ
|文| ⊂)
| ̄|u'u'
""""""""""

558 名前:132人目の素数さん投稿日:03/12/17 01:09
|  |    ∧∧
|_| __( ゚∀゚).ハァハァ
|文| (  し  )つ
| ̄| u'u-u'u'

559 名前:132人目の素数さん投稿日:03/12/17 05:28
不等式ヲタが>>556を解けなかったようです。
|  |
|_| ´Д`)ショボーン
|文| ⊂)
| ̄|u'u'
""""""""""

560 名前:132人目の素数さん投稿日:03/12/17 15:02
>>548
「私が男だというのは正しいね」ではこれ一つで真偽が定まってしまうように見える。
つまり、男である嘘つきはこう言うようには思えないし、女である嘘つきはこう答えうるように見える。
「私は男かという質問にはいと答えるだろう。」くらいの方が問題に適していると思うがどうだろう?

…これで(1)と(2)が同じ命題という釣りだったら…

561 名前:548投稿日:03/12/17 21:28
そうすね。すいません。

562 名前:132人目の素数さん投稿日:03/12/18 01:14
前回の問題がまだ解かれていないのに、新たな追加問題。

h(n) = Σ[k=1,n] (1/k)
とおくとき、n≧2の自然数nに対して

n + Σ[k=1,n-1] h(k)

を簡単にせよ。 結果にはhを用いてもよい。  答えはメール欄。


>>556のヒントいる?

563 名前:132人目の素数さん投稿日:03/12/18 02:24
かnぎnけnさmて=たからくじ
えnねmあnえ=いっぱい
のとき
らnざnわmい
はなにをあらわすか


564 名前:132人目の素数さん投稿日:03/12/18 02:40
n + Σ[k=1,n-1] h(k)
=
n
+1
+1+1/2
+1+1/2+1/3
・・・
+1+1/2+1/3+...+1/(n-1)
=
n
+
(n-1)*1+(n-2)*1/2+(n-3)*1/3+.....(n-(n-1))*1/(n-1)
=
n
+
n(1+1/2+1/3+...1/(n-1))+(-1-1-1-...-1)
=
n
+
n*h(n-1)+(-n+1)
=n*h(n)

565 名前:132人目の素数さん投稿日:03/12/18 02:59
今、ふと思ったんですけど四角形の成立条件ってどうなるんですか?

四角形PQRSの各辺の長さを
PQ=a QR=b RS=c SP=d
とするとき a,b,c,d にはどんな関係があるか?

もちろん答えはわかりません。

566 名前:132人目の素数さん投稿日:03/12/18 03:11
面白いことを考えるなぁ…
とりあえず潰れないために
a+b+c>d, b+c+d>a, c+d+a>b, d+a+b>c …(1)
であることが必要だけど、これだけではマズイね。
2辺が一直線になってしまわない条件も考えないといけない

567 名前:132人目の素数さん投稿日:03/12/18 03:13
>>565
a,b,eとc,d,eで三角形の成立条件をみたすようなeが取れること。
b,c,fとd,a,fで三角形の成立条件をみたすようなfが取れること。
…あと足りない条件てあるんだろうか?
尋ねられるととっさに答えにくいな。

568 名前:565投稿日:03/12/18 03:47
よく考えてみたら

c+d>│a−b│

この条件だけでいいんじゃないのかな?

569 名前: ◆BhMath2chk 投稿日:03/12/18 04:00
四角形の辺の長さになる。
<=>
全て正の実数でありどの数も他の三つの和より小さい。

>>568
(a,b,c,d)=(1,1,1,4)。


570 名前:132人目の素数さん投稿日:03/12/18 15:31
スコココバシッスコバドドトスコココバシッスコバドドトスコココバシッスコバドドトスコココバシッスコバドドトスコココ
スコココバシッスコバドドドンスコバンスコスコココバシッスコバドト ☆_∧_∧_∧_∧_∧_∧_
スコココバシッスコバドドト从☆`ヾ/゛/'  "\' /". ☆  |                    |
スコココバシッスコハ≡≪≡ゞシ彡 ∧_∧ 〃ミ≡从≡= < うぉぉぉぉー、
>>547>>556の解答まだぁ〜〜!!
スットコドッコイスコココ'=巛≡从ミ.(・∀・# )彡/ノ≡》〉≡ .|_  _  _ _ _ _ ___|
ドッコイショドスドスドス=!|l|》リnl⌒!I⌒I⌒I⌒Iツ从=≡|l≫,゙   ∨  ∨ ∨ ∨ ∨ ∨ ∨
スコココバシッスコバドト《l|!|!l!'~'⌒^⌒(⌒)⌒^~~~ヾ!|l!|l;"スコココバシッスコバドドドンスコバンスコスコココ
スコココバシッスコバドドl|l|(( (〇) ))(( (〇) ))|l|》;スコココバシッスコバドドドンスコバンスコスコココ
スコココバシッスコバドド`へヾ―-―    ―-― .へヾスコココバシッスコバドドドンスコバンスコスコココ
              /|\人 _.ノ _||_. /|\

571 名前:132人目の素数さん投稿日:03/12/18 15:35
>>570
あ,そーゆー問題嫌いです。

572 名前:132人目の素数さん投稿日:03/12/18 16:13
ある水槽に16種類のアメーバがいます。
1分間に1回争い、勝てば相手を取り込み2個に分裂します。
2回に1回は引き分けます。
従って1分後には8種類の1個のアメーバと4種類の2個の
アメーバになります。
さて平均して何分後にこの水槽のアメーバは1種類になるでしょうか。
但し、同一種類同士も争うものとします。

573 名前:567投稿日:03/12/18 17:48
>>565の問い
成立条件って書き方しているからこんがらがっていたけど、
a,b,c,dにどんな関係があるかというと>>566,>>569の通りで良いっぽいですね。
でこれが成立条件であるかというと>>566に書いてあるように2辺が一直線になってしまわない条件が足りないと。
なんでこんなことになるかというと、4辺の長さが定まっても四角形は一意には定まらないからなんですね。
3辺の長さが成立条件を満たして定まれば三角形は必ず一意に定まりますから。

以上、>>567でしょぼいことを書いた懺悔のまとめレポート提出。

ユークリッド(超)空間上に点A,B,Cがあり、AB=c,BC=a,CA=bに対し、
a,b,cのどの数も他の2数の和より小さい(つまり等しくならない)時、
それは三角形である。
ユークリッド(超)空間上に点P,Q,R,Sがあり、PQ=a,QR=b,RS=c,SP=dに対し、
a,b,c,dのどの数も他の3数の和より小さい(つまり等しくならない)時、
それは四角形とは限らないどころか同一平面上にあるとも限らない。
…で、そんな当たり前のことを書くなと言われそうだが、
四面体の成立条件とかあるんだろうかとふと思ったんで。
やっぱり三角形の成立条件4回当てはめるだけかなぁ?

574 名前:132人目の素数さん投稿日:03/12/18 21:49
凹んだ四角形もできてしまう罠。
凸多角形となる条件も必要なのでは?

575 名前:567投稿日:03/12/18 21:53
>>574
四角形はという言葉に凹んだ四角形も自然に含ませています。自分は。
というかこの問題(=四角形の成立要件)は辺の長さだけでは語れなくなってきたと思いまする。

576 名前:132人目の素数さん投稿日:03/12/18 21:58
そうだったのか…
凹んだのを入れるなら、四辺形と言ったほうがイクナイ?
しょうーもないことだが

ていうか、最初っから、そう断っとけ!

    _, ,_  パーン
 ( ‘д‘)
   ⊂彡☆))Д´) ←
>>575

577 名前:132人目の素数さん投稿日:03/12/18 22:01
>576
ばぁぁぁぁぁか(プッ

578 名前:567投稿日:03/12/18 22:04
>>576
殴ったね! 父さんにも(以下略

>>577
まぁ、心の奥に留めてやってあげてよぉ。

579 名前:565投稿日:03/12/18 23:09
じゃあ、n角形の場合も

n角形の辺の長さになる
     ↑↓
辺の長さが全て正の実数であり、どの数も他のn−1個の和より小さい。

でいいのかな??



580 名前:567投稿日:03/12/18 23:24
>>579
角度を適切に取れば、それが結論のような気がしますね。

581 名前:132人目の素数さん投稿日:03/12/18 23:27
>>556できた。与式は同次形なので束縛条件蚤k=1を導入して一般性をうしなわない。
S=nΠ[k=1,n]a(k) + (n-1)Σ[k=1,n] (a(k)^n)-( Σ[k=1,n] a(k) )( Σ[k=1,n] (a(k)^(n-1)) )
とおいてコンパクト集合{(ak)|ak≧0、蚤k=1}での最小値が0以上であることをしめす。
最小値がak=1/n以外の点ではありえないことを示す。ak=1/nでない点で最小値をとったと仮定する。
a1>1/n>a2、a1≧akと仮定して一般性を失わない。微少量eについて変換a1→a1-e、a2→a2+eを考える。
この変換の1次の寄与を計算する。
n((a1-e)(a2+e)Π[k≧3]ak-Πak)=n(a1-a2)(Π[k≧3]ak)e+O(e^2)
(n-1)((a1-e)^n+(a2+e)^n-a1^n-a2^n)=n(n-1)(-a1^(n-1)+a1^(n-1))+O(e^2)
(a1-e)^(n-1)+(a2+e)^(n-1)-a1^(n-1)-a2^(n-1)=(n-1)(-a1^(n-2)+a1^(n-2))+O(e^2)
だから
総変化量
=n(a1-a2)(Π[k≧3]ak)e+n(n-1)(-a1^(n-1)+a1^(n-1))+(n-1)(-a1^(n-2)+a1^(n-2))+O(e^2)
≦n(a1-a2)a1^(n-2)-(n-1)a1^(n-2)(na1-1)+(n-1)a2^(n-2)(na2-1)e+O(e^2)
≦a1^(n-2)(na1-na2-(n-1)(na1-1)+(n-1)(na2-1))e+O(e^2)
=a1^(n-2)(na1-na2-(n-1)na1+(n-1)na2)e+O(e^2)
=a1^(n-2)(-n^2+2n)(a1-a2)e+O(e^2)
ゆえこれは負の値をとりうる。これは最小値であったことに矛盾。
ゆえに最小値はak=1/nでとる。この点でS=0は容易。

582 名前:132人目の素数さん投稿日:03/12/19 00:08
マジツマンネ

583 名前:132人目の素数さん投稿日:03/12/19 01:30
20本の紐を左手で握ります。
握った手の上側から出ている20本を2本ずつペアにして結びます。
下側から出ている20本も同じように2本ずつペアにして結びます。

左手を離したときに、紐が一本大きなワッカになっている確率はどれくらいだと思う?

584 名前:サパーリわかりましぇん・・投稿日:03/12/19 02:04
文系のオイラが、1から最後まで全部眺めました。
とっても難しくて、よくわからなかったんだけど、理系の人たちが、こういった数学の
体系の美しさに惹かれていることは、なんとなく理解できました。
待ち時間の平均が35分っていう問題、解説読んでもサパーリわかりましぇん・・

しかし、みなさん、頭が良さそう。
オイラにわかる数字扱うものは、簿記だけです。

585 名前:132人目の素数さん投稿日:03/12/19 02:20
>>583
9!/19・17・15・13・11・9・7・5・3・1

自信なし…

586 名前:132人目の素数さん投稿日:03/12/19 02:21
>>583
すうおりか

587 名前:132人目の素数さん投稿日:03/12/19 02:35
>>583
これは片側を固定して考えればいいわけだよな?片側がどうなった場合でも同様になるから。
すると、もう片側でできるマッチングの数、ぶんのいち?
ってどう考えるんだっけか。

588 名前:583投稿日:03/12/19 02:40
まずは直感でどれくらいになりそうか書いてみて。
そのあと計算してみてよ。

>>586 いや、数オリではないです。

589 名前:585投稿日:03/12/19 02:47
>>588
直感だと1%くらいとオモタ
計算したら>>585(0.05%くらい)にナタ
あってる?

590 名前:132人目の素数さん投稿日:03/12/19 02:58
>>593
(10!÷2×2^10)/(20!/(2!)^10)かな?だめ?

591 名前:583投稿日:03/12/19 02:59
いやいや、意外と確率は高いよー。

592 名前:132人目の素数さん投稿日:03/12/19 03:10
なんか6点で考えてみたら見えた気がする。
18・16・〜・2/19・17・〜・1
って感じ?
電卓うちまちがってなければ、 0.2837731927515っぽい感じ

593 名前:132人目の素数さん投稿日:03/12/19 03:12
あ、ちがう気がしてきた。

594 名前:585投稿日:03/12/19 03:13
18・16・14・12・10・8・6・4・2←分子
 
19・17・15・13・11・9・7・5・3←分母

考え方かえたら答えもかわった。28%くらい??


595 名前:592=593投稿日:03/12/19 03:14
いや、やっぱいいのか。いいんだよきっと!

596 名前:592投稿日:03/12/19 03:16
漏れの場合、分子のほうは、すでにペアになってるのを一筆書きでループにできる場合を数え上げた感じでやったよ

597 名前:585投稿日:03/12/19 03:26
おれは、最初に一本選んで、その紐のもう一個の端以外をペアとして選ぶ確率が18/19
今結んだ紐のもう一個の端が、最初に選んだ紐のもう一個の端以外をペアとして選ぶ確率が16/17
以下同様に…って感じ

2人挑戦して答え一緒だからきっとあってるはず…と思うがどうなの>>583


598 名前:583投稿日:03/12/19 03:37
ちょっと考えていた値と微妙に違ってたから検算してた。
答えはそれで合ってます。

こっちで用意していた答えは、2n本の紐のケースで
(2n-1)!/{(2n-1)!!}^2 = 2^(2n-1)/(n*C(2n,n)) でした。

599 名前:132人目の素数さん投稿日:03/12/19 03:40
20%くらいじゃないの?

600 名前:583投稿日:03/12/19 03:42
これ、パソコンで計算させたら、0.199くらいの値が出てきてたので
「20%くらいでしょ」という解答があるかと密かに期待して
出題したんだけど、どうも計算に誤差がたくさん入ってしまってたみたいです。

ちなみに、プログラム組んでモンテカルロをしてみたら
10000回やって2920回ワッカになりました。

601 名前:583投稿日:03/12/19 03:42
>>599
ありがとう(TT)

602 名前:592投稿日:03/12/19 03:44
正解ワショーイ
ネタ的には不発だったかもしれんが漏れは面白かったよ

603 名前:585投稿日:03/12/19 03:52
意外と確率高いんだなァ

604 名前:132人目の素数さん投稿日:03/12/19 04:02
確率が20%くらいになるのは、40本のときみたい。
そのときの確率が、0.19940....。

多分問題出すときに自分で導いた式が2n本の時の
答えであることを忘れて、n=20で計算してしまったんだと思う。
と自己弁護w

>>592 ありがとね。

605 名前:583投稿日:03/12/19 04:03
604=583ね。名前入れるの忘れた。

606 名前:132人目の素数さん投稿日:03/12/20 18:56
問題です。

麻雀の、天和が起こる確率を求めよ。

607 名前:132人目の素数さん投稿日:03/12/20 20:59
>>606
ただし、円周率を3とする。
が抜けてますよ。

608 名前:132人目の素数さん投稿日:03/12/21 01:20
おしえてage

609 名前:132人目の素数さん投稿日:03/12/21 01:33
F(x)=x^3+x+1とおく。実数xにたいし集合P(x)を
P(x)={p:素数|F(x)≡(x-a)(x-b)(x-c) (mod p) ∃a,b,c∈Z}
とおく。lim[n→∞]P(x)/(x以下の素数の数)をもとめよ。

610 名前:132人目の素数さん投稿日:03/12/21 02:01
訂正
F(x)=x^3+x+1とおく。実数xにたいし集合P(x)を
P(x)={p:素数|p≦x、F(t)≡(t-a)(t-b)(t-c) (mod p) ∃a,b,c∈Z}
とおく。lim[n→∞]P(x)/(x以下の素数の数)をもとめよ。


611 名前:132人目の素数さん投稿日:03/12/21 02:01
http://www.google.co.jp/search?q=%E5%A4%A9%E5%92%8C%E3%80%80%E7%A2%BA%E7%8E%87&ie=UTF-8&oe=UTF-8&hl=ja&lr=

612 名前:132人目の素数さん投稿日:03/12/21 02:15
まだ訂正しきれてない。


613 名前:132人目の素数さん投稿日:03/12/21 02:33
これで桶?
F(x)=x^3+x+1とおく。実数xにたいし集合P(x)を
P(x)={p:素数|p≦x、F(t)≡(t-a)(t-b)(t-c) (mod p) ∃a,b,c∈Z}
とおく。lim[x→∞]P(x)/(x以下の素数の数)をもとめよ。


614 名前:132人目の素数さん投稿日:03/12/21 17:03
>>613
a,b,cはmodpで異なるっていう条件はいらないよね。

615 名前:132人目の素数さん投稿日:03/12/23 23:11
保守。

616 名前:132人目の素数さん投稿日:03/12/23 23:22
>>614
いらない。

617 名前:132人目の素数さん投稿日:03/12/24 03:36
三角形ABCがある。三辺の長さをa,b,cとしてSを面積とする。
a,b,cが自然数、かつ、Sが平方数であるとき、三角形ABCを求めよ。

存在しないと思うなら、理由をつけて示せ

618 名前:132人目の素数さん投稿日:03/12/24 09:20
意外とむずい。答え持ってるこれ?

619 名前:132人目の素数さん投稿日:03/12/24 11:43
>>617
三辺が9・10・17のやつと3・25・26のやつが面積36になった。

620 名前:132人目の素数さん投稿日:03/12/24 12:26
>>617 の問 2 です。

また、Sが一辺の平方数であるとき、三角形ABCを求めよ。

存在しないと思うなら、理由をつけて示せ


621 名前:132人目の素数さん投稿日:03/12/24 12:31
さらに問 3 もあったり…。

一般に、Sが (n/2)a^2 と表せるとき、n の条件を求めよ。
ただし、n は自然数

622 名前:132人目の素数さん投稿日:03/12/24 13:43
>>613
2/3

623 名前:132人目の素数さん投稿日:03/12/24 22:39
>>622
ちがう・・・ハズだが・・・なんでそうなった?

624 名前:132人目の素数さん投稿日:03/12/24 22:40
>>620-621
ヒントきぼんage

625 名前:132人目の素数さん投稿日:03/12/25 03:48
nを正の整数として、平面上にn個のベクトルがある。
今n個のベクトルを
↑A(1) , ↑A(2) , …, ↑A(n)
として、
1 = Σ[k=1,n] | ↑A(k) |
が成立している。

この時、n個のベクトルからなる集合をを三つの部分集合に分割する事ができ、(空集合もOK)
それら三つの集合X,Y,Zは次の条件を満たすようにできる事を示せ。
1) X∪Y∪Z={ ↑A(1) , ↑A(2) , …, ↑A(n) }
2) | Σ[ ↑A(x)∈X ] ↑A(x) | + | Σ[ ↑A(y)∈Y ] ↑A(y) | + | Σ[ ↑A(z)∈Z ] ↑A(xz) | ≧ (3√3)/(2π)

626 名前:132人目の素数さん投稿日:03/12/25 03:49
>>625
おっと、忘れた
3) X∩Y=空集合、 Y∩Z=空集合、 Z∩X=空集合

627 名前:132人目の素数さん投稿日:03/12/25 19:22
>>620-621
ヒントきぼんage Secound Edition


628 名前:132人目の素数さん投稿日:03/12/25 19:24
あげわすれた・・・

629 名前:132人目の素数さん投稿日:03/12/25 21:51
ヘロンの公式かな

630 名前:132人目の素数さん投稿日:03/12/25 21:57
>>629
出題者?

631 名前:629投稿日:03/12/25 22:01
>>630
違。
思いついただけの横レス
でも出来ないぽ

632 名前:132人目の素数さん投稿日:03/12/25 22:28
というか>>620-621自体思いつきでつくった問題くさい。

633 名前:132人目の素数さん投稿日:03/12/26 00:34
>>620

これは整数論の中では知られた問題で, 出来ないことが証明されていたはず。
だから, 思いつきとは限らないであろう…。

634 名前:132人目の素数さん投稿日:03/12/26 00:36
>>633
>これは整数論の中では知られた問題で, 出来ないことが証明されていたはず。
 
まじ?ソースキボン
 
>だから, 思いつきとは限らないであろう…。
 
どうかな?今まで出題者のカキコ一回もないし。絶対思いつきだって。

635 名前:132人目の素数さん投稿日:03/12/26 00:41
三角形の内接円半径r,外接円半径Rとする
2r≦Rを証明せよ

636 名前:132人目の素数さん投稿日:03/12/26 02:24
>>635
それこそヘロンの公式だろ。

637 名前:132人目の素数さん投稿日:03/12/26 04:30
>>625
Give me a Hint, Please!! AGE

638 名前:132人目の素数さん投稿日:03/12/26 23:17
>>625
ヒントくれくれくれくれくれくれくれくれ

639 名前:132人目の素数さん投稿日:03/12/26 23:24
あぁ、スマン>>638
http://www.unl.edu/amc/a-activities/a4-for-students/problemtext/mc97-98-01feb.pdf
これの223ページ24番を写しただけだ答えは持ってない。っていうことで、自分で考えてけろ

640 名前:132人目の素数さん投稿日:03/12/26 23:29
げげ――――――――――――――――――――――――――
なに―――――――――――――――――――――――――――
くそ―――――――――――――――――――――――――――
年末いそがしいのに――――――――――――――――――――

641 名前:132人目の素数さん投稿日:03/12/26 23:39
>>640
すまん。ただ、一ヶ月後にこれの解答が載っている本が届く。
なので、その頃まで待ってもらえれば答えが出せる。


つーか、解答の用意されているはずの問題なんだから答えられると思うけど・・・誰かが。

642 名前:132人目の素数さん投稿日:03/12/26 23:43
期限は一ヶ月ときまりますた。さぁみんなでかんがえよぉ!

643 名前:132人目の素数さん投稿日:03/12/27 00:08
>>635
三角形の3つの中点を通る円(9点円)の半径はR/2。
内接円は三角形の三辺すべてと共有点をもつ円の中で
最も小さな円であるからr≦R/2。
∴2r≦R。

644 名前:132人目の素数さん投稿日:03/12/27 00:35
論理問題だと思うんで
「ビル・ゲイツの面接試験」でぐぐった問題
http://www.atc.ne.jp/seikindo/html/bilgeituno.htm
http://www.isis.ne.jp/mnn/senya/senya0888.html
http://d.hatena.ne.jp/knack/asin/4791760468

1: 太陽は必ず東から出ますか。
2: マンホ−ルの蓋はなぜ丸いのでしょう。
3: マイナス2進法で数を数えなさい。
4: 時計の針は1日に何回重なるでしょう。
5: 富士山を動かすにはどれだけ時間がかかるでしょう。
6: ミシシッピ川の1時間の最大流水量は。
7: アイスホッケーリンクの総重量は?
8: 世界にピアノの調律師は何人いますか。

クイズ板からの転載で
9:
5人の海賊A,B,C,D,Eが100個の宝石を手に入れた。その宝石の分け方の問題。
まず順番を決め、最初の1人目が分配方法を提案し、多数決を採る。
賛成多数の場合はその方法を採用して終了。
賛成人数が反対人数以下ならば、その1人目を殺して次の2人目が同様に提案し、多数決を採る。
以下同様にして、決まらない場合は最後の1人になるまでこれを繰り返すものとする。
さて、1人目はどのような分配方法を提案すればより多くの宝石を手に入れられるだろうか。
もちろん死なずに。

645 名前:644投稿日:03/12/27 00:41
1〜5、9は最低限考える取っ掛かりはあるんだが
6,7,8は取っ掛かりすらない・・

646 名前:132人目の素数さん投稿日:03/12/27 00:54
7 ホッケー場の広さ×氷の厚さ×氷の比重?

647 名前:644投稿日:03/12/27 02:13
>>646
そうなんだよね。
例えばホッケー場のサイズをたまたま知ってたやつが
「広さはx(m^2)で厚さがy(m)であの氷の比重がz(kg/m^3)なので答えはxyz(kg)です」
なんて答えを出すことを求めてるかといえば、
なんか違う気もするんだよなぁ。
俺にとっては(特に9なんかは)久々の論理系の良問だったんで
きっとなにか、こう、取っ掛かりがあるんじゃないかと・・

自分なりにひねり出した解
5:

□□□□●●■■□□□□□□
□□□●●■■■■□□□□□
□□●●■■■■■■□□□□
□●●■■■■■■■■□□□

        ↓

□□□□□□■■●●□□□□
□□□□□■■■■●●□□□
□□□□■■■■■■●●□□
□□□■■■■■■■■●●□

これを「動いた」と定義してしまう。
この「●」を極限まで小さくすると
多くとも富士山の外周を半周する時間ですむ。
さらに回転角も極限まで小さく取る事を考えると以下のようになる。

「富士山の最も外側にある砂粒のうちの一つを、
 外周にそってほんの少し移動させるために要する時間」

648 名前:132人目の素数さん投稿日:03/12/27 02:20
f,T;[0,1]→[0,1]、f(x):=1/2-|x-1/2|、T(x):=2^(-n)f(2^n*x-[2^n*x])
(nは0から∞)とする、Tは無限個の点で値2/3を取ることを示せ。特に
Tが2/3の値をとる点を無限個与えよ。それが出来たら、Tの変域の制限を
変更したときにどうなるかを考えよ。友人への年賀状に書いた問題なので
(即興で作った)、瞬殺だとは思いますが。あと、下らん函数だ、などと
言うと罰が当たりますよ。

649 名前:132人目の素数さん投稿日:03/12/27 02:40
2.蓋が落ちないように。別に円でなくても構わないが(e.g. 正三角形+Π/3の
弧3つの図形、なんていうんだっけ)円が一番素直な形だから。

4.22回?

6.から8.は、見積もりの問題だから実験屋の人とか、社会科学の人は
得意かも。

9.まず、残り二人になった場合、提案者になってしまえば、即、死決定だと
思うんだが... 誰からも不公平だという文句の出ないような公平な分売
の方法なら分かるんだが、この問題は少し違うみたい...

650 名前:648投稿日:03/12/27 02:44
上の問題はもしかすると、一寸難しいかも。とくに、Tを初めて見た人は。

aとbが無理数でa^bが有理数になるようなa,bが存在することを
高校数学における対数以上の知識を用いないで(対数も使っちゃだめ)
示せ。まぁ、対数を使った証明が分からん人は使った証明を考えても
いいけど。

651 名前:648投稿日:03/12/27 02:52
上の問題とは648の問題、という意味です。

次の証明の問題点を述べよ。(数学科のプロの人は瞬殺の筈)
「円に内接する面積最大の三角形は正三角形である」
 一辺を固定して考えれば残りの2辺の長さが等しいときに高さが最大と
なる。同様にして、任意の2辺の長さが等しくなるとき、つまり、正三角形
のときに面積は最大となる。

652 名前:132人目の素数さん投稿日:03/12/27 03:02
>>651
1辺を固定して残りの2辺を等しくした状態から、その2辺が同じ長さという性質を満たしたまま
初めに固定した1辺も上のどちらかの辺(即ち2辺)と長さが等しい状態に動かせるような推論が行われているのがマズイ
でいいのか?
何かうまく表現出来ん…。


653 名前:648投稿日:03/12/27 03:12
>>652
もう少し考えましょう。ヒント書き忘れました。
1.Weierstrass、ディリクレの原理
2.次の証明から考えてみては
「1は最大の自然数である」
なぜなら、nが最大の自然数であるとするとn^2はn以下である。
即ちn^2≦n、i.e.n≦1。nは自然数であったからn=1 q.e.d
651はこれと同じ轍を踏んでいます。

654 名前:132人目の素数さん投稿日:03/12/27 03:15
>>651
面積最大の三角形が存在することを言わなきゃ駄目ってことかな?

3辺をa,b,cとする。
a≠bのときその三角形は面積最大とはなり得ない。
なぜならcを固定して(略
よって面積最大となるためにはa=bであることが必要条件である。
同様にしてb=cが必要条件であることも言えるので、
結局、面積最大となるためには3辺の長さが等しいことが必要条件である。


取りあえずこれは合ってるよね?(あくまで必要条件で攻めてみた)

655 名前:132人目の素数さん投稿日:03/12/27 03:42
馬鹿ばっか

656 名前:132人目の素数さん投稿日:03/12/27 03:56
>>655
その言葉のあとに

見事な証明つけてくれたらなぁ・・・。

657 名前:132人目の素数さん投稿日:03/12/27 04:05
>>655
君のことだな

658 名前:132人目の素数さん投稿日:03/12/27 08:12
>>655
そんな事言えるのなら頼むからこのスレの無数の未解決問題をどうにかしてくれ…

659 名前:132人目の素数さん投稿日:03/12/27 15:32
各辺の長さ1の正四面体1個と
各辺の長さ1のピラミッド (正四角錐---底面は正方形、側面は正三角形)1個を、
正三角形の面で頂点と頂点があうように貼り合わせます。
できる立体は何面体か?

660 名前:132人目の素数さん投稿日:03/12/27 16:42
正四面体の各辺の中点を結ぶと一辺が1/2になった正四面体が4つとれる。
この4つを取り外すと正八面体が出来る。これを2つに分割した奴の1つと
それにくっつく1/2正四面体1つを合わせた物が>>659の物体になる。

作り方から面は6つ。

661 名前:132人目の素数さん投稿日:03/12/27 16:43
作り方から面は6つ。→作り方から面は5つ。

662 名前:132人目の素数さん投稿日:03/12/27 17:03
正四面体の面は4つ。正四角錐の面は5つ。
このふたつの立体が1つの面を共有する。
共有された面は外から見えなくなるから
全部で面は7つ。

663 名前:659投稿日:03/12/27 17:10
5つで正解。
>>662は誤答。

この問題、どこかのテストに出て、
その模範解答が間違っていたといういわくつき問題。

664 名前:132人目の素数さん投稿日:03/12/28 00:00
1から自然数nまでの和をS(n)、自然数nまでの素数の和をP(n)とおく。
P(n)/S(n)の極限を計算してください。

665 名前:132人目の素数さん投稿日:03/12/28 00:40
0

666 名前:甲陽高1 ◆uqmQ5k/uJs 投稿日:03/12/28 00:55
>>664S(n)>P(n)なんやからnを無限に飛ばしたら0になるにきまっとうやろ

667 名前:132人目の素数さん投稿日:03/12/28 00:58
666 名前:甲陽高1 ◆uqmQ5k/uJs :03/12/28 00:55
>>664S(n)>P(n)なんやからnを無限に飛ばしたら0になるにきまっとうやろ


668 名前:132人目の素数さん投稿日:03/12/28 02:05
>>666
ヴぁか丸出しじゃんか…

669 名前:132人目の素数さん投稿日:03/12/28 03:16
>>666
【コヨタン萌え】天才ほいほい2【怒濤(どす)】
http://science2.2ch.net/test/read.cgi/math/1072095905/l50
数学の範囲に拘り数学の真の面白みを知らぬ愚か者よ、帰せよ。

670 名前:132人目の素数さん投稿日:03/12/28 03:40
P(n) := { nまでの素数全部 } としたとき、
P(n)の要素数は、素数定理より、n→∞では
n/(log n) で近似できる。

従ってΣ[k∈P(n)] k ≦ n^2/(log n) と評価でき、

(Σ[k∈P(n)] k ) / Σk ≦ 2n^2/(n(n+1)(log n)) → 0

酔ってるので尻滅裂かも‥‥

671 名前:132人目の素数さん投稿日:03/12/28 12:06
>644.9
三人目、五人目には分け前0で
残り3人で3等分

672 名前:132人目の素数さん投稿日:03/12/28 13:09
>>670であってるような・・・

673 名前:132人目の素数さん投稿日:03/12/28 13:18
>>671
100個の宝石を、どうやって3等分すればよいですか?

674 名前:132人目の素数さん投稿日:03/12/28 16:26
重さで。
っていうか、その答え間違ってるよ。

675 名前:132人目の素数さん投稿日:03/12/28 21:21
z

676 名前:132人目の素数さん投稿日:03/12/29 02:17
>数学的帰納法は普通は
>(I)n=1のときを証明
>(II)n=kのとき成り立つと仮定してn=k+1のときの成立を証明
>という手順を踏みます。
>
>だけどこんな問題を出されたのです。
>
>「数学的帰納法において、kを使わずにnのまま証明して、証明したことになるか否かを述べよ」
>つまり
>(I)n=1のときを証明
>(II)nのとき成り立つと仮定して、n+1のときの成立を証明
>という手順で証明してもいいのかどうか。ということです。

ちょいとおもしろいと思った。

677 名前:644投稿日:03/12/29 08:01
>>671
その場合、BCEが反対するためにAは死ぬ事になる。
Bは自分が提案する番になると
その提案以上の宝石をせしめる事が出来るからだ。

678 名前:132人目の素数さん投稿日:03/12/29 11:01
2人以下の時、Eが必ず百個取れるとすると、
3人になった時に、Cが百個取るとしてもDは受け入れざるを得ない。
4人の時、Bの提案に対しCは必ず反対するだろうが、DとEは
一個でも分け前があれば賛成すべきだ。
5人の時、Aの提案に対しBは反対するが、C,D,Eは
一個でも分け前があれば賛成するだろう。
とすると、Aは98個を自分でとり、一個ずつCDEの適当な二人に
与えるのが正解では無いだろうか?

679 名前:132人目の素数さん投稿日:03/12/29 11:19
ある4けたの自然数をまん中で2けたの自然数2つに分けるとその和は113で、上2けたをそのまま下2けたに移して4けたの自然数を作ると、もとより4851だけ小さい数になりました。
もとの4けたのある自然数は何でしょう?


●▲■◎

●▲+■◎=113

●▲■◎−■◎●▲=4851

680 名前:132人目の素数さん投稿日:03/12/29 11:35
>>678
最後がちょっと違う。Aが97個、Cが1個、DかEに2個、だと思うよ。
次以降に期待できる額が同じなら、殺してしまうという前提においてだけど。

681 名前:644投稿日:03/12/29 12:31
>>680
>次以降に期待できる額が同じなら、殺してしまうという前提

ちょっと強すぎる前提では。
1、死にたく無い
2、1に反しない限り多くの宝石が欲しい
という定義だけでは、彼らが殺したがりなのか生かしたがりなのかまでは確定しないと思う。
そしてどちらの立場を選択するかによって答えが変わってしまう。
(個人的にはここがこの問題のキモだと思ってる)

定義から以下を導けば十分だと思う。
「次に期待できる額が同じならどちらかを確実に選ぶ事はしない。(不確定状態)
ただし彼らは絶対に死にたく無いので、生死不確定な状態よりも確実な生を選ぶ」

結論は同じなんだけどね。

682 名前:132人目の素数さん投稿日:03/12/29 13:45
>>676を誰か教えて。
俺は問題ないと思うんだけど。

683 名前:132人目の素数さん投稿日:03/12/29 14:20
>>681
確定しないからこう前提をおくのが合理的ってことだよ。

残りの人間は、次の提案者が馬鹿であることにして期待して殺してしまうのが
いいし、提案者は残りの人間がそう考える可能性のあることを考慮して、賛成
を確定させるために、次の期待額以上を提案するほうが安全になる。

一番死の危険にさらされているのは提案者だから、「確実な生を選ぶ」のは提案
者にあてはめるべきだよ。





684 名前:132人目の素数さん投稿日:03/12/29 14:26
なるほど

685 名前:132人目の素数さん投稿日:03/12/29 14:42
>>682
普通は問題ないよ。
本当は、「nで成り立つ」というのは「あるnで成り立っている」とするのが正しい。
文脈からは明らかだからそんなにこだわらんでもいいけど。

686 名前:132人目の素数さん投稿日:03/12/29 15:26
>>685
>「nで成り立つ」というのは「あるnで成り立っている」とするのが正しい。

ん、なるほど。

687 名前:132人目の素数さん投稿日:03/12/29 15:27
切符でよくやる、並び替えと括弧と加減乗除だけで10を作るやつ。

1 1 5 8

さあどうぞ。

688 名前:132人目の素数さん投稿日:03/12/29 15:40
8/(1−1/5)=10。


689 名前:132人目の素数さん投稿日:03/12/29 15:46
>>687
答えは>>688のみ。

690 名前:132人目の素数さん投稿日:03/12/29 15:50
さすが数学板。瞬殺か。

太郎君のお母さんには3人の子どもがいました
(子どもたち)
@ D I


それぞれ名前が
@=一郎君

D=五郎君  と言います

さてI君の名前は何でしょう?

691 名前:132人目の素数さん投稿日:03/12/29 15:52
太郎君

692 名前:132人目の素数さん投稿日:03/12/29 15:55
I=野口五郎君


693 名前:644投稿日:03/12/29 17:59
>>683
確定しないから合理的な前提を置くべきなのは同意なんだが、
この問題が成り立つために以下を前提とする必要があると思うんだ。

・彼らは常に最善策を選択する
・そしておたがいその事を知っている

もしこれが成り立たないとしたら
(他人が非最善策をとる可能性を考えなければならないなら)
例えば>>678のような推論は不可能になるんじゃないか?
それか以下のような強引な推論になってしまう。

〜〜〜
A提案が「98-0-1-0-1」であった場合、Eの立場だと
「AかBのどちらかの提案には必ず賛成するつもりだが
Bは馬鹿かも知れないから俺に2枚以上という提案をするかも知れない。
でも俺に0枚という提案をするほどBは馬鹿じゃない。だからAは殺した方がいいな」
だからAとしてはEに2枚という提案が必要である。
〜〜〜

694 名前:132人目の素数さん投稿日:03/12/29 18:26
>>693
そだね。ただ、Eにとって、
Aの「98-0-1-0-1」という提案をAを殺さずに受け入れるのと、
Aを殺して、Bの「98-0-1-1」という提案を受け入れるの
はどちらも最善になる。

AはEが最善策をとるのはしっているが、どちらを選ぶかは分からない。
という状況では、Aにとっては死ぬ確率をゼロにするためにEに2枚与える
のが最善だ、とならない?

提案者にとっては、残りの者の最善策がどちらでもいい場合は、自分を殺す
方を選択するはずだ、と考えて行動するのが死ぬ確率を減らす上で最善とな
る、ということ。

695 名前:132人目の素数さん投稿日:03/12/29 18:53
>>625
できたぜ。
j=1,2,3にたいし↑B(j)(x)=(cos(x+2jπ/3),sin(x+2jπ/3))とおく。
i:1〜nにたいして関数fi(x)をfi(x)=max{↑A(i)・↑B(j)(x) | j=1,2,3}とおく。
このとき∫[0,2π/3]fi(x)dx=|↑A(i)|∫[0,2π/3]cos(x)dx=(√3)|↑A(i)|
よってf(x)=納i]fi(x)とおくとき∫[0,2π/3]f(x)dx=√3。
よって平均値の定理から0<a<2π/3をf(a)=(3√3)/(2π)となるようにとれる。
X'(j)={i | fi(t)=↑A(i)・↑B(j)(a)}とおき、X(j)=X'(j)\(∪[k<j]X'(k))とおく。
さらに↑C(j)=納i∈X(j)]↑A(i)、↑C(j)・B(j)(a)=m(j)とおく。θjをC(j)とB(j)(a)のなす角とする。
納j]m(j)=f(t)=(3√3)/(2π)であり|↑C(j)|≧|↑C(j)||B(j)(a)||cosθj|=|m(j)|
から納j]|↑C(j)|≧納j]|↑C(j)||B(j)(a)||cosθj|=納j]|m(j)|≧|納j]m(j)=f(t)|=(3√3)/(2π)
よってX=X(1)、Y=X(2)、Z=X(3)、とおけばよい。

696 名前:132人目の素数さん投稿日:03/12/29 20:12
>>687
ここの下の方に 1, 1, 5, 8, 10 と入れると結果が表示される。
http://www.amy.hi-ho.ne.jp/odasan/com/programs/number4/number4.html

697 名前:644投稿日:03/12/29 21:11
>>694
>AはEが最善策をとるのはしっているが、どちらを選ぶかは分からない。
>という状況では、Aにとっては死ぬ確率をゼロにするためにEに2枚与える
>のが最善だ

全く異論なし。なので言い方の差だけかも。

確実な生>生死不確定>確実な死

の大きな方を常に選択する、ということだもんね。

さて、6人目を考えると新たなジレンマが発生する。
参加者をZABCDEとするとして、
ZがDに1枚を提案したらDは賛成するか?
(Z案での確実な1枚)と(A案での不確実な2枚)はどちらかが重い?等しい?
うぬぬ・・

698 名前:132人目の素数さん投稿日:03/12/29 21:18
A案では0かも知れない。C,Eに配分する場合。

699 名前:132人目の素数さん投稿日:03/12/29 21:33
>>679
8132


700 名前:132人目の素数さん投稿日:03/12/29 21:38
>>697
考え方は同じだよ。「確実な生>生死不確定>確実な死」に則る。
そういう判断をZができない以上は、確実にZがD(orE)から賛成を得るには3個が必要になる。
同様にCは2枚、Bは1枚が必要となる。5人中3人の賛成を得なければいけないから、
[94,0,1,2,0,3] or [94,0,1,2,3,0]が正解。

701 名前:132人目の素数さん投稿日:03/12/29 21:45
でも[94,0,1,2,0,2] or [94,0,1,2,2,0]でもいいかもしれないな…。
ここは難しい。

702 名前:132人目の素数さん投稿日:03/12/29 22:05
実際にやったら絶対Aは頃される。


703 名前:132人目の素数さん投稿日:03/12/29 22:24
>>702
B,C,D,Eが馬鹿なのか・・・

704 名前:132人目の素数さん投稿日:03/12/29 22:57
>>702 ワロタ
まぁ、nが小さいときから帰納的(因みにプロの言葉では再帰的)に
解いていくべきなのは明らか。それから、当面皆は論理的にものを考える
ことにしよう。まぁ、A〜Eはお互いを良く知ってるかもしれないから、
例えばDのみが論理的にものを考えない場合、Bがnon-logicalで、Dが良く
分からない場合、なんて場合分けしてもいいが、複雑すぎる。
そこで、次の概念を導入する。Xが論理的海賊(Logical Pirates LP)
であるとは海賊集合の元Xが論理的推論者であることと定義する。(w
それから、次回以降の期待額が同じ場合に、提案者を殺すのかそうじゃない
のかもはっきりさせないと。まぁ、これは場合分けして解くことにしても
いいが、非本質的なことに労を費やすことになる。

705 名前:132人目の素数さん投稿日:03/12/29 23:10
651の略解です。
>>次の証明の問題点を述べよ。
>>「円に内接する面積最大の三角形は正三角形である」
>>一辺を固定して(ry
最大値の存在を仮定しています。若しあなたが高校生なら、中心角を
変数にとって二等辺三角形の面積のグラフをかいて、グラフより最大値の
存在は明らか、と言えば有能な教師からは褒めて貰えるでしょう。
理系大学生なら閉区間で(ryを使ってください。

706 名前:132人目の素数さん投稿日:03/12/29 23:13
>>620, >>621
は n についての条件をまとめて解くしかないかも…。

>>634
整数論の中では知られた事実と言うことは聞いた話で, ソースは探すのが難しい…。
海外の論文(文献)にあるだろうけど, 自分の力では探すことが出来ませんでした。


この問題は『楕円曲線』の知識を使って解くことが出来ます。
ヒントになるかな?

707 名前:644投稿日:03/12/29 23:19
>>701
あら。ここは全く反対の考えだ。Dにとって確実により多くの宝石を得るには
目の前の1枚>次の不確定な2枚
かと思ったけど・・(この場合[97-0-1-0-1-1]になる)

この不等号を不確定とすると
[95-0-1-*-*-*] (*のうち2人に2枚)
となるのかな?
確定2枚>不確定2枚
は確実に言えそうだし。

まぁ参加人数は5人までにするのが無難かな。
最善策を定義しきれない。

# どの考えでも言えることだけど、(不確定枚数は期待値を採用するとかね)
# 参加人数がどれだけ増えても提案者以外の獲得枚数は2枚以下になる模様。

# そして相変わらずピアノ調律師の人数は数えられない・・・

708 名前:132人目の素数さん投稿日:03/12/29 23:23
>706
ただの直角三角形とは違うのん?
漏れ題意を違えてる?

709 名前:132人目の素数さん投稿日:03/12/29 23:42
>>707
704さんの言ってる、
「次回以降の期待額が同じ場合に、提案者を殺すのかそうじゃない
のかもはっきりさせないと。」
の部分だよ。

期待値、という考えなら、もしZがEが考慮する不確実さの事前分布を0も2も1/2
と考えた場合、期待値は1となって同じになるからね。

ま、この辺をはっきり定義するべきだ、ってことだね。

>>702
そもそも順番を決めるために一悶着あるはずだしねw
合理的ならこんな分け方に同意するはずもないし。
なんて言ったら問題の意味がないけどw

710 名前:132人目の素数さん投稿日:03/12/29 23:57
>>708
直角三角形とは違います。
直角三角形は今回の場合の特殊な場合ですね。

ちなみに 直角三角形の方についての証明は下記 URL を参照してください。

http://www1.ocn.ne.jp/~yoshiiz/pdf/area_of_right_triangle.pdf

海外のサイトの証明もありますが, URL を忘れてしまいました。
スミマセン。

711 名前:132人目の素数さん投稿日:03/12/30 00:37
>>706 
出題者なの?
 
>この問題は『楕円曲線』の知識を使って解くことが出来ます。
>ヒントになるかな?
 
この問題ってどれよ>>620のこと?>>621のこと?
だいたい>>621みたいな「条件をもとめよ」なんて数学の問題になってると思えない。
究極の条件として

 (ある3辺が整数の三角形が存在してその面積をSとするとき)
 Sが (n/2)a^2 と表せる必要十分条件は16(n/2)^2・a^4=(a+b+c)(-a+b+c)(a-b+c)(a+b-c)が
 整数解a,b,cをもつことである。
 
だってまちがいじゃないんだから。何をもとめてるのかさっぱりわからん。
思いつきで作った問題だって。

712 名前:132人目の素数さん投稿日:03/12/30 00:40
>>621
Sが平方数だからnは平方数の二倍。


713 名前:132人目の素数さん投稿日:03/12/30 00:54
はぁ〜。
じゃ〜 もう模範解答をもう載せておきますね。

△ABCの3辺の長さa,b,cを有理数、底辺aをn、高さhを1とする。
AからBCに下した垂線の足をHとし, BH=u, HC=vとする。
ただし、簡単のため、
   ∠ABC > π/2の場合は、BH=-u (u<0)
   ∠ACB > π/2の場合は、HC=-v (v<0)
とする。このとき、
u+v=n
   b^2=u^2+1
c^2=v^2+1
である。
x=u-v
y=4bc
とすると、
2u=n+x
2v=n-x
より、
   y^2=16b^2c^2
=(4u^2+4)(4v^2+4)
=((n+x)^2+4)((n-x)^2+4)
=x^4-2(n^2-4)x^2+(n^2+4)^2
を得る。点(x,y)は曲線
C_n:y^2=x^4-2(n^2-4)x^2+(n^2+4)^2
の自明でない(y!=0である)有理点である。


714 名前:132人目の素数さん投稿日:03/12/30 00:55
曲線C_nは双有理変換
x=(n^2+4)(X+4)/Y,
y=(n^2+4){2X^3+(n^2+20)X^2+8(n^2+8)X+16(n^2+4)-Y^2}/Y^2 ;

X=(n^2+4)(y+n^2+4-x^2)/{2x^2},
Y=(n^2+4){(n^2+4)(y^2+n^2+4)-(n^2-4)x^2}/{2x^2} ;
により、楕円曲線
E_n: Y^2=X^3+(n^2+8)X^2+4(n^2+4)X
に有理同型である。

n>0なので、E_nのねじれ点群E_n(Q)_{tors}は、
Z/2ZxZ/2Z={(0,0),(-4,0),(-n^2-4,0),O}
である。

n=5のとき、E_5: Y^2=X^3+33X^2+116X
のMordell-Weil群E_5(Q)は、rank=1で、その基底は(-9,30)である。
E_5の有理点からC_5の有理点を計算して、題意を満たす(n,b,c)を探すと、
  (5, 241/120, 409/120),
(5, 16354950889/2152746960, 5858756089/2152746960),
(5, 15224080676149012091689/13756901884302741868440, 63765406834896547107361/13756901884302741868440),
(5, 24218187327122647056442140614568372561841/4468251181812602388323869082371370448480,                   4701093187965500335736065885533838399441/4468251181812602388323869082371370448480),
(5,188505117184298045766443476808536487676309408915680510143752001/155003203560506231309765165551107549384742417766438341246176600,

895804429223810234900835181391095219906663545142679754041146249/155003203560506231309765165551107549384742417766438341246176600),
.....


715 名前:132人目の素数さん投稿日:03/12/30 00:55
のようになる。これらと相似で3辺の長さが整数になる三角形(a,b,c)を計算す
ると、
(600, 241, 409),
(10763734800, 16354950889, 5858756089),
(68784509421513709342200, 15224080676149012091689, 63765406834896547107361),
(22341255909063011941619345411856852242400, 24218187327122647056442140614568372561841,  4701093187965500335736065885533838399441),
(775016017802531156548825827755537746923712088832191706230883000, 188505117184298045766443476808536487676309408915680510143752001,
   895804429223810234900835181391095219906663545142679754041146249),
....
を得る。

参考文献
[1]Allan J.MacLeod, "A simple practical heigher descent for large height
rational points on certain elliptic curves", July 15, 2002.


716 名前:132人目の素数さん投稿日:03/12/30 00:55
>>695 乙。
正解だな。感動したよ。

717 名前:132人目の素数さん投稿日:03/12/30 00:56
n=1,...,30について、3辺の長さが整数で、底辺の長さが高さのn倍に等しい三角形をいくつか計算してみました。

結果は次のようになりました。
n=1,2,3,4,7,10,11,12,16,24,25,27,28,30の場合は、解なし。
n=5,6,8,9,13,14,15,17,18,19,20,21,22,23,26,29の場合は、解あり。

n=6の場合は、最小解[a,b,c]が小さいので、小さい解をもつ問題を出題するなら、こちらの方が良いかもしれません。
あるいは、n=2の場合に解がないことを証明しなさい、という問題も面白いと思います。

楕円曲線E_n          三角形の3辺
n [0,a_2,0,a_4,0] rank(E_n(Q)) E_n(Q)の基底 [a,b,c]
---- ---------------- ------------ ------------ -------------------
1 [0, 9, 0, 20, 0] 0 -- --
2 [0, 12, 0, 32, 0] 0 -- --
3 [0, 17, 0, 52, 0] 0 -- --
4 [0, 24, 0, 80, 0] 0 -- --
5 [0, 33, 0, 116, 0] 1 [-9 : 30 : 1] [600, 241,409],[10763734800, 16354950889, 5858756089],...
6 [0, 44, 0, 160, 0] 1 [-8 : 32 : 1] [120, 29,101],[27415440, 27772729, 4569289],...
7 [0, 57, 0, 212, 0] 0 -- --
8 [0, 72, 0, 272, 0] 1 [-36 : 192 : 1] [120, 113,17],[23974080, 5547169, 19537249],...
9 [0, 89, 0, 340, 0] 1 [-5 : 20 : 1] [9360, 1769,10841],[2193892307923680, 335564764696849],...
10 [0, 108, 0, 416, 0] 0 -- --


718 名前:132人目の素数さん投稿日:03/12/30 00:57
11 [0, 129, 0, 500, 0] 0 -- --
12 [0, 152, 0, 592, 0] 0 -- --
13 [0, 177, 0, 692, 0] 1 [-121 : 858 : 1] [291720, 315121,31849],[986435564564501697360, 827765313659746079929,179031046508032125289],...
14 [0, 204, 0, 800, 0] 1 [-20 : 240 : 1] [2184, 685,1525],[3102786960, 3874500361, 796811929],...
15 [0, 233, 0, 916, 0] 1 [-49 : 630 : 1] [10920, 8297,2753],[1399306337629200, 184391575222969, 1561148767833769],...
16 [0, 264, 0, 1040, 0] 0 -- --
17 [0, 297, 0, 1172, 0] 1 [11236 : 245973 : 64] [116433779280,8405427241, 111771268729],[11670441288604186032511890990181342444475040,
                     9569225044530597073751214140775621146980849, 2233967780403995054623788795320612578673809],...
18 [0, 332, 0, 1312, 0] 1 [-200 : 2240 : 1] [6254640,6532649, 439289],
                   [121912996594096227283152480, 96186506119044368085563761, 26834058234370029973355281],...
19 [0, 369, 0, 1460, 0] 1 [31680 : 675120 : 1331]  [45798405066480, 32279148533449, 13821198008281],
                  [379625543589728572904666977237492035627972880961288160         86182251259758751530022455146839302807111409456623409,
                   463890202432451443335875107166868472678484477836703569],
20 [0, 408, 0, 1616, 0] 1 [-324 : 2880 : 1] [46800, 54781, 8269],[1328412804494162400, 1323626120375260129, 66733491312342529],...


719 名前:132人目の素数さん投稿日:03/12/30 00:58
21 [0, 449, 0, 1780, 0] 1 [-125 : 2200 : 1] [121265760, 112752209, 10409969], [14794720810392116418692000253120,
                    1167572755316636045893632472801, 13881540004580080404719114203681],...
22 [0, 492, 0, 1952, 0] 1 [-200 : 3360 : 1] [11651640,11532109, 545749],[835113361625825952930340080,
                    300163776632474933570433721, 538698603464021032708637641],...
23 [0, 537, 0, 2132, 0] 1 [9984 : 301600 :27]
???????
24 [0, 584, 0, 2320, 0] 0 -- --
25 [0, 633, 0, 2516, 0] 0 -- --
26 [0, 684, 0, 2720, 0] 1 [-40 : 960 : 1] [15600, 5641,10009],[3859613000224800, 5654150911242289, 1798724912017489],...
27 [0, 737, 0, 2932, 0] 0 -- --
28 [0, 792, 0, 3152, 0] 0 -- --
29 [0, 849, 0, 3380, 0] 2 [-13 : 312 : 1], [-729 : 7830 : 1]
[737760, 31681,719329],[412064257777120320, 335396785055745169, 78269171337328849],...
30 [0, 908, 0, 3616, 0] 0 -- --


720 名前:132人目の素数さん投稿日:03/12/30 02:28
>>713
これのどこが解答なのかわからん。
nでパラメトライズされた楕円曲線の有理点をもとめる問題に還元したとして
でnに関する条件ってのはなんなんだ?答えがたとえばn≡3 (mod8)のときは
解があるとかなんとかそんな形になるならともかくこんなのが答えっていわれて
なっとくいくわけね―。
大体このスレで出題者が思いつきでつくった問題かどうかは重要な問題で
何回も出題者のレスをもとめられてるのにそれにちゃんと答えるつもりがないなら
出題すんなよ。

721 名前:132人目の素数さん投稿日:03/12/30 02:35
あれ、なんか腹の中から黒い感情がこみ上げてくるよ。

722 名前:132人目の素数さん投稿日:03/12/30 03:00
>>713-715>>717-719
不正解。


723 名前:132人目の素数さん投稿日:03/12/30 03:37
すみませんが>>644の完全な解答をお願いしまつ

724 名前:132人目の素数さん投稿日:03/12/30 10:26
一辺が1の正方形ABCDが有ったとする。
ADの中点をE、BCの中点をFとし、AB上で点Rを移動させる。
Rと点E、点Fを結び延長した線と、CDを延長した線との交点を
それぞれP,Qとすると、△RPQの面積は1となる。
底辺PQの長さはRに無関係で2である。
r=ARとする
|RP|=√(1+4r^2)
|RQ|=√(1+4(1−r)^2)
この二つの長さがともに有理数になれば、この△RPQの
整数倍の相似な三角形は題意を満たすはず。


725 名前:132人目の素数さん投稿日:03/12/30 10:39
rが有理数なら、既約分数をn/mとおける(m>0)
|RP|=√(m^2+4n^2)/m
|RQ|=√(m^2+4(m-n)^2)/m
m^2+4n^2 及び m^2+4(m-n)^2 が共に平方数となれば良い

726 名前:644投稿日:03/12/30 11:26
>>704
ああ、そう言えば
Xが論理的人道主義的海賊(Logical Humanity Pirates LHP) であるとは、
海賊集合の元Xが論理的推論者でかつ、
宝石の数がいっしょなら殺しを避けてしまう
と定義した場合の考察をしてなかった。

E[100]
D[0-100]
ここでDが1枚でも欲張ると殺される。
人道主義的海賊Eにとって、Dの命は宝石一個よりは軽い。

C[100-0-0]
Dは自分の番との獲得枚数が一緒なので賛成する。

B[100-0-0-0]
ここで人道主義炸裂。

A[100-0-0-0-0]
4/5の賛成を集め無事可決。

>>723
いやぁ、、本を買うという方法がありまっせ。

727 名前:132人目の素数さん投稿日:03/12/30 12:54
題意で明らかにされていない海賊の行動原理はあるものの、
回答としては
100-0-0-0-0と97-0-1-2-0と97-0-1-0-2の三種類があるんやね。

728 名前:132人目の素数さん投稿日:03/12/30 15:00
>>621
(a,b,c,n)=
(4,13,15,3),(84,949,1025,5),
(3,25,26,8),(84,965,1009,10),
(8,123,125,15),(28,1345,1371,18),
(5,122,123,24),(28,1347,1369,30),
(12,425,427,35),(4,193,195,42),
(8,505,511,42),(7,339,340,48),
(12,1189,1199,55),(6,481,485,60),
(16,1015,1017,63),(9,724,725,80),
(15,1489,1496,88),(20,1989,1991,99),
(15,3361,3374,112),(11,1325,1326,120),
(24,3443,3445,143),(13,2190,2191,168),
(11,2665,2670,216),(15,3367,3368,224),
(3,865,866,272),(4,2701,2703,585)。


729 名前:132人目の素数さん投稿日:03/12/30 16:00
>>727
100-0-0-0-0は明らかに無いだろ。

730 名前:132人目の素数さん投稿日:03/12/30 22:55
あのとき見た問題での条件:
・提案者も1票持ち・同数可決(実質1.5票持ち)
・自分の命が何より
・他人の血が流れるのを見るのは好き
・しかし何人死のうと金貨1枚には値しない

つまり同じ枚数の時のみより後の案を好む
つまり分け前たったの1枚で手のひらを返す

逆算して表にすると200人まで芸術的結果になったなぁ。
で201人目と203人目で何かが起こったのですよ。ええ。

0.5票持ちとか−0.5票持ち(自分以外で「過」半数必須)とかだとどうなるんだろ?

731 名前:730投稿日:03/12/30 23:23
あ、勿論海賊は全員論理的だったとしてです。

【補題】さて同様の配分が100枚ずつ連続で何回か(例えば3回)あったらどうなるのだろうか?
上座に近い人間は提案権を得ようと目の前の金貨1枚を捨てるのか?

追加条件:
死者の財産の再配分はしない(現実的にはまずありえないが)
累計資産により差別しない(再配分が無くても現実的には金持ちは恨まれるものだが)

でも「これまでの配分結果に(頭数の変動を除き)影響されない」ようにしないと
逆算で計算し切れないし。

判断基準の順位:
長く生き延びる>手持ち金貨が多い(処刑時も)>斬殺ショーを多く楽しむ

732 名前:132人目の素数さん投稿日:03/12/31 02:50
>>644がいまだによくわからない_| ̄|○

正答は>>683でおk?

733 名前:132人目の素数さん投稿日:03/12/31 03:20
さぁて、今年も後僅かなわけだが、
2004年にちなんだ問題って言うのはまだでんのか?

一応、某サイトにもうぷったが、ここにも即興で作った
拙い2004年問題上げとくな・・・・・・まぁあまりにもつまらないわけだが。


---問題---
Σ[k=1,167] k/(x-k) ≧ 7
を満たすxの存在する範囲の長さを求めよ。
----------


で・・・誰か、もっと面白い新年問題プリーズ。

734 名前:132人目の素数さん投稿日:03/12/31 03:52
>>733
某サイトってどこですか?
その問題にも興味ある

735 名前:132人目の素数さん投稿日:03/12/31 12:14
題意は同数否決じゃないの?

736 名前:132人目の素数さん投稿日:03/12/31 12:22
>733
xに制約条件(整数とか、非負とか)ないと厳しいです

737 名前:132人目の素数さん投稿日:03/12/31 12:30
(7になるxの和)−(∞になるxの和)。


738 名前:132人目の素数さん投稿日:03/12/31 13:37
>>733
http://www.kalva.demon.co.uk/imo/isoln/isoln884.html

これが元ネタ。出題方法間違えたかな・・・
えーと、>>734その某サイトにも同じ問題を上げているから
見ても意味がない。っていうかまだ、更新されていないので
問題が上げられるのは、後日っていうところだね。



>>736
そうかなぁ? グラフ描いてみたけど、なんとなく求まりそうだけど?

739 名前:132人目の素数さん投稿日:03/12/31 17:40
>>738
元ネタ、って答えを載せてるようなもんじゃないw
2004になるように作り変えた、ってことね?

740 名前:132人目の素数さん投稿日:04/01/02 00:02
>>730
>・提案者も1票持ち・同数可決(実質1.5票持ち)

問題に深みがないような。

741 名前:132人目の素数さん投稿日:04/01/02 11:37
(そろそろ、まとめページが欲しいですね)

で、問題:
紅白のカードがそれぞれn枚ずつあり、これらを横一列に並べます。
左からm枚目までのカードに注目したとき、どんなmについても、
紅のカード≧白のカード
となるような並べ方は何通りありますか。
nについての関数で、f(n)=○○○という風に解答して下さい。

742 名前:132人目の素数さん投稿日:04/01/02 12:54
f(n)=0,5×(n!/(2×0,5n!)) ?



743 名前:132人目の素数さん投稿日:04/01/02 13:48
階乗を使うのは正解ですが、まだ間違いです。
例えば、n=3のときには、

紅紅紅白白白
紅紅白紅白白
紅紅白白紅白
紅白紅紅白白
紅白紅白紅白

となるので、f(3)=5 で、5通りとなります。

744 名前:132人目の素数さん投稿日:04/01/02 14:04
>>741
f(n)=(2n)!/((n+1)!*n!)

745 名前:132人目の素数さん投稿日:04/01/02 14:13
大正解! ネタを明かすと、カタラン数をちょっとアレンジしてみました。

http://web2.incl.ne.jp/yaoki/report.htm
http://www80.sakura.ne.jp/~aozora/taiwa/node85.html


746 名前:132人目の素数さん投稿日:04/01/02 15:20
次の数列を求めよ
11、40、11、58、87、17、63、47、30、・・・


747 名前:132人目の素数さん投稿日:04/01/02 15:54
単調増加(減少)じゃない数列の一般項って求めるのムズそう。
11→40
11→58
と、2通りあるってことは、単に a_n+1 - a_n を考えても無駄ってことだよね。


748 名前:毒男板から出張中投稿日:04/01/02 16:09
四つの玉を一つの袋に入れます。玉は黒が一つ、白が一つで、他の2つの玉は赤です。
袋をよく振って、誰かが袋から2つの玉を取り出し、取り出した玉の一つは赤だ、と言いました。
もう一つの玉がやはり赤である可能性は、どれくらいあるでしょう?


749 名前:132人目の素数さん投稿日:04/01/02 16:12
>>748
当然の事かも知らんが一応。

> 取り出した玉の一つは赤だ

> 取り出した玉2つの内、少なくとも一つは赤だ
という意味だよな?

750 名前:毒男板から出張中投稿日:04/01/02 16:14
>>749
はい、そうです

751 名前:132人目の素数さん投稿日:04/01/02 16:22
>>748
楽勝! >>749 の仮定の通りだとして、確率1/5

752 名前:毒男板から出張中投稿日:04/01/02 16:29
>>751
正解です!!

ところで、といってはなんですが。
なぜそう言う解答に至ったのか、説明して貰えませんか?

只今の論点は、答えはなぜ1/3でなく1/5なのかなのです。

753 名前:毒男板から出張中投稿日:04/01/02 16:33
もっと言うと、
赤玉1と赤玉2の組合せは
@最初に取ったのが赤1 もう一方は赤2
A最初に取ったのが赤2 もう一方は赤1
の二通りの組合せがあるのではないか。

というところです。

754 名前:132人目の素数さん投稿日:04/01/02 16:35
>>753
ここ行ったら?

分からない問題はここに書いてね145
http://science2.2ch.net/test/read.cgi/math/1071929807/


755 名前:132人目の素数さん投稿日:04/01/02 16:35
>>752
書き出したら分かる訳だが・・・
赤1赤2、赤1黒、赤2黒、赤1白、赤2白

756 名前:132人目の素数さん投稿日:04/01/02 16:39
>>753
ああ、そういうことね。
>赤1黒、赤2黒、赤1白、赤2白
では赤玉を区別するのに、なんで赤玉同士だと区別しないかってこと?

…説明が難しいな。

757 名前:132人目の素数さん投稿日:04/01/02 16:41
>>753
それは組み合わせとは呼ばない。順列。
赤1赤2と赤2赤1は同じ組み合わせだから区別しようがない。

758 名前:132人目の素数さん投稿日:04/01/02 16:47
>>756
難しくないだろ。順列で考えて赤1赤2と赤2赤1を区別するなら、赤1黒と黒赤1も
区別して考えなきゃいけないだろ?

759 名前:132人目の素数さん投稿日:04/01/02 16:47
>>752 じゃあ説明しますと、赤玉は2つあるので、赤A,赤Bと表記すると、
1(赤A,赤B)
2(赤A,白)
3(赤A,黒)
4(赤B,白)
5(赤B,黒)
6(白,黒)
の6通りが考えられる。「取り出した玉2つの内、少なくとも一つは赤」ということは、最後の6は除外される。
よって、確率1/5

760 名前:132人目の素数さん投稿日:04/01/02 16:47
>>753
「赤1黒」と「赤2黒」を区別するんなら
「赤1赤2」と「赤2赤1」も区別しないと!ってことか。
前者は組み合わせが違い、後者は順序が違うよね。
この2つの区別は意味が違う。
組み合わせが問題になっているので順序の区別は必要ない。

761 名前:132人目の素数さん投稿日:04/01/02 16:49
>>757
ありがとうございます。

762 名前:毒男板から出張中投稿日:04/01/02 16:50
>>755-760
ありがとうございます。
これで帰れます。

763 名前:132人目の素数さん投稿日:04/01/02 16:53
>>753の考え方で(無駄な手順を踏んで)解いても、
結局1/5になるはずなんだけどな。
赤1と赤2のとこだけ特別扱いするこたない。

764 名前:132人目の素数さん投稿日:04/01/02 22:32
箸休めにでもドゾー。

中学生になった気分で求めよ(中学数学の範囲で。三角関数は使用不可)

問題1
∠A=108°、AB=AC、の二等辺三角形ABCの3辺の比

問題2
∠A=30°、AB=AC、の二等辺三角形ABCの3辺の比


おおまかにでも方針なり過程なりは書いてね。じゃなきゃどんな解き方したか分からんから・・・

765 名前:132人目の素数さん投稿日:04/01/03 00:02
>>764
2次方程式の解の公式は使ってOK?

766 名前:132人目の素数さん投稿日:04/01/03 00:06
>>765
中学範囲だからおk

767 名前:132人目の素数さん投稿日:04/01/03 00:08
>>764
(1) は、108°を 36°と72°に分けて、後は相似。
(2) は、AC上にDを、∠ABD=45°となるようにとって、
DからABに垂線を下ろした。
後は相似と三角定規で。

むずひ……。

768 名前:132人目の素数さん投稿日:04/01/03 00:25
>>767
正解!

解法はいくらでもあるけどね。(2)なんか特に。
(2)は∠CAD=30°となるBとは異なる点DをBC上にとって
角の二等分線の公式で、AB:AD=BC:CDってやって解くのが一番楽かな?

いずれにせよめんどいけどね。
でもこれが実際に高校入試で出てたりするんだよなぁ。

769 名前:132人目の素数さん投稿日:04/01/03 00:31
すまん。意味通じるとは思うけど訂正

× 点DをBC上にとって
○ 点Dを直線BC上にとって

770 名前:132人目の素数さん投稿日:04/01/03 07:50
年末にフジテレビで確率ネタの番組をやってました。
そこでモンティホールディレンマがとりあげられてました。
知らなかったのですが、面白いので改題してみました。

3つの箱のうちひとつだけがあたり。主催者だけがあたりの箱を知っている。

(1) 回答者が箱をひとつ選んだあと、主催者は残り2つのうちはずれの
ひとつをオープンして見せる。ここで回答者は選ぶ箱を替えることができる。
回答者は箱を替える方が得か?(もとの問題)
(2) 回答者が箱をひとつ選んだあと、第三者が残り2つのうちひとつを
オープンしたところはずれだった。ここで回答者は選ぶ箱を替えることができる。
回答者は箱を替える方が得か?

771 名前:132人目の素数さん投稿日:04/01/03 09:47
(2)替えても替えなくても同じ。

772 名前:132人目の素数さん投稿日:04/01/03 15:09
円形の池で女の子がボートをこいでいると池のほとりに痴漢が現れた。
痴漢は池の周囲を先回りして女の子を池のほとりで待ち構えまている。
痴漢の走る速さは女の子がボートをこぐ速さの4倍である。

問題:女の子が無事逃げるにはどうしたらいいか。

既出だったらスマン

773 名前:132人目の素数さん投稿日:04/01/03 17:54
>772
大声を出して助けを呼ぶ。

774 名前:132人目の素数さん投稿日:04/01/03 18:06
>>772
携帯で助けを呼ぶ。

775 名前:132人目の素数さん投稿日:04/01/03 18:20
>>772
入水自殺
天国へ逃げられます

・・・てかこのスレでは既出ではないかもしれんが、
回答を真面目に書く気にはならん。

776 名前:132人目の素数さん投稿日:04/01/03 18:32
>>773近くに助けを呼べるような人はいません
>>774携帯は持ってません
>>775現実世界でないと逃げたことになりません

一応ちゃんとした数学の問題です

777 名前:775投稿日:04/01/03 18:47
>>776
一応言っておくが知ってるよ。
放置された私のWebページ上にある問題。
1-(1/4)πと1/4の大小関係。

778 名前:132人目の素数さん投稿日:04/01/03 19:30
岸にあがっところで痴漢に出会ったら
股間を蹴っ飛ばして逃げる。

779 名前:132人目の素数さん投稿日:04/01/03 20:06
パパ、マジレスしちゃうぞ。
池の直径を1として、中心から直径1/4よりわずかに内側で
ぐるぐる回っていれば、痴漢−中心−自分が一直線になる位置が取れるはず。
そこからおもむろに陸に向かって漕げば、距離3/8。痴漢は
π/2回ってくるのでかろうじて逃げ切れる。

こう?

780 名前:132人目の素数さん投稿日:04/01/03 21:06
痴漢のスピードをどこまで上げられるか考えると面白そう。
かなり難しそうだけど。

781 名前:770投稿日:04/01/03 22:50
770です。はじめに答えありきで作ったんで、つまんなかったスマソ
(1) 選択を替えるべき
(2) 替えても意味なし(>>771さん正解)

782 名前:775投稿日:04/01/03 22:52
>>780
1-(1/x)π < 1/x
なるx

783 名前:132人目の素数さん投稿日:04/01/04 01:14
>>782
ダウト。陸に向かって漕ぐところまでは>>779と同様にして、
半径の中点まで来たら進行方向を痴漢と反対方向に60度曲げ、
あとは陸まで直進すれば、xの値は4.2までは上げられる。

784 名前:780=783投稿日:04/01/04 01:18
痴漢の最大速度に対応する女の子の軌跡は、
何かの螺旋になる気がする。

785 名前:132人目の素数さん投稿日:04/01/04 02:09
>>779正解!
正解者が出たら>>780を問題2にしようと思ってました。

問題2:女の子が無事逃げられるための(ボートの速さに対する)痴漢の速さの最大値(または上限)を求めよ。

4.2倍よりもっと上げられます。

786 名前:132人目の素数さん投稿日:04/01/04 02:47
円周上を動く痴漢を常に正面に見るようにボートを漕ぐ。
計算マンドクセ

787 名前:132人目の素数さん投稿日:04/01/04 06:02
この問題の出題者ではありませんが、追問として
「また、このとき女の子が漕ぐ距離を求めよ」なんてのはどうでしょう?

788 名前:132人目の素数さん投稿日:04/01/04 06:13
池をx^2+y^2=1、女の子と痴漢の初期位置それぞれ(0,0),(-1,0)
女の子と痴漢の速さをv、kvとしたとき

(dx/dt)^2+(dy/dt)^2=v^2
{(sin kvt)+y}(dy/dt)={(cos kvt)+x}(dx/dt)

こんな微分方程式が出てきた。解けねぇ‥

789 名前:779投稿日:04/01/04 11:46
>>785
求める上限をxとすると、池の中心、直径の1/xより外側を女の子が
移動しているときは、痴漢のほうが角速度は速いので、痴漢は一方向に
追いかけるほうが有利。
一方女の子は、1/xの円周上で中心をはさんで痴漢と反対の位置をとった
あと、痴漢が移動をはじめたら上陸点を決めてそこへ直線で進むほうが
有利。
すると結局痴漢の移動方向と逆に90度折れてまっすぐ進むのが最適で、
このときx上限は
x=4.6033...(tan(θ)=θ+π、x=1/cos(θ)の解)

かな?

790 名前:132人目の素数さん投稿日:04/01/04 12:19
ちょっとまってくれ。
ボートから降りた後、高速で走る痴漢から走って逃げ切れるのか?

791 名前:132人目の素数さん投稿日:04/01/04 23:09
>>789
>あと、痴漢が移動をはじめたら上陸点を決めてそこへ直線で進むほうが
>有利。
>すると結局痴漢の移動方向と逆に90度折れてまっすぐ進むのが最適で、
 
この部分は証明がいるのでは?

792 名前:132人目の素数さん投稿日:04/01/05 00:14
モンティーホールジレンマの一般化。

n個の箱にm個のあたりが入っている。(n-m>=2,m>=1,ひとつの箱にあたりが2個以上入ることはない。)
今、ゴンザレス君は箱Aを選んだ。あたりがどの箱にあるかを知っているクリスティーナちゃんは
あたりが入っていない箱Bをあけてゴンザレス君に見せて、今なら別の箱に変えてもいいよ、と言った。

この条件の下で、ゴンザレス君から見て、箱をA,B以外に変えたときにあたる確率は、箱Aのままの場合
の何倍になるか?

ただし、クリスティーナちゃんは箱A以外ではずれの箱を一様の確率で選ぶとする。
(今はその結果が箱Bだった、ということ。)

793 名前:132人目の素数さん投稿日:04/01/05 00:27
>>772
今夜はこのネタでオナるか
妄想力をかき立てられた

794 名前:投稿日:04/01/05 01:30
>722
無限に早くできる気がする。

795 名前:投稿日:04/01/05 01:31
あ,今の>>785


796 名前:132人目の素数さん投稿日:04/01/05 01:50
そのこころは?

797 名前:775投稿日:04/01/05 02:20
この(>>772>>780)問題ってこんなに奥が深かったのか・・・
もうちょい考えてみよう・・・

798 名前:132人目の素数さん投稿日:04/01/05 23:14
>>792
n-m>=2が肝ですね。クリスティーナちゃんは必ずはずれ箱を
あけられるので、最初にゴンちゃんが選んだAがあたりの確率はm/n、
Aじゃない箱のあたり確率の総和は(m/n)*(n-1)で不変。
なので、箱を変更した場合にあたり確率は、
((m/n)*(n-1)/(n-2))/(m/n)=(n-1)/(n-2)倍になる。
かな?

799 名前:132人目の素数さん投稿日:04/01/06 00:30
>>798
結果は合ってるよ。でも、
「最初にゴンちゃんが選んだAがあたりの確率はm/n、Aじゃない箱のあたり確率の総和は(m/n)*(n-1)で不変。」
は自明な事実ではなく、クリちゃんがはずれを一様に選ぶことから導かれるんだよ。

問題がもし、「クリちゃんは箱Bを選べるとき(すなわち箱Bがはずれのとき)は絶対に箱Bを選ぶ(ことをゴンちゃんは知っている)」
場合はどうなる?

800 名前:132人目の素数さん投稿日:04/01/06 00:44
勝手に>>772の問題を弄ってみる。

時刻 t における女の子の位置を点 G_t 、痴漢の位置を点 R_t とする。
女の子と痴漢の移動速度をそれぞれ V_g 、V_r とおく。

1.勝手に初期条件を決める。
 ・女の子の初期位置 G_0 は、湖の中心。
 ・痴漢の初期位置 R_0 は、円周上のある点。
(それぞれを任意のままにしておくと、女の子が岸のすぐ傍からスタートしたり
 痴漢が湖の遥か遠方からスタートしたりしてしまうケースを想定しなくてはならず、
 上限等を求めるのが困難)

2.移動速度不変を明示
 ・女の子、痴漢はそれぞれ常に任意の方向へ一定の速度で移動することができ、
  方向転換によるロスや同方向に進み続けることによる加速等は無いものとする。

3.勝手に痴漢の移動ルールを制限
 ・痴漢は、常に円周上で最も女の子に近い点A_tについて、弧 R_t A_t が短い側の
  弧上をA_t に向かって移動する。
(女の子の移動アルゴリズムを分析して対処したり、フェイントをかけたりといった
 知的な行動や、無駄な遠回りなどは行わないことを定義しておく)

4.勝手に脱出条件を定める
 ・女の子が円周上の任意の点に到達したとき、G_t R_t 間に d 以上の距離が
  無くてはならない。
(d を「0より大きければ可」などとしてしまうと、『アルキメデスと亀』の屁理屈によって
 痴漢は無限に近い速さを持っていても絶対に女の子を捕まえられない)


801 名前:800投稿日:04/01/06 01:05
とりあえず、G_t と R_t は原点を湖の中心に置いて(長さ, 角度)の座標系で表すと計算し易そうかな。
R_0の座標を (r,0) (r は湖の半径)として、R_t は (r , θ_rt) と1変数で表せるし、
G_t を ( x_t, θ_gt ) と置くと、x_t ≠ 0 のとき A_t は (r, θ_gt) になるので、
V_r とか d とかを角速度・角度差に換算しておけば、痴漢のほうの動きは簡単な式になりそう。


802 名前:132人目の素数さん投稿日:04/01/06 10:03
>>800
4.は必要かな?少なくともアルキメデスと亀の話は違うと思う。
どんなに小さなdをとっても、ある距離以上進めばアルキメデスは
亀との距離をd以下にできるのだから。
むしろ、女の子と池の中心との距離を単調増加させるという条件を
つけないといくらでもトリビアルに解が増えてしまいそう。

803 名前:132人目の素数さん投稿日:04/01/06 13:50
>>785
求める上限をxとすると、池の中心から、直径の1/xより内側を
ぐるぐる回っていれば、痴漢−中心−自分が一直線になる
位置が取れるはず。
そこからおもむろに、常に痴漢−中心−自分が一直線になる
ようにして陸に向かって漕げば、xの値にかかわらず、また痴漢が
どんな動きをしても、必ず逃げ切れる。


804 名前:798投稿日:04/01/06 14:56
>>799
たしかにそうですね。

逆にクリスティーナがはずれ箱をえらぶ方針によらない問題を考えると、

1. 最初に選んだ箱を替えない。
2. 最初に選んだ箱から、別の(開かれていない)箱のうちの無作為に選んだ
一つに替える。
という2つの方針の、あたり箱をひく事前確率の比ならば(n-2):(n-1)

という感じかな。

805 名前:132人目の素数さん投稿日:04/01/06 14:56
>>803
>そこからおもむろに、常に痴漢−中心−自分が一直線になる
>ようにして陸に向かって漕げば、

無理だろ

806 名前:789投稿日:04/01/06 16:32
>>791
この問題、ちゃんとは説明できないなあ。自分では正しいと思うけど
自信はない。

痴漢と女の子の速さをそれぞれx(>1)、1とし、池の半径をxとする。
従って、痴漢は池の周囲を、中心から見て角速度1で移動できることになる。
女の子の方の角速度についてみると、池の中心から半径1の円の内側では痴漢より
速く移動できるが、外側では痴漢より遅くしか動けない。

ここで
・女の子は、初期状態では池の中心にいるとする(これは勝手に決めたけど
出題の意図に反しないはず。池の何処からでも中心には行けるので。)

逃げる手順としては、
(1) 女の子は、池の中心半径1の円周上で、痴漢-池の中心-女の子が一直線となる
位置をとる。 *1
(2) 女の子は、(1)の直線に沿って池の周方向へ漕ぎはじめる。
(3) 痴漢は、女の子が池の中心半径1の円から外へαだけ進んだ時点で右周り
(左でもいい)に池の周に沿って走りはじめる。 *2
(4) 女の子は、(3)で痴漢が移動をはじめた時点で、進行方向を痴漢と逆側に
曲げ上陸点に直進する。 *3
(5) 上陸点-池の中心を結ぶ線と(1)の直線とのなす角度をθとする。
-π<θ<πかつx*cos(θ)>=1の範囲で考えればよい。*3

つづく

807 名前:789投稿日:04/01/06 16:33
つづき

女の子が逃げ切れるためには、
(π+θ)^2 > x^2-2*x*cos(θ)+1

ここでF(θ,x)=(π+θ)^2-(x^2-2*x*cos(θ)+1)とすると、
∂F/∂θ=2*(π+θ-x*sin(θ))、
∂^2F/∂θ^2=2*(1-x*cos(θ))だから、上記θの範囲では、
x>=x_1のとき、∂F/∂θはπ+θ-x*sin(θ)=0となり得、このとき
Fは最大値-(x*cos(θ)-1)^2 <=0 をもつ。
x<x_1のとき、∂F/∂θはπ+θ-x*sin(θ)>0であり、x*cos(θ)=1なるθで
Fは最大値(π+θ)^2-(x*sin(θ))^2 >0 となる。
ここでx_1=1/cos(θ_0), π+θ_0=tan(θ_0)である。

したがって、女の子が逃げ切れるためには、上記x_1=4.6033...が上限値であり
また、上陸点として、x*cos(θ)=1すなわち(1)の直線から、痴漢が回ってこない側へ
90度向きをかえて直進したとき到達する点とすればよい。

*1 女の子は、池の中心半径1の円より内側では、痴漢との位置関係を自由に
選べる(中心から任意の距離r<1の円周上を、痴漢より速い角速度で回れるので。)

*2 女の子が池の中心半径1の円より外側に漕ぎ出したら、痴漢は一方向に追い
かけるほうが有利。女の子-中心-痴漢のなす角度を単調に減らしていくことが
できるので。もし途中で追いかける方向を反転すると、痴漢-池の中心-女の子
が一直線になる瞬間(1)と同様の状況になり、かつ女の子は池の中心から1以上
離れている可能性があり、不利。

*3 女の子は、池の中心半径1の円より外側に漕ぎ出したら、この円の周上、内側
には戻らないとする(戻ったら*1より、初期状態からやり直しと同様なので。)


808 名前:132人目の素数さん投稿日:04/01/06 17:41
>>806
>(4) 女の子は、(3)で痴漢が移動をはじめた時点で、進行方向を痴漢と逆側に
>曲げ上陸点に直進する。

なんで「直進」なの?
螺旋状に進んだほうが効率が良くなる可能性はないの?

809 名前:789投稿日:04/01/06 18:23
>>808
自信がある訳じゃないけど、
・痴漢が一方向全速力でしか追ってこないのなら、最適解の軌跡はスタートした
ときに決まってしまい、よって上陸点も一意に決まるだろう。
・上陸点が決まっているなら、そこへ到達するには線速度の上限が決まっている
のだから直進するのが一番速いはず。
という発想です。

810 名前:132人目の素数さん投稿日:04/01/06 18:30
>>809
軌道を変えると、痴漢の動きも変わることに注意。

811 名前:789投稿日:04/01/06 18:50
>>810
この場合、女の子の動きかたによって痴漢が動きかた(一方向回転、
中心からみた角度が追い付くまでは全速力)を変更するメリットって
あるんだろうか。それが想像できないんですよ。

812 名前:808投稿日:04/01/06 18:52
>>809
90度向きを変えて直進して到達するだろう点よりも
さらに最初痴漢がいた側の点に到達したい場合は?
この点に直進で行こうとすると中心からの距離が単調増加にならずナンセンスだ。

>>810
中心からの距離が単調増加になるように動くのであれば、それはない。

813 名前:132人目の素数さん投稿日:04/01/06 18:53
>>808
「効率よく」の定義によるんじゃない?
最短時間で逃げるなら、可能な限り直進すべき。
常に痴漢との距離を最大に保ちたいのであれば、>>803になる。

ちなみに>>803のケースで、痴漢が一方向全速力で移動すると
仮定すると、ボートの軌跡は螺旋状にぐるぐる回るはず。
中心からスタートして、ボートがこの軌跡で移動して、
中心からの距離が直径の((1/2)-(x/π))倍になった時点で最も
近い陸地に向かって直進すると、最短時間で逃げられる。
(直進開始後の任意の時刻に痴漢が向きを変えたとしても、必ず逃げられる)


814 名前:132人目の素数さん投稿日:04/01/06 18:55
誤:((1/2)-(x/π))倍
正:((1/2)-(π/x))倍


815 名前:132人目の素数さん投稿日:04/01/06 19:03
>>806-807
これじゃだめだろ。Step(4)で女の子が進行方向を曲げた直後に痴漢が
走る方向を反転させうる。つまり
痴漢がどんな戦略をとって女の子を追跡してもおいつけないことの証明にはなってない。

816 名前:808投稿日:04/01/06 19:11
>>815
痴漢が走る方向を反転させたら女の子に有利になるだけ。
>>807の*2

817 名前:789投稿日:04/01/06 19:21
>>816
自分でいうのも何だけど、そうも言い切れないような。
どちらも相手の動きをみて、遅れなく反応できるという条件なので
どちらも池の中心をはさんだまま動き出せないという問題がある。

ただ、一度女の子-中心-痴漢の直線が崩れたら、あとは痴漢は
一方向に追うべきだとは思うけど。

818 名前:808投稿日:04/01/06 19:36
>>808の疑問を図にしてみた。

http://garden.under.jp/lounge2chup/img/lounge2ch470.jpg

>>806さんによれば矢印1より矢印2の方がいいらしい。
でも矢印3の経路は考慮しなくていいの?ということです。

819 名前:132人目の素数さん投稿日:04/01/06 19:39
>>817
その問題は無視していいんじゃないの。

820 名前:132人目の素数さん投稿日:04/01/06 20:04
>>816
もちろん女の子に有利になるけど有利になったことを利用して女の子は進行方向を
変化させないといけない。痴漢もそれをみてまた進行方向を変化させうる。
女の子も痴漢も相手の出方に応じて進行方向を変化させうるので結局証明は
相手がどんな戦略をとったとしても女の子が逃げうることをしめさないといけないんだから
ダメだとおもう。「有利になる」じゃなくて「痴漢がこうきたら女の子はこう応じる」という具合に
記述しないと。

821 名前:789=806投稿日:04/01/06 20:08
>>818
その図右半分、2の到達点の下側の領域の岸に到達する経路で、
内側の円の中に入らない最短経路は、
出発点から内円周に沿って動き、その接線が上陸予定点にあたるような
点から直進するような経路になるはず。(糸を張って引っ張ればそうなるよね。)
内円周をうごく角速度が痴漢角速度と同じことを考えると、結局
それは矢印2と同じことになるはず。
他の螺旋経路を通れば、同じ上陸点により遅く到達するので
それは解とはならないはず。

822 名前:132人目の素数さん投稿日:04/01/06 20:42
答えはでてないんだけど>>785の問題を数学的にどう定式化するか考えてみた。
以下D={(x,y) | x^2+y^2≦1} S={(x,y) | x^2+y^2=1}
で以下のように解釈してみたらどうだろう?
 
―問題―
正の実数vで以下の条件をみたす関数F(θ,p):S×D→R^2が存在するものの
下限をもとめよ
(条件)
(I) |F(θ,p)|≦v
(II) 任意の|dθ/dt|≦1をみたす関数R→Sにたいして積分方程式
∫[0,t]F(θ(u),p(u))du=p(t) (0≦t≦T)
を満足するようなp(t),Tが存在してさらにそれが
(i)p(t)∈D
(ii)p(t)∈S⇒t=T
(iii)p(T)≠θ(T)
も同時に満足する。
 
つまり女の子の逃避の戦略は痴漢の位置θと女の子の現在地pについてどちらのむきに
どれだけの速度でこぐかをあたえる関数F(θ,p)であたえられてその戦略にしたがって
逃走すればT秒後にはSに到達しその位置はθ(T)でないと。
どう?もちろんまだ全然とけないんだけど。

823 名前:132人目の素数さん投稿日:04/01/06 20:44
痴漢、大人気だなw

824 名前:132人目の素数さん投稿日:04/01/06 20:45
>>821
>>818の赤い点(現在地)からは、矢印1の上陸地点の方が矢印2の上陸地点
の方が明からに近い。と言うことは、矢印2の方向に進み出した瞬間、
痴漢は向きを変えるはず。
すると女の子も向きを変えなければいけない。仮に180度向きを変えると、
痴漢もまた逆向きになる。これを繰り返すと、現在地から動けなくなる。

ゆえに矢印2は成立しない。矢印1しかありえないと思うけど。


825 名前:824投稿日:04/01/06 20:51
>>807 の*2で、
>もし途中で追いかける方向を反転すると、痴漢-池の中心-女の子
>が一直線になる瞬間(1)と同様の状況になり、
ここまでは正しいが、
>かつ女の子は池の中心から1以上離れている可能性があり、不利。
痴漢-池の中心-女の子が一直線になったにもかかわらず、角速度は
痴漢の方が上なんだから、痴漢が有利になる。


826 名前:132人目の素数さん投稿日:04/01/06 21:08
最高難度
新しいメルセンヌ素数を探す

827 名前:132人目の素数さん投稿日:04/01/06 22:48
>>824
痴漢がめざすのは女の子から最短距離にある岸であって
矢印2の上陸地点ではないよ。

828 名前:824投稿日:04/01/06 23:00
>>827
痴漢(または女の子)がめざす場所がどこであっても、
女の子が内円周からわずかでも外へ出た瞬間に、
痴漢は女の子と負号の異なる角速度で移動する。


829 名前:789=821投稿日:04/01/07 00:18
>>824
まあ、確かに789の方針では817にも書いたように、両者とも動かず
だめだよね。ただ、最近点に直進した場合の速度比の最大値は
π+1=4.14...だけど、比が4.2の場合でも確実にいける方法が
あることはわかっているわけで、直進以外の最適解があるはず。
自分にはちょっとわからないけど…

830 名前:132人目の素数さん投稿日:04/01/07 00:36
とりあえず>>785の解答を聞きたい。ホントにできてんのかな?

831 名前:824投稿日:04/01/07 00:43
>>829
おれもまだちゃんと考えてないんだけど、最初矢印1の方向(垂直)に
進んで、それから痴漢と角速度の符号が等しくなるように、向きを
少しずつ変えていくのがいい思う。
痴漢は停止している訳にはいかないから、どちらかの向きに全力疾走
する。もし途中で痴漢が向きを変えたら、角速度の絶対値を少しずつ
小さくしていって、痴漢―中心―女の子が一直線上になったときに
再び垂直方向になるような動き方が最適になるんじゃないかな。
そうすれば、x>4.2でも逃げ切れるはず。


832 名前:808投稿日:04/01/07 00:52
>>825
>痴漢-池の中心-女の子が一直線になったにもかかわらず、角速度は
>痴漢の方が上なんだから、痴漢が有利になる。

そうか?
女の子が有利だと思うが。

>>828
>痴漢(または女の子)がめざす場所がどこであっても、
>女の子が内円周からわずかでも外へ出た瞬間に、
>痴漢は女の子と負号の異なる角速度で移動する。

んなことないでしょ。
>>827の言うとおり、痴漢が目指すのは
その時々で女の子から最短距離にある岸でしょ。

833 名前:132人目の素数さん投稿日:04/01/07 00:53
最初にぐるぐる回るとき、円じゃなくて楕円軌道を取って回れば少し外側を回れるかな?

834 名前:808投稿日:04/01/07 00:54
なんで痴漢が逆向きに走ろうとするのかさっぱりわからん。

835 名前:808投稿日:04/01/07 01:01
>>821
そうですね。
矢印3は考えなくていいですね。
となると俺は矢印2が解であるに一票。

836 名前:132人目の素数さん投稿日:04/01/07 01:04
なあなあ、痴漢に捕まらないように
池のほとりまで逃げ切れたとしよう。
ここからが問題だよ。
当然女の子が陸地に上がってからも
痴漢は追いかけてくるんだろ?
周りには助けてくれる椰子なんかいないみたいだし
とどのつまりが
女の子はお触りに対して甘受しなけりゃいけないってことだろ?
それともなにかい?
女の子が痴漢から逃げ切れるかどうかを
数学的に証明できるってのかい?

837 名前:132人目の素数さん投稿日:04/01/07 01:12
女の子の地上での移動速度を痴漢のそれよりも大きいと定めれば、問題なく逃げ切れるよ。

838 名前:808投稿日:04/01/07 01:13
男の最適戦略としては、
「男が池の中心を見据えた時に女の子が左側にいる限りは時計回りに、
 池の中心を見据えた時に女の子が右側にいる限りは反時計回りに追う」
でよろしいかな?
女の子の進行方向は関係なしなんだけどもいいでしょうか?

で、女の子の戦略としては、
内円周から少し出たとこで一瞬待つ。
仮に男が時計回りに追ってきたとしたら、
右に90度進行方向を曲げて一直線に岸へ向かう、と。
(男から見ると常に中心より左側に女の子がいることになるので、
 男は時計回りに追い続けるはず)

839 名前:あ(794)投稿日:04/01/07 01:22
お前らアホだな。まだやってたのかこの問題
中学生の僕にでも分かるぞ。

840 名前:あ(794)投稿日:04/01/07 01:23
ちなみに,痴漢の速さは無限に速くしても女の子は逃げれる

841 名前:808投稿日:04/01/07 01:25
俺は男が進行方向を変えるケースは考えるに値しないと思うが、

もし仮に男が進行方向を変えて来るような馬鹿な奴だったら、
女の子はジグザグにこいで岸へ向かえば、男は反対側の岸でうろうろしてんじゃない?w


842 名前:824投稿日:04/01/07 01:26
>>832
>んなことないでしょ。
んなことあるよ。矢印2の向きへ進み出して、痴漢に>>828
戦略を取られたら、女の子は向きを変えない訳にはいかない。

向きを変えたら、角速度の速い痴漢の方が勝つ。


843 名前:824投稿日:04/01/07 01:28
>>841
>女の子はジグザグにこいで岸へ向かえば、男は反対側の岸でうろうろしてんじゃない?w
ジグザグの極限を取ると、矢印1の向きになるはず。


844 名前:808投稿日:04/01/07 01:31
>>843
そのとき男は反対側の岸でうろうろしている、と。
それでいいのか?w

845 名前:あ(794)投稿日:04/01/07 01:32
そーか2chねらーは皆小学生だったのか

846 名前:808投稿日:04/01/07 01:56
>>842
>矢印2の向きへ進み出して、痴漢に>>828
>戦略を取られたら

こんなの考える必要あるの?
明らかに損してるじゃん。
こんな馬鹿な男が相手だったらどんなに早くても逃げ切れるよ。

っていうか「人」のイメージに惑わされてない?
数学的に円と2つの点を考えれば

>「男が池の中心を見据えた時に女の子が左側にいる限りは時計回りに、
> 池の中心を見据えた時に女の子が右側にいる限りは反時計回りに追う」

男の戦略はこれ以外考えられないんだが。
この戦略に対して逃げ切れるかどうかで上限速度が求まるはず。

847 名前:132人目の素数さん投稿日:04/01/07 02:00
ちなみにこの問題って男の加速度の上限は無し?

848 名前:132人目の素数さん投稿日:04/01/07 02:01
じゃあ、その場合の女の子の戦略は
「女の子が池の中心を見据えたときに男が延長線上に居るように逃げる」
ですか?

849 名前:132人目の素数さん投稿日:04/01/07 02:03
>>848
それが可能なのは例の図で女が”内”に居るとき

850 名前:808投稿日:04/01/07 02:03
>>848
内円周を出たら男の方が角速度が速くなるから
常に真反対岸に男を見据えることは不可能でしょ。

851 名前:132人目の素数さん投稿日:04/01/07 02:04
ってか此の問題、池の形変えて類似問題いっぱい作れそう

852 名前:132人目の素数さん投稿日:04/01/07 02:07
ちなみに螺旋の線の長さって求まる?

853 名前:132人目の素数さん投稿日:04/01/07 02:11
>>852
求まるのは求まるんじゃね

854 名前:132人目の素数さん投稿日:04/01/07 02:15
>>833ってダメ?
円の半径と同じ長軸半径を持つ楕円軌道をまわれば1周にかかる時間が少なくなる
ってことはより外側をまわっても痴漢-池の中心-女の子となる位置関係に来れると思うんだけど

855 名前:132人目の素数さん投稿日:04/01/07 02:16
>>852
逃げ切るのに要した時間が求まれば、女の子の速度と掛けるだけで求まりそう。


856 名前:132人目の素数さん投稿日:04/01/07 02:21
>>855
逃げるのに要する時間を求めるのに使いたかったんだが

857 名前:808投稿日:04/01/07 02:22
>>854
男が>>383の戦略をとってる場合は駄目。
女が内円周を出た瞬間から「角 男-中心-女」は単調減少しつづけるため、
内円周の外部でこれが180度になることは有り得ない。

858 名前:808投稿日:04/01/07 02:24
>>383じゃなくて>>838の間違いだ。

>「男が池の中心を見据えた時に女の子が左側にいる限りは時計回りに、
> 池の中心を見据えた時に女の子が右側にいる限りは反時計回りに追う」

こいつね。

859 名前:132人目の素数さん投稿日:04/01/07 02:30
>>856
いずれにせよこの問題の解は螺旋にはならないんじゃん?

860 名前:132人目の素数さん投稿日:04/01/07 02:30
>>857
それは円軌道を回っている場合でしょ?
楕円軌道なら角速度も増えない?

861 名前:808投稿日:04/01/07 02:32
>>860
円軌道が一番角速度早いでそ

862 名前:808投稿日:04/01/07 02:36
んなことないか。
あ、そうか、楕円軌道だと中心からの距離が減少するのか。
そうなるとどうなるんだ・・・?

ちょっと時間を。

863 名前:132人目の素数さん投稿日:04/01/07 02:36
痴漢が超高速で走れる時、

  池  の  水  面  を  も  走  れ  る  !!

分かったか低脳ども

864 名前:132人目の素数さん投稿日:04/01/07 02:38
>>861
んーなんか説明しずらいんだけど
池のなかのある点pから中心に点対称な点qに行くのにぐるっと円を描くより
楕円のほうが近道な気がする

865 名前:808投稿日:04/01/07 02:42
なんかちょっと混乱してた。

>池のなかのある点pから中心に点対称な点qに行くのにぐるっと円を描くより
>楕円のほうが近道な気がする

確かにそうなんだけど、
内円周の外にいる限りは楕円的に進もうが直線的に進もうが
絶対角速度は男より遅いから、やはり無理かと。

866 名前:132人目の素数さん投稿日:04/01/07 02:53
>>865
そうなのかあ・・・
寝ながらもっかいよく考えてみまつ。
おやすみ〜

867 名前:808投稿日:04/01/07 02:56
男は「角 男-中心-女」を減少させる向きに動いている。

女の角速度が男の角速度より早ければ、この角度を増加させることができるし、
男の角速度が女の角速度より速ければ、この角度は減少していく。

内円周の外部では女はどの方向に動いても常に「女の角速度<男の角速度」なので、
内円周の外部で「角 男-中心-女」を180度にすることはできない。

----------
こんな感じかな。
間違ってたらスマソ。
俺も寝よ。

868 名前:132人目の素数さん投稿日:04/01/07 03:03
>池のなかのある点pから中心に点対称な点qに行くのにぐるっと円を描くより
>楕円のほうが近道な気がする

わあい円に勝った、じゃ意味ないと。
男の角速度相手に勝たないと意味ないぞと。

869 名前:132人目の素数さん投稿日:04/01/07 03:50
まあ、もまえらよーく聞けここんとこが一番大事だからな
痴漢しようって男がπとか楕円とか
理 解 で き る わ き ゃ ね ー だ ろ

870 名前:132人目の素数さん投稿日:04/01/07 09:58
>>869
理解できそうな高学歴者も結構居るぞ。
http://perape.hp.infoseek.co.jp/column/d00/colm_d11npls.htm


871 名前:789投稿日:04/01/07 10:03
楕円だと、男に遠日点?で待たれてしまうのでだめだと思う。
楕円でいいなら、極限として中心を通る線分往復運動になるけど、
たとえばその軌跡自体を回転させてゆくとしても、その回転自身が
男の角速度より速くないと。

872 名前:132人目の素数さん投稿日:04/01/07 13:38
これ、昔、数セミに出てた問題だな。
たしか、
陸上での女の子の速度 > 痴漢の速度
って条件もついてたはず。

873 名前:132人目の素数さん投稿日:04/01/07 14:01
昔考えたとき、池からの脱出にかかる時間を最大にする痴漢の戦略は、
・池の中心、女の子、痴漢が直線上にないときは、
痴漢は女の子との角度を小さくする方向に走る。
・池の中心をはさんで、女の子と痴漢が逆側にいるときは、
痴漢はどちらかの方向に走る。
と、なってた。


874 名前:872=873投稿日:04/01/07 14:38
女の子が、池からの脱出にかかる時間を最小にするためには、
池の中心、女の子、痴漢が直線上にあるとき以外は、直線的に動くべき。
(適当な証明)
女の子が点Aから点Bへ直線以外の経路Cで動くのが最良の戦略になったとする。
(経路Cの途中および端点で、池の中心、女の子、痴漢は直線上に並ばないとする)
しかし、AからBへ直線的に動けば、当然Bへはより早く到達することができ、
Bに到達した時点での女の子と痴漢の角度は、経路Cで動いた場合より大きい。
女の子にとっては、Bに到達した時点での痴漢との角度が大きいほうがいいので、
直線的な経路のほうが経路Cより優れている。

875 名前:132人目の素数さん投稿日:04/01/07 14:53
だからもう
>>806-807
で答え出てるじゃん

876 名前:872=873投稿日:04/01/07 15:23
話の流れが見えないけど、池の中心から脱出する場合は、まず、
女の子−池の中心−痴漢 の直線を保ったままある半径のところまで螺旋状に動いて、
そこから直線的に岸を目指すのが最短経路だと思う。
証明はめんどくさいから省略。

877 名前:872=873投稿日:04/01/07 15:42
連続カキコスマソ。
>>875
俺も >>806-807 は正しいと思うけど、これはxの上限を求めてるだけでしょ?
今問題になってるのは最短経路の話のほうだし。
俺は、池の任意の地点からの最短の脱出経路に興味があるんだけど、
誰か考えてくれない?

878 名前:132人目の素数さん投稿日:04/01/07 15:51
池の中心から脱出する場合の最短経路は、
女の子−池の中心−痴漢 の直線を保ったまま内円周のふちまで螺旋状に動き、
内円周から少し出たとこで一瞬待ち、
男と逆向きに進行方向をθ°変えて直線的に岸を目指す経路。

男の速度が 4.1415・・・ (=1+π) 以下ならば θ=0
男の速度が 4.6033・・・ (tan(θ)=θ+π、x=1/cos(θ)の解) ならば θ=90

男の速度がこれより速くなると脱出不可能。




それぞれの場合の最短経路の長さは誰か頼む。

879 名前:132人目の素数さん投稿日:04/01/07 17:19
内円周の内部で螺旋的に動く部分の経路は結構簡単に求まりそうだな。

要するに、女のθ方向の速度(角速度)と男の速度(角速度)の差が、
女のr方向の速度に充てられるわけだ。
三平方の定理使うかな。

んで、そのr方向の速度を内円周の半径分積分すれば、
内円周のふちに到着するまでの時間が求まる。

時間が求まれば距離も求まる。

880 名前:132人目の素数さん投稿日:04/01/07 17:22
>>878
それは男の速度が1+π以上のときでしょう。
例えば男の速度がπ以下ならば最初から一直線でいい。

881 名前:132人目の素数さん投稿日:04/01/07 18:14
>>880
「あと残りは一直線に一番近い岸まで行けば、ぎりぎり逃げられるな」ってところまで、
内円周は小さくできるのか。

882 名前:132人目の素数さん投稿日:04/01/07 18:22
基本的には女の子−池の中心−痴漢 の直線を保ったまま螺旋状に動き、

男の速度が 4.1415・・・ (=1+π) 以下ならば、
「あとは一直線に進めばぎりぎり逃げられるな」というとこまで来たら一直線に逃げる。
(最初から一直線に逃げれば良い場合も含む)

男の速度が4.1415・・・ (=1+π) 以上ならば、
角速度が男と同じになるとこまで来たら一瞬待ち男と逆方向に適当に角度を付けて一直線に逃げる。

男の速度は 4.6033・・・ (tan(θ)=θ+π、x=1/cos(θ)の解) が上限速度。
このとき上記の角度は90°

これでいいのかな?

883 名前:132人目の素数さん投稿日:04/01/07 19:57
既出かもしれないけど

封筒が2つあります
片方にはもう片方の2倍の金が入っています
あなたは一方を選んだ
1万円入っていた
となるともう片方には2万円か5000円入っている事になります

この時点でもう片方の封筒と交換すれば、+1万か−5000円なので変えたほうが平均では得になる

じゃあ最初からもう片方を選んだほうが得なのか
否、どっちを選んでも一緒
という事はとにかく最初の一つを選んで、それを無条件で交換するほうが得なのか?
2番目に選んだほうが常に得なのか

説明お願いします

884 名前:132人目の素数さん投稿日:04/01/07 20:43
>>877
いや。俺は>>806-807ではダメだと思う。
上限値Xをもとめるということはもとめられているのは
(1)x<Xであるなら女のこの側にある戦略があってその戦略で逃走すれば
痴漢がどんな追跡しても逃走できること。
(2)x>Xであるなら女のこがどんな戦略をとっても痴漢に捕捉されるか永遠に
脱出できないかのいづれかであること。
の2つをしめさないとだめ。まず(1)については>>806-807で示されてるのは
男の行動が
  
>「男が池の中心を見据えた時に女の子が左側にいる限りは時計回りに、
> 池の中心を見据えた時に女の子が右側にいる限りは反時計回りに追う」
 
この戦略にしたがってる場合のみであってたとえば
 
「男は1秒ごとにサイコロふって奇数なら時計回り、偶数なら反時計回りにするむ。」
 
みたいな戦略をとってる場合には成立しない。どんな追跡をうけても逃げきれることを
示さないといけないなら一見不合理な↑このような戦略でも逃避できることをしめさないと
だめだと思う。
(2)の部分については手付かずじゃないの?

885 名前:789投稿日:04/01/07 21:03
>>884
806-807を書いた本人です。
何度か指摘されてるとおり(1)としての解にはなってないと、自分でも
思ってます。
逆に(2)は満たしてるのではないかと考えてます。
男の方の方針を考えると、
0. 女が中心付近、男より速く回れる内周円内にいるとする。
1. 女が内周円からでた瞬間に、男-中心を結ぶ直線の右か左かにより
女がいる側に走り始める。線上にいたらどちらでも。
2. 女が内周円内に戻ったら、0にもどる。
3. 女が内周円内に戻らず、上陸するなら必ず追いつける。
という状態になっていると思います。

886 名前:132人目の素数さん投稿日:04/01/07 21:15
>>884
だからさあ

>「男が池の中心を見据えた時に女の子が左側にいる限りは時計回りに、
> 池の中心を見据えた時に女の子が右側にいる限りは反時計回りに追う」

この戦略に対して逃げ切ることができるのであれば、
他のどんな戦略に対してでも逃げきれるに決まってるじゃん。
自明。

887 名前:132人目の素数さん投稿日:04/01/07 21:20
>>885
男が>>885の戦略で追跡した場合に男の速度が
4.6033・・・ (tan(θ)=θ+π、x=1/cos(θ)の解)より真に大きいとき女のこがどんな
軌道で脱出を試みても逃避できないことの部分の証明がいまいちよく分かりません。
もう少し詳しくおながいします。

888 名前:132人目の素数さん投稿日:04/01/07 21:26
>>886
そんなことはない。たとえば女のこの逃避の戦略が
>>806-807のような戦略なら、つまり
 
>(1) 女の子は、池の中心半径1の円周上で、痴漢-池の中心-女の子が一直線となる
>位置をとる。 *1
>(2) 女の子は、(1)の直線に沿って池の周方向へ漕ぎはじめる。
>(3) 痴漢は、女の子が池の中心半径1の円から外へαだけ進んだ時点で右周り
>(左でもいい)に池の周に沿って走りはじめる。 *2
 
であってそのあと男がどういう行動をとろうと逃避方向を変化させないなら
男の追跡戦略が>>884のような「サイコロ戦略」をとっていて途中で追跡方向を変化させた場合
捕捉されてしまう。女のこが無事逃避するためには男の行動にしたがって逃避方向を
変化させないといけない。男の行動にしたがってどのように自分の逃避行動を変化させるかの
方法を具体的にしめすかなんかしないと解答として十分ではないと思う。

889 名前:132人目の素数さん投稿日:04/01/07 21:48
>>888
男は上限速度で追いかけるとする。

>>806-807で示された最適戦略(男女とも)に従えば、
「女の中心からの距離」と「男-中心-女の角度」は1対1対応する。

これは
「女が中心からある距離にいるとき、男-中心-女の角度がいくら以上であれば逃げ切れるか」の
ギリギリのラインを示している。

だからもし男が反転して「男-中心-女の角度」を増やすような真似をしたら、
なんのことはない、「女の中心からの距離」と「男-中心-女の角度」のバランスが取れるまで、
外向きにこいでれば良い。

バランスが取れたら最適戦略に従い、向きをあらためて決定すれば良い。

890 名前:132人目の素数さん投稿日:04/01/07 21:55
>>889
それだと男が「停止する」という選択をとったとき動けないのでは?
あとこれより男の速度が大きいと女のこは逃げられないことの証明がよくわからないんですが。

891 名前:132人目の素数さん投稿日:04/01/07 21:55
うーん、わかりにくいかなあ・・・

要するに、男が反転したら「男-中心-女の角度」が増えるから
その分「余裕」が出来るんだよ。
「余裕」があるうちは悠々と外向きにこいでれば良い。
ただそれだけの話なんだが。

892 名前:132人目の素数さん投稿日:04/01/07 22:01
>>890
男が停止したら、女は悠々と外を目指せばよい。
角度に余裕があるうちは。

893 名前:132人目の素数さん投稿日:04/01/07 22:02
>>891
いや、逃避可能ってところはわからないでもないんですが。
なんか「自明」とかなんとかいわれてもそこまで「自明」な気はしないです。
やっぱりキチンと証明しないと。もちろん掲示板のひまつぶしなんで試験の解答とか
論文とかで通用するほどの証明を要求するつもりはないけど掲示板のひまつぶしという枠内ですら
通用するレベルの証明にはなってない気が。

894 名前:872=873投稿日:04/01/07 22:04
>>891
十分わかりやすいかと。
>>890
男が停止したときにも「余裕」が生じるわけだから、
その余裕の許すかぎり外向きにこいでいればいい。

895 名前:893投稿日:04/01/07 22:09
うーん。オレに賛同してくれる人はいないみたい。まあオレ自身あの戦略で
キチンと逃げられることの証明になってるといえなくもないような気はするけど。
(というか自分のノートにはきちんと証明してみたんですが書くのメンドイ)
それよりあれが上限であることの証明は結構ムズイ。だれかきちんと手元のノートに
まとめてみた人いる?

896 名前:132人目の素数さん投稿日:04/01/07 22:10
>>890
>あとこれより男の速度が大きいと女のこは逃げられないことの証明がよくわからないんですが。

逆に聞くけど、>>818の図でどの地点に向かう?
で、そこへは直線で向かわないの?

(男は常に時計回りで追いかけるとする。)

897 名前:132人目の素数さん投稿日:04/01/07 22:16
>>896
オレに聞かれても・・・俺は男の速度が
4.6033・・・ (tan(θ)=θ+π、x=1/cos(θ)の解)より真に大きいとき
には女のこは逃避しえないことの証明がわからないんだけで具体的に反例をもってるわけじゃないよ。

898 名前:132人目の素数さん投稿日:04/01/07 22:21
>>897
内円周の外部では常に「男-中心-女の角度」は単調減少。
これがゼロになったらアウト。

だからまず内円周の内部で180°にしてスタート。

内円周の外に出たら男は時計回りに1方向で追いかけてくる。

女は上陸地点を定めたらそこへ直線で進むのが得策。
0°から90°の範囲で考えればよい。
その結果90°進行方向を曲げるのが一番いいとのこと。

899 名前:132人目の素数さん投稿日:04/01/07 22:22
女がくねくね進んだら端的に損でしょ。

900 名前:132人目の素数さん投稿日:04/01/07 22:26
>>898-899
だめだ。わからん。もう諦めた。もう解決ということでいいです。

901 名前:132人目の素数さん投稿日:04/01/07 22:28
>>900
説明下手くそですまん。
目の前で図を書いて手でなぞったりすれば分かってもらえる自信はあるんだが。

902 名前:789投稿日:04/01/07 22:41
>>887
まず、女が逃げ切るにはどこかに上陸しなければならない。
今、男は>>885のように、一方向にしか追ってこないので、内周円を
出た時点で上陸点は決まる。そしてその上陸点への男が到達する
時間は決まっているので、女はそこに最も早くつく(短い)軌跡を
とるのがよい。
そしてそれは、自分の前方(内周円の自分の位置での接線でより前)
であれば自分の位置と上陸点を結ぶ直線。後方であれば、内周円に
そって回っていって接線が上陸点にあたるところから直線となる。
後者は、内周円上での角速度が男と同じなので、結局最初の位置から
接線に沿って進んだのと同じであり、前者だけ考えれば十分。

いま、男と女の速度をそれぞれx,1とし、池の半径をxとすると男の
最大角速度は1、女が男より早く回れる内周円の半径も1となる。
上陸点を、女と池の中心を結ぶ直線からθだけ回転した位置にとると、
男がそこに到達するまでの最大時間はπ+θ、女がそこに到達する
最短時間は√((x*sin(θ))^2+(x*cos(θ)-1)^2)である。
したがって
F=(π+θ)^2-((x*sin(θ))^2+(x*cos(θ)-1)^2)
が負ならば男は追いつける。
∂F/∂θ=2*(π+θ-x*sin(θ))はx>4.66033...の時には
x*cos(θ)<1(前方)の範囲内で0になり、Fはそこで最大値
F=-(x*cos(θ)-1)^2 <0!
となる。つまりx>4.66033...の時には>>885の方針で、
男は必ず追いつけることになる。

903 名前:132人目の素数さん投稿日:04/01/07 22:53
ひとくぎりついたのかな?出題してみる。
-問題-
fが下に凸な関数。実数列a0<a1<・・・<anとb0・・・bnが
(i)b0,bn>0
(ii)巴i=0,蚤ibi=0
をみたすとき巴if(ai)≧0を示せ。
――
某スレの問題かんがえてるとき思いついた問題。たぶん成立してるとおもうけど。
そんなむずかしくないけどもう1000間近なので手ごろな問題ということでどうでしょう?

904 名前:132人目の素数さん投稿日:04/01/07 23:00
>>883はどう?

905 名前:132人目の素数さん投稿日:04/01/07 23:08
>>904
既出。過去ログ嫁

906 名前:132人目の素数さん投稿日:04/01/07 23:13
>>904
というかなんでこのスレで質問するの?

907 名前:132人目の素数さん投稿日:04/01/08 22:27
>>903
どっかでみた問題

908 名前:中川泰秀投稿日:04/01/09 08:53
{正}24角形を用いて、円周率が3.1になることを証明せよ。

909 名前:132人目の素数さん投稿日:04/01/09 08:54
¾

910 名前:132人目の素数さん投稿日:04/01/09 09:07
1〜9の整数を1個づつもちいて
□x□x□x□x□+□x10+□+□-□=1997


911 名前:132人目の素数さん投稿日:04/01/09 10:47
>>910
2x4x5x6x8+7x10+1+9-3=1997

912 名前:132人目の素数さん投稿日:04/01/09 16:17
>>911
アンタ、、、すげぇぜ

913 名前:132人目の素数さん投稿日:04/01/09 21:05
1から9までの数字を全部使って数を表すことを
「小町算」といいます。

(例) 1×2×34×5×6-7×8×9=1975
987+6×54×3+21=1980

このように1から9までの数を「増加の順」か「減少の順」に
並べて適当に記号(今回は+、−、×、÷の4種類)を入れ、
年数を表すのです。

では問題です。
今年、2004年を小町算で表してみてください。



914 名前:132人目の素数さん投稿日:04/01/09 21:11
>>913
1987+3×6+5−2−4=2004


915 名前:132人目の素数さん投稿日:04/01/09 21:18
>>914
1〜9までの数字が「増加の順」か「減少の順」でなければいけません。

916 名前:132人目の素数さん投稿日:04/01/09 21:29
1−2+345×6+7−8×9=2004

917 名前:132人目の素数さん投稿日:04/01/09 21:30
  1+2+3/4*5*67*8-9
  1+2+345*6-78+9
  1-2+345*6+7-8*9
  1-2+345*6-7*8-9


918 名前:132人目の素数さん投稿日:04/01/09 22:27
>>917
これで全解。降順の解はないみたいだね。

コード書いてるうちに先越されて悔しいから、去年のぶんをw(来年は解なし)
2003
=12+345*6-7-8*9
=12+34*56+78+9
=12+3*456+7*89
=1*2+345*6-78+9
=1*2+3/4*5*67*8-9
=1+23+45*6*7+89
=1+2+34*56+7+89

919 名前:917投稿日:04/01/09 22:32
うるさかったから、ネットで調べただけ。
あなたの方が偉いよ。
調べた限りでは、あらゆる値について検索済みのようだったよ。例えば来年のも、、。
100から始まったらしい。

920 名前:132人目の素数さん投稿日:04/01/09 22:35
>>919
みつけたURLおながいしまつ

921 名前:917投稿日:04/01/09 22:38
(―1+2+3)×((―4―5+6)+7×8×9)=2004


922 名前:917投稿日:04/01/09 22:39
>>http://www2.ocn.ne.jp/~mizuryu/toukou2/nissi61.html

923 名前:917投稿日:04/01/09 22:50
元の検索先はどっかいった。
しかし、これをプログラミングの課題として出す教授がいて、
その言わばレポートの内容(プログラム)なのだが、それもネットにある。


924 名前:132人目の素数さん投稿日:04/01/11 10:44
1〜6の整数を1個ずつ用い、
+、−、×、÷、累乗を使い、解が2004になるようなものをあげよ。

925 名前: ◆BhMath2chk 投稿日:04/01/11 12:00
2×(5+6−1)^3+4=2004。


926 名前:132人目の素数さん投稿日:04/01/13 17:04
[x]をxを超えない最大の整数であるとし、{x}=x-[x]と定義する。
この時、任意の正の数nに対して
{ n*√3 } > a/( n√3 )
を満たす、最大の実数aを求めよ。

927 名前:132人目の素数さん投稿日:04/01/13 22:39
Aに「Bに『Cに〜と聞いたら〜と答えるか?』と聞いたら〜と答えるか?」と聞く。


こういうのが答えになるような正直者嘘つき者問題って作れないかな。
昔から考えてるんだけど思い浮かばない。

928 名前:132人目の素数さん投稿日:04/01/13 22:54
>>926
あ、間違えた。

[x]をxを超えない最大の整数であるとし、{x}=x-[x]と定義する。
この時、任意の正の整数nに対して
{ n*√3 } > a/( n√3 )
を満たす、最大の実数aを求めよ。


が正しい

正の数じゃなくて正の整数ね。
出典は某国数学オリンピック2000年の問題だ。

929 名前: ◆BhMath2chk 投稿日:04/01/14 02:00
正の整数nにたいして√(3)nの整数部分をmとすると
m^2≦3n^2でm^2=3n^2やm^2+1=3n^2とはならないので
m^2+2≦3n^2になる。
 (√(3)n−m)√(3)n
≧(√(m^2+2)−m)√(m^2+2)
=m^2+2−m√(m^2+2)
>m^2+2−m(m+1/m)
=1。
よってa=1は条件を満たす。
3・1^2−1^2=2で
3(2a+b)^2−(3a+2b)^2=3a^2−b^2なので
3n^2−m^2=2はいくらでも大きな整数解を持つ。
n,mを3n^2−m^2=2を満たす正の整数にとると
 (√(3)n−m)√(3)n
=m^2+2−m√(m^2+2)
<m^2+2−m(m+1/m−1/2m^3)
=1+1/2m^2。
mはいくらでも大きく取れるのでa≦1。
よって条件を満たすaの最大値は1。


930 名前:132人目の素数さん投稿日:04/01/15 21:50
n個からr個とりだす組合せの数をC(n,r)で表す。
nを3以上の整数とするとき、次式の和を求めよ。
 Σ[k=3 to n] 1/C(k,3)

931 名前:132人目の素数さん投稿日:04/01/16 19:56
>>930
3[1/2-1/{n(n-1)}]。
1/{(n-1)n(n+1)} = [1/(n-1) - 2/n + 1/(n+1)} / 2 で計算した。

932 名前:132人目の素数さん投稿日:04/01/16 21:11
>>927
こんなのどうでしょう。
道が2つに分かれていて、片方は天国に、もう片方は地獄に続いています。
3人の人がいて、1人は嘘つき、1人は正直者と反対の答えをするアンチ正直者、1人は嘘つきと反対の答えをするアンチ嘘つきです。
質問を1つだけして必ず天国に行くにはどうしたらいいでしょう?

答えは、Aに「Bに『Cにこっちが天国かと聞いたらはいと答えるか?』と聞いたらはいと答えるか?」と聞いて、答えがはいならそっちに、いいえなら違う方に行く。

933 名前:132人目の素数さん投稿日:04/01/16 23:45
>>932
む、なんか面白そう。

934 名前:932投稿日:04/01/17 01:19
正直者、嘘つき、アンチ正直者でもよかったかも

935 名前:132人目の素数さん投稿日:04/01/17 01:35
>>932
えと、「嘘つき」と「アンチ正直者」って何が違うの?

936 名前:132人目の素数さん投稿日:04/01/17 02:17
>>935
「『こっちが天国か』と聞かれたらあなたはYESと答えますか?」

指した道が実際に天国に通じる道だとすると、

嘘つきはYES
アンチ正直者はNO

937 名前:132人目の素数さん投稿日:04/01/17 08:47
こういうスレもある。

正直族と嘘つき族
http://science2.2ch.net/test/read.cgi/math/1053610505/

938 名前:132人目の素数さん投稿日:04/01/17 23:19
次のパラドックスの謎を解いて下さい。

あなたの目の前に、封印された封筒Aと封筒Bが置かれています。
片方には確実にもう片方の2倍の金額のお金が入っています。外からでは中にいくら入って
いるかはわかりません。
あなたはどちらか好きな方の封筒を1つ持ち帰ることができます。ただし家に帰るまで
中身を見てはいけません。

さて、あなたは今封筒Aを手に取りました。
封筒Aに入っている金額をN円とすると、封筒Bには2*N円か1/2*N円がそれぞれ1/2の確率で
入っていることになります。
ということは、封筒Bに入っている金額の期待値は
1/2*2*N + 1/2*1/2*N = 5/4*N円
となります。したがって封筒Aを戻して封筒Bを持ち帰った方が有利です。

ところが、もし最初に手に取ったのが封筒Bだとしたらどうでしょうか。上と全く同じ理由で
封筒Aが有利になってしまいます。

939 名前:132人目の素数さん投稿日:04/01/17 23:22
>>938
http://science2.2ch.net/test/read.cgi/math/1046437047/205-

940 名前:132人目の素数さん投稿日:04/01/17 23:47
>>938 あるいは、
http://science2.2ch.net/test/read.cgi/math/1040540700/
の327ぐらいから。

941 名前:132人目の素数さん投稿日:04/01/18 01:10
>>939-940
そうでしたか・・・

942 名前:132人目の素数さん投稿日:04/01/18 01:25
すみません、超ーーー長くなりますが質問です。
元の文章は三浦俊彦「論理学入門」日本放送出版協会、2000年、970円です。

この本では、「宇宙人はいない可能性が高い」という論証をしており、
めっちゃ面白いのですが、
途中、どうしてもよく理解できないところがあり、解説をお願いしたいのです。

p186-p190の要旨をこちらに転載します。

943 名前:132人目の素数さん投稿日:04/01/18 01:25
-------------------------------------------------------------
生命誕生から(宇宙間の通信ができる)
電波による通信が可能な文明の確立まで、約40億年が経過している。

太陽の寿命は約100億年だが、
惑星系に生命に適した環境を許すであろう期間はもう少し限定される。
地球環境のさまざまな変化も考慮に入れると、
地球型惑星で生命が進化し続けられる全期間は太陽の寿命よりもかなり短い。

すると、天文学的スケールで見ると、
「地球型惑星が生命そして文明を育みうる時間のタイムリミット」と
ほとんど同じだけの時間が経ってから、地球上にようやく通信文明が実現したことになる。

大まかに言って、知的生命に至るまでの生物進化のペースと、
惑星環境の安定期とが、なぜか地球の場合、一致している。
つまり人類は、いわば「ギリギリで間に合った」のである。

この事実は何を物語っているだろうか?

期限内にきわめて起こりにくい種類のことがたまたま起きてしまった場合、
それは期限切れギリギリになって起こる可能性が一番高い。

(とても合格しそうにない学力の者が多数受験して
 たった1人だけ偶然に受かった場合、
 彼はほぼ間違いなく、合格基準最低点すれすれで受かっているはずだろう)

私たち人類はきわどい選択効果の産物だったのだろう。
かくして、地球外知的生命は存在しそうにない。
-------------------------------------------------------------

944 名前:132人目の素数さん投稿日:04/01/18 01:26
わからないのは
「期限内にきわめて起こりにくい種類のことがたまたま起きてしまった場合、
 それは期限切れギリギリになって起こる可能性が一番高い」という部分です。

たとえ話は理解できるのに、なお理解できません。
たとえ話の「学生の得点が合格点ギリギリ」はわかるのですが、
それが「期限ギリギリ」にも適用できるという理屈がわかりません。

たとえば販売期間が1年間で当選番号を即日発表する宝くじは、
1月1日に買って当たる確率も、12月31日に買って当たる確率も同じように思えます。
というわけで、よろしくお願いします。

945 名前:132人目の素数さん投稿日:04/01/18 02:30
>>944
上の文での
「期限内にきわめて起こりにくい種類のことがたまたま起きてしまった」
というのは、
「宝くじのように単純に確率的に起こりにくい」ということではなく、
「期限が長ければ起こりうるのに期限が短いために起こりにくい」ということなのでは?

こういうたとえ話はどうだろう。

制限時間30分以内に円周率を100桁覚えてもらう。
覚えた人は30分たってなくてもその場で手をあげてもらう。
覚えられなかったら退場してもらう。
このような実験を何回も何回も行ったが誰も成功しない。(=期限内にきわめて起こりにくい)
が、しかしここにたまたま成功した人が現れたとする。
たぶんこの人は制限時間ギリギリに手をあげたのだろう。


946 名前:132人目の素数さん投稿日:04/01/18 08:12
上手い説明だな。俺942じゃないけど。

947 名前:132人目の素数さん投稿日:04/01/18 08:33
でも宇宙は広いからいてもおかしくないよね。

948 名前:中川泰秀投稿日:04/01/18 09:04
1角形{いちかっけい}とを何かを説明せよ。

949 名前:中川泰秀投稿日:04/01/18 09:06
上記は、三角形とか五角形に対する、{1角形}ですよ。

950 名前:132人目の素数さん投稿日:04/01/18 09:59
>>945 さん

942-944です。
945さんのたとえ話だと、今度は期限の話になっていますね。
これですっきりしました。ありがとうございました。

951 名前:945投稿日:04/01/18 22:16
>>950
どもども。

952 名前:132人目の素数さん投稿日:04/01/19 00:20
950です。ありがとうございました。>>951 さん
お礼がてら、三浦俊彦「論理パラドクス−論証力を磨く99問」二見書房、2002年10月2日、
1500円+税、からコピペして紹介します。

---------------------------------------------------------
これまでに生まれた人間はみな、身長3メートル以下だった。
よって、これから生まれる人間もみな、3メートル以下だろう。
これは多数のサンプルから一般的真理を導く帰納法で、合理的な推論である。

さて、同様の帰納的推論により、次のように論証してみよう。

これまでに生まれた人間はみな、西暦3000年以前に生まれた。
よって、これから生まれる人間もみな、西暦3000年以前に生まれる。
つまり、西暦3000年以降に人間は生まれない。したがって、人類の滅亡は近い。

この推論は、形としてはちゃんとした帰納法になっているように思われる。
しかし間違った推論であることも明らかだ。

しかし、どこが間違っているのか。
見えすいた詭弁なのに、ちゃんと答えようとすると案外難しい。
明確に述べてみよう。
-------------------------------------------------------------------

953 名前:945投稿日:04/01/19 00:31
>>952
>これまでに生まれた人間はみな、身長3メートル以下だった。

(人間の身長は時代とともにさほど変化しないだろう)

>よって、これから生まれる人間もみな、3メートル以下だろう。


>これまでに生まれた人間はみな、西暦3000年以前に生まれた。

(人間の生まれ年は時代とともに変化する)

>よって、これから生まれる人間もみな、西暦3000年以前に生まれる。 とは言えない。


括弧の中身が違う、と。
こんなもんでどうでしょ?

954 名前:132人目の素数さん投稿日:04/01/19 00:33
偏ったサンプル?

955 名前:132人目の素数さん投稿日:04/01/19 00:37
調べる対象となる性質を持っている可能性と、
サンプルとして選ばれる可能性との間に
強い相関関係があってはまずいわけだな。

956 名前:132人目の素数さん投稿日:04/01/19 00:48
風呂から出てきたらもう正解がカキコされているので、びっくりした952です。

>>953-955さん
みなさん正解です。さすが数学板、読んでて気持ちいいです。
いちおう本の正解をコピペしておきます。

[答え]

帰納法になってはいるが、サンプルが偏っているので、無効である。

「これまでに生まれた人間はみな、西暦3000年以前に生まれた」というのは、真実だ。
1人の例外もないので、それこそ帰納法のサンプルとしては申し分ないように思われる。

しかしもちろんそうではない。

私たちが観察できる人間は、西暦3000年以前に生まれた人間であることが定められている。(中略)
帰納法の正しい通用のためには、観察されたものが、
たまたま観察されなかったものも含めた全体を代表する典型
(ランダムなサンプル)でなければならないのである。

よく使われる比喩を挙げよう。
湖で綱を使って1日中魚をとり続けたところ、どの魚も体長1cm以上であった。
この湖にいる魚は全て体長1cm以上である、と結論してよいか。

そのためには、網の細かさを確かめなければならない。

網の目が、1cm未満の魚は洩れてしまうほどの粗さであったなら、
いくらすくい上げても体長1cm以上の魚だけが選択されてくる。(p125-p126)

957 名前:132人目の素数さん投稿日:04/01/19 00:52
sin(x-a)-sin(x)=0のとき
xをaを用いてあらわせ

958 名前:132人目の素数さん投稿日:04/01/19 01:11
>>957
aが0の時しかなりたたんような


959 名前:953投稿日:04/01/19 01:11
>>956
なるほどね。
その網の例は分かりやすくていいね。

こういうのもっときぼん。

960 名前:132人目の素数さん投稿日:04/01/19 01:41
>>959さん

めちゃくちゃな長さになりますし、三浦俊彦の本のパクりなので気が引けるのですが、
あまりにも面白いので、おそるおそるコピペさせていただきます。

三浦 俊彦「論理サバイバル―議論力を鍛える108問」二見書房、2003年5月25日、1500円+税、ISBN4-576-03077-9。

ある恐ろしい実験が行われています。
最初に部屋の中に10人を閉じ込め、サイコロを2つ振ります。
そして・・・。

[実験T]

6−6以外の目が出たら、その10人を釈放し、次に別の100人を部屋に送り込む。
またサイコロを振って、同じ手続きにしたがう。

この要領で、N番目には新たな(10のN乗)人の人間を部屋に送り込んで、
6−6が出れば全員射殺しておしまい。6−6が出るまで続く実験というわけだ。

部屋はとてつもなく広い(あるいは、人数が増えると自動的に拡がる)ので、
被験者たちの人数がどんなに増えようとも収容できる。

あなたはいま、自分がこの実験に選ばれて、部屋にいることがわかった。
あなたが射殺される確率はいくらだろうか。

961 名前:132人目の素数さん投稿日:04/01/19 01:41
[正解]

まわりを見回して、部屋に100人いるのか、1万人か、1億人いるのかを見きわめることで推測しようとしても無駄である。
あなたが何番目のグループに入っているとしても、あなたのいるグループが射殺される確率は、1/36だ。3%弱。
だから心配するには及ばない。

962 名前:132人目の素数さん投稿日:04/01/19 01:42
ところがこのクイズは、さらにすこし変形して続くのです。

963 名前:132人目の素数さん投稿日:04/01/19 01:43
[実験U]

実験者がTのように1回ごとにサイコロを振るのではなく、
初めにまとめて振って、6−6が出るまでの系列を記録しておいた。

たとえば1回目は3−5、2回目は1−6・・・というふうに目の列を記録したのである。
そのうえでTと同様の手順でN回目に(10のN乗)人を部屋に送り込みつづけ、
6−6が対応している回において全員射殺し、実験終了。

このような設定のもとで、あなたはいま、
自分がこの実験に選ばれて、部屋にいることがわかった。
あなたが射殺される確率はいくらだろうか。

964 名前:132人目の素数さん投稿日:04/01/19 01:43
[正解]

今度は、Iとは事情が違っている。

ここでは、全部で何人がこの部屋に入れられるかが初めから決まっている。
その決まった人数のうちの、大多数が射殺されることもわかっている。

たとえば3回目に6−6が出たのであれば1110人中1000人射殺、
6回目に6−6ならば1111110人中1000000人射殺。

少なくとも90%が射殺されるのである。

とすれば、この実験のために選ばれた人間のうち、
あなたは別段えこひいきされているわけではないだろうから、
多数派に属する、つまり最後の回のグループに割り振られている可能性が一番高い。

あなたが射殺される確率は90%以上である。

965 名前:132人目の素数さん投稿日:04/01/19 01:44
さて、長ーーーーい準備の後で、いよいよ問題です。

よく似た状況なのに、TとUでは、射殺される確率が「3%未満」対「90%以上」と劇的な差がある。
これは変ではないだろうか?

サイコロをいつ振るかということは、
6−6が出る確率そのものには何の変化ももたらさないはずである。
なのにこの違い。

TとUでこれほど確率の相違が出るのはどうしてだろう。
理由を述べてください。

966 名前:132人目の素数さん投稿日:04/01/19 02:00
1と2で考えている確率が違うから
1では、その回で6-6の出る確率を考え、2では自分が射殺される回に割り振られる確率を考えているから

967 名前:132人目の素数さん投稿日:04/01/19 02:03
IIでは、サイコロの振られる回数の上限が
決定されてるということが本質的に違うんだな。

968 名前:132人目の素数さん投稿日:04/01/19 02:21
あまりにも面白いかもしれないが、あまりにもガイシュツだな。

969 名前:132人目の素数さん投稿日:04/01/19 03:15
>>966
それは出題者の求める答えになってなくないか?

970 名前:132人目の素数さん投稿日:04/01/19 08:42
なんか変だな・・・

問題に不備があるのか?
人口無限とかそこらへんにタネがあるのかしら

971 名前:132人目の素数さん投稿日:04/01/19 09:55
IIの実験は、例えば100000人が殺される予定になった場合、
その人が被験者となった瞬間に母集団111110人に含まれることが限定されてしまうから。
(111111人目以降はそもそも呼ばれない)

全ての人間を、10人・100人・1000人…のグループに振り分ける、という作業から考えた場合、
100000人のグループに含まれる可能性よりも111…111011111人に含まれる可能性のほうが
遥かに高いから、あまり心配することはない。


972 名前:132人目の素数さん投稿日:04/01/19 16:25
らいおんの家

973 名前:132人目の素数さん投稿日:04/01/19 23:23
>>972さん その通り。そこからコピペです。

答え

Tの場合は、実験がオープンである。Uの場合はクローズドだ。どういうことか。
Tの場合は、実験が現在進行形なのである。

あなたが部屋に入れられたときには、実験はまだ終わっていない。
つまり、永遠に6−6が出ずに、人数ばかりが10倍10倍に増えつつ
いつまでも銃が火を吹かない可能性が残っているのである。
無限に人間が増えてゆくかもしれないのだ。

一方、日の場合は、6−6が出るまでサイコロを振りつづけ、
6−6が出てから、人々を部屋に入れ始めている。
つまり、いつかは6−6が出ることが保証されている。
これが決定的な違いだ。

現実にはTの場合も必ず6−6が出るときがくるはずだが、
理論的にはいつまでも6−6以外の目ばかり出続けることはありうる。
現実的ではないが、理論的なこの可能性が、
Tの場合に「無限に続くかもしれない…‥・」楽観主義を正当化するのである。
Uではその望みはない。

したがって自分はまず9割方、多数派すなわち殺され組に入っているだろう、
という推測が正しい。

Tの場合にはその多数派(殺され組)がまだ単なる可能性に過ぎないので、
実は永遠に存在しないままかもしれないのである。(pp209-212)

974 名前:132人目の素数さん投稿日:04/01/20 03:05
実験1で、
10000回連続で6-6が出なかったら10000回目に部屋にいた連中はみな殺す。

という条件がついたらどうなる?

975 名前:132人目の素数さん投稿日:04/01/20 03:07
つまりこういう感じ



6−6以外の目が出たら、その10人を釈放し、次に別の100人を部屋に送り込む。
またサイコロを振って、同じ手続きにしたがう。

この要領で、N番目には新たな(10のN乗)人の人間を部屋に送り込んで、
6−6が出れば全員射殺しておしまい。6−6が出るまで続く実験というわけだ。

ただし、10000回連続で6-6が出なかったら10000回目に部屋にいた連中はみな殺す。

あなたはいま、自分がこの実験に選ばれて、部屋にいることがわかった。
あなたが射殺される確率はいくらだろうか。

976 名前:132人目の素数さん投稿日:04/01/20 03:52
コレ分かる人いる?(携帯用URLでスマソ;)
http://game4.2ch.net/test/r.i/quiz/1074337999/9

977 名前:132人目の素数さん投稿日:04/01/20 06:00
http://game4.2ch.net/test/read.cgi/quiz/1074337999/9


978 名前:132人目の素数さん投稿日:04/01/20 06:18
>>977
★なぞなぞ★その2
9 :( ・∀・)つ〃∩ヘェーヘェーヘェー :04/01/18 09:43

ある人が2から99までの2つの数を考えた。その数の積と和をそれぞれP氏(積)とS氏(和)に伝えた。すると二人の間で以下のような会話がなされた。
P: 2つの数は分からない。
S: 私も分からないが、あなたがそう言うのは分かっていた。
P: 分かった。
S: 私も分かった。
ではこの数はなにか。どのようにして2つの数が分かったのか?

979 名前:132人目の素数さん投稿日:04/01/20 06:20
>>978
異なる2数という前提だろうね。

Pがすぐに分からなかったから、2つの素数の積ではない。


980 名前:976投稿日:04/01/20 09:29
Pに二つの数が分からない事をSは分かったから、二つの素数の和でもないんだな。
ゴールドバッハ予想(200ぐらいまでは確認済だろ)により二つの数の和は奇数だと分かる。
よって二つの数の片っぽは奇数、もう片っぽは偶数。
…あとは┐(´〜`)┌ワカンネ

981 名前:976投稿日:04/01/20 11:39
分かった、と言うか見付けた。二数が4と13なら会話が成立するね。
Pが教えられた数字(積)は52。これから考えられる二数は(2,26)と(4,13)の二種類。
ところが前者だとSが「君も分からないって事は分かったよ」なんて言えないから、解は後者だとPに分かる。
Sが教えられた数(和)は17。これから考えられる二数は(2,15)(3,14)(4,13)…(8,9)の七種類。
ところが(4,13)以外の六種類を考えると、いずれもPが二数を「分かる」事はできない。
よって解は(4,13)だとSにも分かる。

982 名前:976・続き投稿日:04/01/20 11:47
ただ、この方法だとめっちゃ面倒くさい。
二つの素数の和とならない数を、小さい方からシラミ潰しに調べていっただけだからなぁ。
それに、他に解を持つ可能性もある。

983 名前:132人目の素数さん投稿日:04/01/20 14:40
976氏のやり方でいいんじゃないかな。
P: 2つの数は分からない。
S: 私も分からないが、あなたがそう言うのは分かっていた。

ここまでで2数の和Sは、
6<=S<=196、Sは奇数かつSは2+素数でない。

P: 分かった。

2数の積Pは奇数×偶数に一意に分解される(他は偶数×偶数となる。)
→P=4p (pは素数)、2数は4とp

S: 私も分かった。

2+pは素数でない。
したがって2数は(4,p), p=7, 13,19, 23...

違うかな。

984 名前:132人目の素数さん投稿日:04/01/20 22:09
>>975
それだと、終わりがあるわけだから、
実験2と同じような考え方を適用できると思う。

つまり死ぬ確率は90%超。

985 名前:983投稿日:04/01/20 22:16
2数の積Pは奇数×偶数に一意に分解される(他は偶数×偶数となる。)
→P=4p (pは素数)、2数は4とp
これから、間違ってるな。他にも間違ってるけど。すまん。

986 名前:132人目の素数さん投稿日:04/01/20 22:40
なかなかいい問題だね。
全くわかんあいけど

987 名前:132人目の素数さん投稿日:04/01/20 23:10
yahoo ( in English ) watcher
http://science2.2ch.net/test/read.cgi/math/1020334993/
これの81から4と13。


988 名前:132人目の素数さん投稿日:04/01/21 03:14
>>973
オープンだとかクローズだとか言ってるのは、いかにも論理学者っつうか哲学者っぽい説明だね。
俺はこの例ははじめて見たけど、数学的に説明すると次の通り。

これはただの囚人の考え方の違い、すなわち事前確率の違いとして説明される。
Xを、囚人が選ばれた番目の試行回数、Nを試行が終わった回数とすると、今求めているのは、Xがわかった
ときの、N=Xである条件付確率の、Xの分布による期待値を求めたい、ということ。

すなわちE(P(N=X|X))=E(P(N=X))=E(P(X=N|N))を求めたい。

基本的にはNの分布は、幾何分布p*q^(n-1) (p=1/36,q=1-p)に従うとわかっているので、Nの方が条件付けやすく、
ΣP(X=N|N=n)p*q^(n-1)を求めたい、ってことになる。

2番目の例では、P(X=N|N=n)=P(X=n|N=n)、すなわち試行がN回で終わったとしたとき、自分がN回目にいる
確率を、10^n/(10+…+10^n)と考えている。

n回で終わったのを確認し、10+…+10^n人を人数で一様に選ぶ、という仮定をおいている、ということ。
この場合は、その期待値ΣP(X=N|N=n)p*q^(n-1)も9割以上だ。
(ちゃんとは求まらないが、10^n/(10+…+10^n)→1、Σp*q^(n-1)=1だから、そんなかんじ)

ただ、これは勝手に囚人がそう選ぶのが普通だ、と考えているだけで、人数で一様ではなく、それぞれの回数で一様
に選ばれた、という可能性だってある。すなわち、自分がn回までのどの順番にいるかはランダムだと考え、
P(X=N|N=n)=1/nと設定するという考えもある。
期待値はちょっと難しい和(Σ(1/n)*p*q^(n-1))になるけど、これはかなり小さくなることが分かるでしょう。

989 名前:132人目の素数さん投稿日:04/01/21 03:14
もうひとつの考え方は、普通に毎回自分が選ばれる確率は最初っから決まっており、試行終了回数が決まってから決
まるものではない、と考えた場合で、この場合は、E(P(X=N))=pとなり、これが1番目の例である。

具体的には、例えば自分がk-1回目までは選ばれないで、k回目に選ばれる確率がp(k)で、Nに関係ないととした場合、
P(X=n,N=n)は、n回目で試行が終了し、かつ自分がn回目に選ばれる確率だから、
P(X=n,N=n)=(1-p(1))…(1-p(n-1))p(n)*pq^(n-1)。
また、P(X=n)=(1-p(1))…(1-p(n-1))p(n)*q^(n-1)だから、P(N=X|X=n)=p、すなわちE(P(X=N))=pである。

このように、事前情報をどのように考える確率に組み込むかによって確率は変わってくるのが普通で、これはその
顕著な例。ぱっと見、一般的と考えうる事前確率の設定が複数ある場合によくパラドックスとして引用される。
3囚人とかモンテシージレンマとかも全部そう。

990 名前:989投稿日:04/01/21 03:17
>>989 モンティーホールジレンマだな、最後。なんだよモンテシーってw

991 名前:132人目の素数さん投稿日:04/01/21 13:46
>>987
結局この問題は、手計算じゃ難しいのかな。
あきらめて、しらみ潰しで調べるとたしかに(4,13)しかないけど、
うまく説明できない。数を1000以下としても他に(4,61)が増えるだけみたい。
片方は4くらいは説明できるかな。

992 名前:132人目の素数さん投稿日:04/01/22 09:50
ヽ(´ー`)ノ次スレは?

993 名前:132人目の素数さん投稿日:04/01/22 15:01
面白い問題おしえてーな 八問目
http://science2.2ch.net/test/read.cgi/math/1074751156/

994 名前:132人目の素数さん投稿日:04/01/22 20:07
>>993
どうもありがと

995 名前:132人目の素数さん投稿日:04/01/23 09:54
|-`)

996 名前:132人目の素数さん投稿日:04/01/23 19:57
c⌒っ*゚ー゚)っうめ

997 名前:132人目の素数さん投稿日:04/01/23 23:12
plum

998 名前:132人目の素数さん投稿日:04/01/23 23:27
(゚∀゚)キュンキュン

999 名前:132人目の素数さん投稿日:04/01/23 23:28
'A`)

1000 名前:132人目の素数さん投稿日:04/01/23 23:28
1000キタ━━━━━(゚∀゚)━━━━━ !!!

1001 名前:1001投稿日:Over 1000 Thread
このスレッドは1000を超えました。
もう書けないので、新しいスレッドを立ててくださいです。。。


DAT2HTML 0.26 Converted.